You are on page 1of 78

Jawetz Medical Microbiology 27th ed.

(D) Regulation of genes involved in diverse


physiologic processes
SECTION 1 (E) All of the above

Chapter 1: The Science of Microbiology 8. A 16-year-old female patient presented to her


family physician with a complaint of an abnormal
1. Which one of the following terms characterizes vaginal discharge and pruritus (itching). The patient
the interaction between herpes simplex virus and a denied having sexual activity and recently
human? completed a course of doxycycline for the treatment
(A) Parasitism of her acne.An examination of a Gram-stained
(B) Symbiosis vaginal smear revealed the presence of gram-
(C) Endosymbiosis positive oval cells about 4–8 μm in diameter.
(D) Endoparasitism Her vaginitis is caused by which of the following
(E) Consortia agents?
(A) Bacterium
2. Which one of the following agents lacks nucleic (B) Virus
acid? (C) Protozoa
(A) Bacteria (D) Fungus
(B) Viruses (E) Prion
(C) Viroids
(D) Prions 9. A 65-year-old man develops dementia,
(E) Protozoa progressive over several months, along with ataxia
and somnolence. An electroencephalographic
3. Which one of the following is a prokaryote? pattern shows paroxysms with high voltages and
(A) Bacteria slow waves, suggestive of Creutzfeldt-Jakob
(B) Algae disease (CJD). By which of the following agents is
(C) Protozoa this disease caused?
(D) Fungi (A) Bacterium
(E) Slime molds (B) Virus
(C) Viroid
4. Which one of the following agents simultaneously (D) Prion
contains both DNA and RNA? (E) Plasmid
(A) Bacteria
(B) Viruses 10. Twenty minutes after ingesting a raw clam, a
(C) Viroids 35-year-old man experiences paresthesias of the
(D) Prions mouth and extremities, headache, and ataxia.
(E) Plasmids These symptoms are the result of a neurotoxin
produced by algae called
5. Which of the following cannot be infected by (A) Amoeba
viruses? (B) Blue-green algae
(A) Bacteria (C) Dinoflagellates
(B) Protozoa (D) Kelp
(C) Human cells (E) None of the above
(D) Viruses
(E) None of the above Chapter 2: Cell Structure

6. Viruses, bacteria, and protists are uniquely 1. A 22-year-old man presents with a painless 1-cm
characterized by their respective size. True or ulcer on the shaft of his penis. Inguinal
false? lymphadenopathy is present. The patient admits
(A) True trading drugs for sex and has several sexual
(B) False partners. An RPR test result is positive, and syphilis
is suspected; however, a Gram stain of a swab
7. Quorum sensing in prokaryotes involves specimen from the ulcer shows no bacteria.
(A) Cell–cell communication Treponema pallidum, the causative agent of
(B) Production of molecules such as acetylated syphilis, cannot be visualized by light microscopy
homoserine lactone (AHL) because
(C) An example of multicellular behavior (A) It is transparent.
(B) It cannot be stained by ordinary stains.
Jawetz Medical Microbiology 27th ed.
(C) It has a diameter of less than 0.2 μm. 7. In the fall of 2001, a series of letters containing
(D) The wavelength of white light is too long. spores of Bacillus anthracis were mailed to
(E) Rapid movement of the organism prevents members of the media and to U.S. Senate offices.
visualization. The result was 22 cases of anthrax, with five
deaths. The heat resistance of bacterial spores,
2. Chloramphenicol, an antibiotic that inhibits such as those of Bacillus anthracis, is partly
bacterial protein synthesis, will also affect which of attributable to their dehydrated state and partly to
the following eukaryotic organelles? the presence of large amounts of
(A) Mitochondria (A) Diaminopimelic acid
(B) Golgi complex (B) d-Glutamic acid
(C) Microtubules (C) Calcium dipicolinate
(D) Endoplasmic reticulum (D) Sulfhydryl-containing proteins
(E) Nuclear membrane (E) Lipid A

3. Which of the following structures is not part of the 8. Which of the following terms does NOT describe
bacterial cell envelope? the bacterial chromosome?
(A) Peptidoglycan (A) Haploid
(B) Lipopolysaccharide (B) Diploid
(C) Capsule (C) Circular
(D) Gas vacuole (D) Nucleoid
(E) S-layer (E) Feulgen positive

4. A group of teenagers became ill with nausea, 9. Lysozyme cleaves the β1→4 linkage between
vomiting, severe abdominal cramps, and diarrhea
(A) d-Alanine and the pentaglycine bridge
after eating undercooked hamburgers from a local
(B) N-Acetylmuramic acid and d-alanine
restaurant. Two of the teenagers were hospitalized
(C) Lipid A and KDO
with hemolytic-uremic syndrome. Escherichia coli
(D) N-Acetylmuramic acid and N-acetylglucosamine
O157:H7 was isolated from the patient’s stools as
(E) d-Alanine and d-alanine
well as from uncooked hamburgers. The H7 refers
to which bacterial structure?
10. Mycoplasma species lack which of the following
(A) Lipopolysaccharide
components?
(B) Capsule
(A) Ribosomes
(C) Flagella
(B) Plasma membrane
(D) Fimbriae
(C) Both DNA and RNA
(E) S-layer
(D) Lipids
(E) Peptidoglycan
5. Which of the following components is present in
gram-negative bacteria but not in gram-positive
Chapter 3: Classification of Bacteria
bacteria?
(A) Peptidoglycan
1. Eubacteria that lack cell walls and do not
(B) Lipid A
synthesize the precursors of peptidoglycan are
(C) Capsule
called
(D) Flagella
(A) Gram-negative bacteria
(E) Pili
(B) Viruses
(C) Mycoplasmas
6. Group A streptococci are the most common
(D) Serovar variant
bacterial cause of pharyngitis in school-age children
(E) Bacilli
5–15 years of age. The most important cell
component involved in adherence of this bacteria to
2. Archaebacteria can be distinguished from
fibronectin, which covers the epithelial surface of
eubacteria by their lack of
the nasopharynx is
(A) DNA
(A) Capsule
(B) RNA
(B) Lipoteichoic acid
(C) Ribosomes
(C) Flagella
(D) Peptidoglycan
(D) Lipoprotein
(E) Nucleus
(E) O-antigen
3. A 16-year-old cystic fibrosis patient is admitted to
the hospital. A sputum culture yields Burkholderia
Jawetz Medical Microbiology 27th ed.
cepacia. Subsequently, there are two other patients a lag of 20 minutes, the E coli enter the logarithmic
with B cepacia bacteremia, and the organism is phase of growth. After 3 hours of logarithmic
cultured from the sputum of four additional patients. growth, the total number of cells is
During this nosocomial outbreak of B cepacia, 50 (A) 2560
environmental and 7 patient isolates are being (B) 5012
subtyped to identify the source of the outbreak. (C) 90
Which of the following techniques would be most (D) 1028
useful in this endeavor? (E) 1,000,000
(A) Culture
(B) Ribotyping 2. A 73-year-old woman is admitted to the hospital
(C) 16S rRNA sequencing for intravenous treatment of an abscess caused by
(D) Antimicrobial susceptibility testing Staphylococcus aureus. Subsequent to her
(E) Nucleic acid sequencing treatment and discharge from the hospital, it is
necessary to disinfect the hospital room. One
4. An unculturable gram-positive microorganism has thousand of the S. aureus cells are exposed to a
been visualized in tissue specimens obtained from disinfectant. After 10 minutes, 90% of the cells are
patients with a previously undescribed disease. killed. How many cells remain viable after 20
Which of the following techniques would be most minutes?
useful in identifying this organism? (A) 500
(A) Serology (B) 100
(B) PCR amplification and sequencing of rRNA (C) 10
genes (D) 1
(C) Multilocus enzyme electrophoresis (E) 0
(D) SDS-polyacrylamide gel electrophoresis
(E) Pulsed field gel electrophoresis 3. The action of which of the following agents or
processes on non-spore forming bacteria can be
5. The DNA polymerase from Thermus aquaticus is reversed?
an important component of DNA amplification (A) A disinfectant
methods such as the polymerase chain reaction. (B) A bactericidal agent
This organism is capable of growing at (C) A bacteriostatic agent
temperatures above 100°C. Organisms that are (D) Autoclaving at 121°C for 15 minutes
capable of growth at these temperatures are (E) Dry heat at 160–170°C for 1 hour
referred to as
(A) Mesophiles 4. The growth rate of bacteria during the
(B) Psychrophiles exponential phase of growth is
(C) Halophiles (A) Zero
(D) Thermophiles (B) Increasing
(E) Chemolithotrophs (C) Constant
(D) Decreasing
6. A bacteria with a genome having a G + C content (E) Negative
of 45% harbors a plasmid encoding a gene with a
G+ C content of 55%. Which one of the following 5. A physician obtains a sputum specimen from a
conclusion could be drawn? patient suffering from tuberculosis. This sputum
(A) This gene codes for a cell wall peptidyl sample contains one viable Mycobacterium
transferase tuberculosis, an organism with a slow doubling time
(B) This gene codes for a critical bacterial in vitro of 48 hours, which corresponds to a growth
cytochrome rate constant in vitro of (κ) of 0.04 h-1. Estimating
(C) This gene codes for a unique transfer RNA that the biomass of this single mycobacterial
(D) This gene codes for a plasmid RNA-dependent organism is 2.3 × 10-13 g and assuming that this
DNA polymerase organism is immediately going to enter log
(E) This gene codes for an antigenically diverse phase growth, how many hours will it take to
capsular polysaccharide produce 10-6 g of biomass?
(A) 4 hours
Chapter 4: Growth, Survival and Death (B) 40 hours
(C) 400 hours
1. A 23-year-old woman has 10 Escherichia coli (D) 4000 hours
inoculated into her bladder while having sex. This (E) 40,000 hours
organism has a generation time of 20 minutes. After
Jawetz Medical Microbiology 27th ed.
6. A specimen of pasteurized goat’s milk is cultured following inhibits energy-yielding cellular
for the presence of Brucella melitensis, an organism processes?
known to infect animals on an adjacent farm. The (A) 5-Methyltryptophan
milk is declared safe to consume; however, some of (B) Cyanide
those who consume it develop brucellosis. Which (C) Hydrogen peroxide
one of the following would best explain the disparity (D) Ethanol
between the culture results and the patients’ (E) Lysozyme
illnesses?
(A) Bacteria in the milk were viable but not 11. Which of the following is the most resistant to
cultivatable destruction by chemicals and heat?
(B) Incomplete pasteurization of the milk (A) Spores of Aspergillus fumigatus
(C) The organisms in the milk were in the lag phase (B) Mycobacterium tuberculosis
when tested (C) Ebola virus
(D) The milk had a high level of a bactericidal (D) E coli
antibiotic in it when tested (E) Spores of Bacillus anthracis
(E) There was posttest contamination of the milk
Chapter 5: Cultivation of Microorganisms
7. Working as a medical missionary in rural India,
you spray the umbilicus of a newly born infant with 1. Most microorganisms pathogenic for humans
a solution containing the chemical structure in the grow best in the laboratory when cultures are
associated figure to prevent a tetanus infection. incubated at
Which class of chemical agent does this structure (A) 15–20°C
belong? (B) 20–30°C
(C) 30–37°C
(D) 38–50°C
(E) 50–55°C

2. The process by which microorganisms form ATP


during the fermentation of glucose is characterized
by
(A) Coupling of ATP production with the transfer of
(A) Alcohol class electrons
(B) Aldehyde class (B) Denitrifcation
(C) Bisphenol class (C) Te reduction of oxygen
(D) Peroxygen class (D) Substrate phosphorylation
(E) Quaternary ammonium class (E) Anaerobic respiration

8. Your superior requests that you sterilize some 3. The major effect of a temperature of 60oC on the
surgical instruments. Which one of the following growth of a
agents would you use? mesophile such as Escherichia coli is to
(A) Benzoic acid (2%) (A) Destroy the cell wall
(B) Isopropyl alcohol (2%) (B) Denature proteins
(C) Glutaraldehyde (2%) (C) Destroy nucleic acids
(D) Hydrogen peroxide (2%) (D) Solubilize the cytoplasmic membrane
(E) Quaternary ammonium compound (2%) (E) Cause formation of endospores

9. The growth rate of bacteria during the maximum 4. Polymerization of building blocks (eg, amino
stationary phase of growth is acids) into macromolecules (eg, proteins) is
(A) Zero achieved largely by
(B) Increasing (A) Dehydration
(C) Constant (B) Reduction
(D) Decreasing (C) Oxidation
(E) Negative (D) Assimilation
(E) Hydrolysis
10. Chemical agents can interfere with the normal
reaction between a specific enzyme and its 5. A strain of E coli does not require vitamins when
substrate (chemical antagonism). Which one of the grown in a defined medium consisting of glucose,
Jawetz Medical Microbiology 27th ed.
mineral salts, and ammonium chloride. This is (B) Peptidoglycan
because E coli (C) Capsular polysaccharide
(A) Does not use vitamins for growth (D) Deoxyribonucleic acid
(B) Obtains vitamins from its human host (E) Phospholipids
(C) Is a chemoheterotroph
(D) Can synthesize vitamins from the simple 2. The synthesis of which of the following cell
compounds provided in the medium components is
(E) Ammonium chloride and mineral salts contain determined entirely by enzyme specificities?
trace amounts of vitamins (A) DNA
(B) Ribosomal RNA
6. Which of the following is NOT a mechanism for (C) Flagella
generating metabolic energy by microorganisms? (D) Lipopolysaccharide
(A) Fermentation (E) Protein
(B) Protein synthesis
(C) Respiration 3. The steps leading to the synthesis of
(D) Photosynthesis peptidoglycan occur in the
(E) C and D cytoplasm, on the cytoplasmic membrane, and
extracellularly.
7. Which of the following terms best describes a Which antibiotic inhibits an extracellular step in
microorganisms peptidoglycan
that grows at 20°C? biosynthesis?
(A) Neutralophile (A) Cycloserine
(B) Psychrotroph (B) Rifampin
(C) Mesophile (C) Penicillin
(D) Osmophile (D) Bacitracin
(E) Termophile (E) Streptomycin

8. The ability to assimilate N2 reductively via NH3 is 4. Amino acids are found in the protein,
called peptidoglycan, and capsule of bacteria. Which of
(A) Ammonifcation the following amino acids is found
(B) Anammox only in peptidoglycan?
(C) Assimilatory nitrate reduction (A) l-Lysine
(D) Deamination (B) Diaminopimelic acid
(E) Nitrogen fixation (C) d-Glutamate
(D) l-Alanine
9. Which of the following is NOT assimilated by (E) None of the above
eukaryotic cells?
(A) Glucose 5. The ability to use compounds and ions other than
(B) Lactate oxygen as
(C) Sulfate (SO42-) terminal oxidants in respiration is a widespread
(D) Nitrogen (N2) microbial trait.
(E) Phosphate (PO43-) Tis capacity is called
(A) Photosynthesis
10. Bacteria that are obligate intracellular (B) Fermentation
pathogens of humans (C) Anaerobic respiration
(eg, Chlamydia trachomatis) are considered to be (D) Substrate phosphorylation
(A) Autotrophs (E) Nitrogen fxation
(B) Photosynthetic
(C) Chemolithotrophs 6. The primary route of carbon assimilation used by
(D) Hyperthermophiles organisms
(E) Heterotrophs that can use CO
2 as a sole source of carbon is
Chapter 6: Microbial Metabolism (A) Hexose monophosphate shunt
(B) Entner-Doudoroff pathway
1. The synthesis of which of the following cell (C) Embden-Meyerhof pathway
components is (D) Glyoxalate cycle
dependent on a template? (E) Calvin cycle
(A) Lipopolysaccharide
Jawetz Medical Microbiology 27th ed.
7. The peptidoglycan biosynthetic pathway is of Chapter 7: Microbial Genetics
particular importance in medicine because it 1. Mutations in bacteria can occur by which of the
provides a basis for selective antibacterial action of following
several chemotherapeutic agents. All of the mechanisms?
following antibiotics inhibit steps in peptidoglycan (A) Base substitutions
biosynthesis EXCEPT (B) Deletions
(A) Cycloserine (C) Insertions
(B) Vancomycin (D) Rearrangements
(C) Bacitracin (E) All of the above
(D) Streptomycin
(E) Penicillin 2. The form of genetic exchange in which donor
DNA is introduced to the recipient by a bacterial
8. The regulation of enzyme activity provides fne virus is
control of metabolic pathways. Which of the (A) Transformation
following regulatory mechanisms (B) Conjugation
provides fne control of a biosynthetic pathway? (C) Transfection
(A) Catabolite repression (D) Transduction
(B) Induction (E) Horizontal transfer
(C) Feedback inhibition
(D) Attenuation 3. The enzyme DNAse degrades naked DNA. If two
(E) None of the above strains of bacteria from the same species were
mixed in the presence of DNAse,
9. The biosynthetic origin of building blocks and which method of gene transfer would be most likely
coenzymes can inhibited?
be traced back to relatively few precursors called (A) Conjugation
focal metabolites. Which of the following are focal (B) Transduction
metabolites? (C) Transformation
(A) α-Ketoglutarate (D) Transposition
(B) Oxaloacetate (E) All of the above
(C) Phosphoenolpyruvate
(D) Glucose 6-phosphate 4. Replication of which of the following requires
(E) All of the above physical integration with a bacterial replicon?
(A) Single-stranded DNA bacteriophage
10. Which of the following is NOT a component of (B) Double-stranded DNA bacteriophage
peptidoglycan? (C) Single-stranded RNA bacteriophage
(A) N-Acetyl muramic acid (D) Plasmid
(B) N-Acetyl glucosamine (E) Transposon
(C) Lipid A
(D) Pentaglycine 5. The formation of a mating pair during the process
(E) Diaminopimelic acid of conjugation in Escherichia coli requires
(A) Lysis of the donor
11. Which of these pathways gives a cell the (B) A sex pilus
potential to produce (C) Transfer of both strands of DNA
the most ATP? (D) A restriction endonuclease
(A) TCA cycle (E) Integration of a transposon
(B) Pentose phosphate pathway
(C) Glycolysis 6. Why do bacteria contain restriction enzymes?
(D) Lactic acid fermentation (A) To cleave RNA for incorporation into ribosome
(E) Entner-Doudoroff pathway (B) To extend the length of bacterial chromosomes
(C) To prevent foreign DNA from incorporating into
12. During the process of oxidative phosphorylation, a bacterial genome
the energy of (D) To process the exons from prokaryotic mRNA
the proton motive force is used to generate (E) To proteolytically cleave nuclear promoters
(A) NADH
(B) ADP 7. If the arrangement of bases on the coding DNA
(C) NADPH strand is
(D) Acetyl CoA 5′CATTAG3′, then a corresponding strand of mRNA

(E) ATP will be


Jawetz Medical Microbiology 27th ed.
which one of the following? (D) Phagocytosis
(A) 5′GTAATC3′ (E) Presentation of pathogen to helper T cells
(B) 5′CUAAUG3′
(C) 5′CTAATG3′ 7. Within the innate immune response, this cell acts
(D) 5′GUAAUC3′ by killing virus-infected cells:
(E) 5′CATTAG3′ (A) T cell
(B) NK cell
(C) Macrophage
(D) Neutrophil
Section II Immunology
(E) B cell
Chapter 8: Immunology
1. The immunoglobulin class most frequently 8. The interaction of two IgG molecules binding to
responsible for inhibition of bacteria on mucosal antigen followed by the binding of C1 to the Fc
surfaces is: portion of the antibody results in which of the
(A) IgG following?
(B) IgM (A) Initiation of antigen presentation
(C) IgA (B) Initiation of classic complement pathway
(D) IgE (C) Initiation of alternative complement pathway
(E) IgD (D) Initiation of the mannose-binding lectin-binding
complement pathway
2. As part of the innate immune response, which
cells participate in phagocytosis? 9. What is a characteristic of the adaptive immune
(A) Macrophages and mast cells response and not of the innate response?
(B) Macrophages and plasma cells (A) Physical barriers
(C) NK cells and neutrophils (B) Chemical barriers
(D) Macrophages and neutrophils (C) Clonal expansion of effector cells
(E) T cells and mast cells (D) Inflammatory mediators
3. Which of the cytokines attract neutrophils and (E) Phagocytosis
inhibit bacteria?
(A) IFN-γ 10. Which genetic mechanism generates antibodies
(B) IL-8 of the same specificity but different immunoglobulin
(C) IL-2 classes?
(D) IL-6 (A) V gene segment recombination
(E) TGF-β (B) Class switching
(C) Somatic hypermutation
4. MHC class II molecules are critically important in (D) Junctional variability due to imprecise V, D, and
what immunologic process? J joining
(A) Antigen presentation (E) Gene duplication, ie, multiple V, D, and J gene
(B) Phagocytosis segments
(C) Immunoglobulin class switching
(D) CD8+ T cell cytotoxicity 11. Which major antibody molecule has the ability
(E) Opsonization to cross the placenta?
(A) IgG
5. MHC class I molecules are critically important in (B) IgA
what immunologic process? (C) IgM
(A) IgE-mediated histamine release (D) IgE
(B) Phagocytosis (E) IgD
(C) Immunoglobulin class switching
(D) CD8+ T-cell cytotoxicity 12. A man in his twenties presents in the
(E) Opsonization emergency room with shortness of breath and
fatigue. He is also very pale. Two days earlier he
6. The host response to the interaction of a was given penicillin for an infection. He had
pathogen with its specific TLR generates which of penicillin previously without problems and stated
the following? that he had “no allergy” to penicillin. Laboratory
(A) IgG production testing shows that antibodies to penicillin are
(B) Cell activation and production of cytokines and present in the patient’s serum and that he is
chemokines breaking down his own red blood cells. He is
(C) Immunoglobulin class switching
Jawetz Medical Microbiology 27th ed.
diagnosed with immune hemolytic anemia. The an in vitro counterpart of the type IV hypersensitivity
patient has which type of hypersensitivity reaction? reactions
(A) Type I seen in the TB skin test?
(B) Type II (A) Immunoblot for TB antigen
(C) Type III (B) EIA analysis of sera from TB patient
(D) Type IV (DTH) (C) Immunofluorescence assay for TB antibody
(D) IFN-γ production by white blood cell (WBC)
13. Which one of the following cell types expresses treated with
receptors for IgE on its cell surface that stimulate TB antigen
the cell to mount a response to parasites such as
worms? 20. Which of the following laboratory assays can be
(A) T cells used to detect
(B) B cells the number and types of immune cells in the
(C) NK cells peripheral blood?
(D) Mast cells (A) Immunofixation electrophoresis
(E) Dendritic cells (B) EIA
(C) Flow cytometry
14. NK cells express a killer immunoglobulin-like (D) Immunoblot
receptor (KIR), which recognizes:
(A) MHC class I molecules
(B) Cell adhesion molecules
Section III Bacteriology
(C) Glycophospholipid molecules
(D) CD40 molecules Chapter 9: Pathogenesis of Bacterial Infection

15. Prior to class switching, all B cells bound to 1. A 22-year-old woman who works in a plant
antigen have which of the following antibody nursery presents with a history of fever and cough
classes on their surface? for 2 months. Over this period of time she has lost 5
(A) IgA kg. Chest radiography shows bilateral upper lobe
(B) IgG infiltrates with cavities. A stain of her sputum shows
(C) IgM acid-fast bacilli. The likely means by which the
(D) IgE patient acquired her infection is
(A) Sexualactivity
16. IgE-mediated histamine release is classifed as (B) Ingesting the microorganisms in her food
what type of hypersensitivity reaction? (C) Holding onto contaminated hand rails when she
(A) Type I takes
(B) Type II public transportation
(C) Type III D) Handling potting soil
(D) Type IV (E) Breathing aerosolized droplets containing the
microorganism
17. IFN-α and IFN-β are produced by the virus-
infected cell due to 2. During a pandemic of a well-characterized
the interaction of virus nucleic acid with which of the disease, a group of 175 airline passengers flew
following? from Lima, Peru, to Los Angeles. Lunch on the
(A) C3 (third component of complement) plane included crab salad, which was eaten by
(B) Defensins about two- thirds of the passengers. After landing in
(C) TLR pathway Los Angeles, many of the passengers transferred to
(D) IL-12 other flights with destinations in other parts of
California and other Western states. Two of the
18. Which two cytokines play an important role in passengers who stayed in Los Angeles developed
attracting neutrophils to the site of an infection? severe watery diarrhea. The status of the other
(A) IFN-α and IFN-γ passengers was unknown. The likely cause of the
(B) IL-8 and IL-17 diarrhea in the two passengers is
(C) IL-2 and IL-4
(D) IL-6 and IL-12 (A) Escherichia coli O157:H7 (lipopolysaccharide O
antigen 157; flagellar antigen 7)
19. Which one of the following laboratory assays is (B) Vibrio cholerae type O139 (lipopolysaccharide
considered to be O antigen 139)
(C) Shigella dysenteriae type 1
Jawetz Medical Microbiology 27th ed.
(D) Campylobacterjejuni evidence of moderate renal failure. This patient’s
(E) Entamoeba histolytica illness was likely to be caused by which of the
following?
3. A 65-year-old woman has a long-term central (A) Lipopolysaccharide
venous catheter for intravenous therapy. She (B) Peptidoglycan
develops fever and subsequently has multiple blood (C) A toxin that is a superantigen
cultures positive for Staphylococcus epider- midis. (D) A toxin that has A and B subunits
All of the S epidermidis isolates have the same (E) Lecithinase (alpha toxin)
colony morphology and antimicrobial susceptibility
pattern, suggest- ing that they are the same strain. 7. The organism most likely to be responsible for
An S epidermidis biofilm is thought to be present on the patient’s disease (Question 6) is
the catheter. Which one of the follow- ing (A) E coli
statements about such an infection is correct? (B) Corynebacterium diphtheriae
(C) Clostridium perfringens
(A) The biofilm containing the S epidermidis is (D) Neisseria meningitidis
likely to wash off the catheter. (E) Staphylococcus aureus
(B) Production of an extracellular polysaccharide
inhibits growth of the S epidermidis, limiting the 8. Which of the following is most likely to be
infection. associated with the formation of a bacterial biofilm?
(C) The S epidermidis in the biofilm are likely to be (A) Airway colonization in a cystic fibrosis patient
more susceptible to antimicrobial therapy because with a mucoid (alginate-producing) strain of P
the bacteria have decreased rates of metabolism. aeruginosa
(D) The quorum-sensing ability of S epidermidis (B) Urinary tract infection with E coli
results in increased susceptibility to antimicrobial (C) Meningitis with N meningitidis
therapy. (D) Tetanus
(E) The complex molecular interactions within the (E) Impetigo caused by S aureus
biofilm make it difficult to provide effective
antimicrobial therapy, and it is likely the catheter will 9. Regarding bacterial type III secretions systems,
have to be removed to cure the infection. which of the following statements is correct?
(A) They are commonly found in gram-positive
4. The first microorganism to satisfy Koch’s commensal
postulates (in the late 19th century) was bacteria.
(A) Treponemapallidum (B) They play an important role in the pathogenesis
(B) Stenotrophomonas maltophilia of toxin-induced diseases of Clostridium species,
(C) Mycobacterium leprae tetanus, botulism, gas gangrene, and
(D) Bacillusanthracis pseudomembranous colitis.
(E) Neisseria gonorrhoeae (C) They cause release of effectors of
pathogenesis into the extracellular environment,
5. Which of the following statements about promoting bacterial colonization and multiplication.
lipopolysaccharide is correct? (D) They directly inject bacterial proteins into host
(A) It interacts with macrophages and monocytes cells across bacterial and host cell membranes,
yielding release of cytokines. promoting pathogenesis of infections.
(B) The toxic component is the O side chain. (E) Mutations that prevent the bacterial type III
(C) It forms holes in red blood cell membranes secretion from functioning enhance pathogenesis.
yielding
hemolysis. 10. Which of the following statements is correct?
(D) It causes hypothermia. (A) Lipopolysaccharide is part of the cell wall of E
(E) It causes paralysis. coli.
(B) Cholera toxin is attached to the flagella of V
6. A 27-year-old man had a rhinoplasty. A nasal cholerae.
tampon was placed to control the bleeding. (C) The lecithinase of C perfringens causes
Approximately 8 hours later, he developed diarrhea.
headache, muscle aches, and abdominal cramps (D) Toxic shock syndrome toxin-1 is produced by
with diarrhea. He then developed an erythematous hemolytic stains of S epidermidis.
rash (resembling sunburn) over much of his body,
including the palms and soles. His blood pressure is 11. A 15-year-old Bangladeshi girl develops severe
80/50 mm Hg. The nasal tampon remained in place. watery diarrhea.The stool looks like “rice water.” It is
His liver enzyme tests were elevated, and there was voluminous—more than 1 L in the last 90 minutes.
Jawetz Medical Microbiology 27th ed.
She has no fever and seems otherwise normal (B) Degrades lecithin in eukaryotic cell membranes
except for the effects of loss of fluid and (C) Causes release of tumor necrosis factor
electrolytes. The most likely cause of her illness is (D) Inhibits elongation factor 2
(E) Causes increased adenylate cyclase activity
(A) Clostridiumdifficileenterotoxin
(B) A toxin with A and B subunits Chapter 10: Normal Human Microbiota
(C) Shigella dysenteriae type 1 that produces
Shiga toxin 1. A 26-year-old woman visits her physician
(D) Enterotoxigenic E coli that produces heat-labile because of an unusual vaginal discharge. On
and heat- examination, the physician observes a thin,
stable toxins homogeneous, white-gray discharge that adheres
(E) Staphylococcal enterotoxin F to the vaginal wall. The pH of the discharge is 5.5
(normal, <4.3). On Gram stain, many epithelial cells
12. The most important thing that can be done to covered with gram-variable rods are seen. Bacterial
treat the patient (Question 11) is vaginosis is diagnosed. Which one of the following
(A) To give her ciprofloxacin normal genital flora microorganisms is greatly
(B) To give her a toxoid vaccine decreased in bacterial vaginosis?
(C) To give her the appropriate antitoxin (A) Corynebacterium species
(D) To treat her with fluid and electrolyte (B) Staphylococcus epidermidis
replacement (C) Prevotella species
(E) To culture her stool to make the correct (D) Candida albicans
diagnosis and then (E) Lactobacillus species
treat specifically
2. Certain microorganisms are never considered to
13. A 23-year-old woman has a history of recurrent be members of the normal flora. They are always
urinary tract infections, including at least one considered to be pathogens. Which one of the
episode of pyelonephritis. Blood typing shows the P following organisms fits into that category?
blood group antigen. Which of the following is likely (A) Streptococcus pneumoniae
to be the primary cause of her infections? (B) Escherichia coli
(A) E coli that produce heat-stable toxin (C) Mycobacterium tuberculosis
(B) E coli with K1 (capsular type 1) antigen (D) Staphylococcus aureus
(C) E coli O139 (lipopolysaccharide O antigen 139) (E) Neisseria meningitidis
(D) E coli with P-pili (fimbriae)
(E) E coli O157:H7 (lipopolysaccharide O antigen 3. A 9-year-old girl develops fever and severe pain
157; flagellar on the right side of her throat. On examination,
antigen 7) redness and swelling in the right peritonsillar area
are seen. A peritonsillar abscess is diagnosed. The
14. A 55-year-old man presents with gradually most likely organisms to be cultured from this
increasing weight loss, abdominal pain, diarrhea, abscess are
and arthropathy. During the evaluation process, a (A) S.aureus
small bowel biopsy is done. After process- ing, (B) S. pneumoniae
examination of the specimen by light microscopy (C) Corynebacterium species and Prevotella
reveals periodic acid-Schiff–positive inclusions in melaninogenica
the bowel wall. Which of the following tests could be (D) Normaloralnasalflora
done to confirm the diag- nosis of Whipple disease (E) Viridans streptococci and C albicans
caused by Tropheryma whipplei?
(A) Culture on agar media 4. A 70-year-old man with a history of diverticulosis
(B) Polymerase chain reaction amplification and of the sigmoid colon experiences a sudden onset of
sequencing severe left lower quadrant abdominal pain. Fever
of an appropriate segment of DNA develops. The severe pain gradually subsides and
(C) Cocultivation with E coli is replaced by a constant aching pain and marked
(D) Insituhybridization abdominal tenderness. A diagnosis of probable
(E) Direct fluorescent antibody test ruptured diverticulum is made, and the patient is
taken to the operating room. The diagnosis of
15. Which of the following best describes the ruptured diverticulum is confirmed, and an abscess
mechanism of action of diphtheria toxin? next to the sigmoid colon is found. The most likely
(A) Forms pores in red blood cells causing bacteria to be found in the abscess are
hemolysis
Jawetz Medical Microbiology 27th ed.
(A) Mixed normal gastrointestinal flora (B) (D) Urethra
Bacteroides fragilis alone (E) Vagina
(C) E coli alone
(D) Clostridium perfringens alone 11. Hundreds of phylotypes have been identified in
(E) Enterococcus species alone the human stomach; however, the only
microorganism that has been shown to persist is
5. Antimicrobial therapy can decrease the amount (A) Lactobacillus casei
of susceptible (B) Lactobacillus acidophilus
bowel flora and allow proliferation of relatively (C) E coli
resistant colonic bacteria. Which one of the (D) Helicobacter pylori
following species can proliferate and produce a (E) Bifidobacteria
toxin that causes diarrhea?
(A) Enterococcus species 12. Resident flora is commonly found in the
(B) S epidermidis (A) Liver
(C) Pseudomonas aeruginosa (B) Urethra
(D) Clostridium difficile (C) Kidneys
(E) B fragilis (D) Salivary glands
(E) Gall bladder
6. Which one of the following microorganisms can
be part of the normal vaginal flora and cause 13. Resident flora is absent from the
meningitis in newborns? (A) Pharynx
(A) C. albicans (B) Lungs
(B) Corynebacterium species (C) Small intestine
(C) S epidermidis (D) Synovial fluid
(D) Ureaplasmaurealyticum (E) Conjunctiva
(E) Group B streptococci
14. A 65-year-old woman was admitted with
7. Dental plaque and periodontal disease can be squamous cell carcinoma of the cervix. She
thought of as a continuum of what type of underwent extensive gynecologic surgery and was
physiological process? maintained postoperatively on broad-spectrum
(A) Biofilm formation intravenous antibiotics. The patient had a central
(B) Normal aging venous catheter placed on the day of surgery.
(C) Abnormal digestion Beginning 3 days postoperatively, the patient
(D) Exaggerated immune response became febrile. On day 8, cultures of blood and of
(E) Chewing gum the tip of the central line both grew gram-positive
organisms that were ovoid and reproduced by
8. Which one of the following microorganisms is budding. Which of the following microorganisms is
closely associated with dental caries? most likely responsible for the patient’s condition?
(A) C albicans (A) Saureus
(B) Streptococcus mutans(C) P melaninogenica(D) (B) S epidermidis
Neisseria subflava(E) S epidermidis (C) Enterococcus faecalis
(D) C. albicans
9. Anaerobic bacteria such as B fragilis occur in the (E) Saccharomyces cerevisiae
sigmoid colon in a concentration of about 1011/g of
stool. At what concentration do facultative 15. The most likely portal of entry for the organism
organisms such as E coli occur? in Question 14 is
(A) 1011/g (A) During gynecologic surgery
(B) 1010/g (B) Aspiration
(C) 109/g (C) During placement of the central venous
(D) 108/g catheter
(E) 107/g (D) During placement of IV line for administration of
antibiotics
10. S pneumoniae can be part of the normal flora of (E) Intubation while under anesthesia
5–40% of people. At what anatomic site can it be
found?
(A) Conjunctiva
(B) Nasopharynx
(C) Colon
Jawetz Medical Microbiology 27th ed.
Chapter 11: Spore-forming Gram-Positive
Bacilli: Bacillus and Clostridium Species 5. The reported incubation period for inhalational
anthrax can be up to
1. A housewife who lives on a small farm is brought (A) 2 days
to the emergency department complaining of (B) 10 days
double vision and difficulty talking. Within the past 2 (C) 3 weeks
hours, she noted a dry mouth and generalized (D) 6 weeks
weakness. Last night she served home-canned (E) 6 months
green beans as part of the meal. She tasted the
beans before they were boiled. None of the other 6. A food commonly associated with Bacillus cereus
family members are ill. On examination, there is food poisoning is
symmetrical descending paralysis of the cranial (A) Fried rice
nerves, upper extremities, and trunk. The correct (B) Baked potato
diagnosis is which one of the following? (C) Hot freshly steamed rice
(A) Tetanus (D) Green beans
(B) Strychnine poisoning (E) Honey
(C) Botulism
(D) Morphine overdose 7. Tetanus toxin (tetanospasmin) diffuses to
(E) Ricin intoxication terminals of inhibitory cells in the spinal cord and
brainstem and blocks which of the following?
2. Which one of the following is an important (A) Release of acetylcholine
virulence factor of (B) Cleavage of SNARE proteins
Bacillus anthracis? (C) Release of inhibitory glycine and γ-
(A) Protective antigen aminobutyric acid
(B) Lipopolysaccharide (D) Release of Protective Antigen
(C) Pili (E) Activation of acetylcholine esterase
(D) A toxin that inhibits peptide chain elongation
factor EF-2 8. A 45-year-old man who immigrated to the United
(E) Lecithinase States 5 years ago sustained a puncture injury to
the lower part of his right leg when his rotary lawn
3. A young man sustains major soft tissue injury mower threw a small stick into his leg. Six days
and open fractures of his right leg after a later, he noticed spasms in the muscles of his right
motorcycle accident. One day later, he has a leg; on day 7, the spasms increased. Today—day
temperature of 38°C, increased heart rate, 8—he had generalized muscle spasms, particularly
sweating, noticeable in the muscles of his jaw. He was unable
and restlessness. On examination, the leg is to open his jaw and came to the emergency
swollen and tense, with thin, dark serous fluid department (ED). In the ED, you see a man who is
draining from the wounds. The skin of the leg is alert and lying quietly in bed. A door slams down
cool, pale, white, and shining. Crepitus can be felt the hall, and suddenly he has general muscle
in the leg. His hematocrit is 20% (~50% of normal), spasm with arching of his back. The correct
and his circulating hemoglobin is normal. His serum diagnosis is which of the following?
shows free hemoglobin. Which of the following (A) Botulism
microorganisms is the most likely cause of this (B) Anthrax
infection? (C) Gas gangrene
(A) Clostridium tetani (D) Tetanus
(B) Staphylococcus aureus (E) Toxic shock syndrome
(C) Escherichia coli
(D) Bacillus anthracis 9. Which of the following statements about tetanus
(E) Clostridium perfringens and tetanus toxoid is correct?
(A) Tetanus toxin kills neurons.
4. For the patient described in Question 3, which of (B) Tetanus toxoid immunization has a 10% failure
the following is likely to be responsible for the rate.
hemolysis? (C) The mortality rate of generalized tetanus is less
(A) Elongation factor than 1%.
(B) Tetanospasmin (D) Double vision is commonly the first sign of
(C) Lecithinase tetanus.
(D) Streptolysin O (E) Tetanus toxin acts on inhibitor interneuron
(E) Toxin B synapses.
Jawetz Medical Microbiology 27th ed.
15. All of the following statements regarding
10. A 67-year-old man had surgery for a ruptured Clostridium perfringens are correct EXCEPT:
sigmoid colon diverticulum with an abscess. A (A) It produces an enterotoxin.
repair was done, and the abscess was drained. He (B) It produces a double zone of β-hemolysis when
was treated with intravenous gentamicin and grown on
ampicillin. Ten days later and 4 days after being blood agar.
discharged from the hospital, the patient developed (C) Some strains are aerotolerant.
malaise, fever, and cramping abdominal pain. He (D) It is the most common cause of antibiotic-
had multiple episodes of diarrhea. His stool was associated
positive for occult blood and the presence of diarrhea.
polymorphonuclear cells. On sigmoidoscopy, the (E) It can cause intravascular hemolysis.
mucosa was erythematous and appeared to be
inflamed, and there were many raised white to Chapter 12: Aerobic Non-Spore-forming Gram-
yellowish plaques 4–8 mm in diameter. Which of the Positive Bacilli: Corynebacterium, Listeria
following is the likely cause of the patient’s
problem? 1. Three months ago, a 53-year-old woman had
(A) Staphylococcus aureus enterotoxin surgery and chemotherapy for breast cancer. Four
(B) Bacillus cereus toxin weeks ago, she developed a cough occasionally
(C) Clostridium difficile toxins productive of purulent sputum. About 2 weeks I
(D) Clostridium perfringens toxin ago, she noted a slight but progressive weakness of
(E) Enterohemorrhagic Escherichia coli her left arm and leg. On chest examination, rales
were heard over the left upper back when the
11. Infant botulinum has been associated with all of patient breathed deeply. Neurologic examination
the following Clostridium species EXCEPT: confirmed weakness of the left arm and leg. Chest
(A) Clostridium baratii radiography showed a left upper lobe infiltrate.
(B) Clostridium septicum Contrast- enhanced computed tomography showed
(C) Clostridium butyricum two lesions in the right hemisphere. Gram stain of a
(D) Clostridium botulinum purulent sputum specimen showed branching gram-
positive rods that were partially acid fast. Which of
12. Which of the following food items is most the following organisms is the cause of this patient’s
frequently associated with infant botulism? current illness?
(A) Corn syrup (A) Actinomyces israelii
(B) Canned infant formula (B) Corynebacterium pseudodiphtheriticum
(C) Liquid multivitamins (C) Aspergillus fumigatus
(D) Honey (D) Nocardia farcinica
(E) Jarred baby food (E) Erysipelothrix rhusiopathiae

13. All of the following are properties characteristic 2. The drug of choice to treat this patient’s infection
of Bacillus anthracis EXCEPT: (Question1) is (A) Penicillin G
(A) Motility on wet mount examination (B) Trimethoprim–sulfamethoxazole
(B) Medusa head colonies (C) Gentamicin
(C) Poly-d-glutamic acid capsule (D) Amphotericin B
(D) In vitro susceptibility to penicillin (E) A third-generation cephalosporin
(E) Absence of hemolysis on 5% sheep blood agar
3. It is particularly difficult to differentiate
14. Which of the following statements regarding Erysipelothrix rhusiopathiae from
vaccination for Bacillus anthracis is correct? (A) Corynebacterium diphtheriae
(A) It is routinely available for all citizens of the (B) Bacillus cereus
United States. (C) Actinomyces israelii
(B) Recombinant vaccine trials have shown good (D) Nocardia asteroides
safety and (E) Lactobacillus species
efficacy.
(C) The current vaccine is well tolerated. 4. Movement of Listeria monocytogenes inside of
(D) A single dose is adequate after exposure to host cells is caused by
spores. (A) Inducing host cell actin polymerization
(E) Vaccination of animals is not useful. (B) The formation of pili (fimbriae) on the listeriae
surface
(C) Pseudopod formation
Jawetz Medical Microbiology 27th ed.
(D) The motion of listeriae flagella of the finger. He sought medical therapy. The
(E) Tumbling motility violaceous nodule was aspirated, and after 48
5. An 8-year-old boy, who recently arrived in the hours of incubation, colonies of a gram-positive
United States, develops a severe sore throat. On bacillus that caused greenish discoloration of the
examination, a grayish exudate (pseudomembrane) agar and formed long filaments in the broth culture
is seen over the tonsils and pharynx. The were noted. The most likely cause of this infection is
differential diagnosis of severe pharyngitis such as (A) Lactobacillus acidophilus
this includes group A streptococcal infection, (B) Erysipelothrix rhusiopathiae
Epstein-Barr virus (EBV) infection, Neisseria (C) Listeria monocytogenes
gonorrhoeae pharyngitis, and diphtheria. The cause (D) Rhodococcus equi
of the boy’s pharyngitis is most likely: (E) Nocardia brasiliensis
(A) A gram-negative bacillus
(B) A single-stranded positive-sense RNA virus 11. A biochemical reaction that is useful in the
(C) A catalase-positive, gram-positive coccus that identification of the causative agent of the infection
grows in clusters in Question 10 is
(D) A club-shaped gram-positive bacillus (A) Catalase positivity
(E) A double-stranded RNA virus (B) Acid fastness using modified Kinyoun stain
(C) Esculin hydrolysis
6. The primary mechanism in the pathogenesis of (D) Tumbling motility
the boy’s dis- ease (Question 5) is (E) Production of H2S
(A) A net increase in intracellular cyclic adenosine
monophosphate 12. Listeria monocytogenes is frequently a
(B) Action of pyrogenic exotoxin (a superantigen) foodborne pathogen because
(C) Inactivation of acetylcholine esterase (A) It can survive at 4°C.
(D) Action of enterotoxin A (B) It survives under conditions of low pH.
(E) Inactivation of elongation factor 2 (C) It survives in the presence of high salt
concentrations.
7.Corynebacterium jeikeium is (D) All of the above are correct.
(A) Catalase negative
(B) Gram negative 13. After recovery on laboratory media, the aerobic
(C) Often multidrug resistant Actinomycetes are best identified by
(D) Motile (A) An automated system used in the laboratory
(E) Common but clinically unimportant (B) Classical biochemicals
(C) Antigen detection tests such as an ELISA
8. Which of the following aerobic gram-positive (D) Molecular methods such as 16S rRNA gene
bacilli is modified acid-fast positive? sequencing
(A) Nocardia brasiliensis
(B) Lactobacillus acidophilus 14. Which of the following statements regarding
(C) Erysipelothrix rhusiopathiae Rhodococcus equiis correct?
(D) Listeria monocytogenes (A) It is transmitted person to person.
(B) It causes tuberculosis in cattle.
9. Skin diphtheria as occurs in children in tropical (C) It is a rare cause of pulmonary infection in
areas typically humans.
(A) Does not occur in children who have been (D) It produces a black pigment on sheep blood
immunized with diphtheria toxoid agar.
(B) Is clinically distinct from skin infections
(pyoderma, impetigo) caused by Streptococcus 15. A hospitalized patient who had an indwelling
pyogenes and Staphylococcus aureus urinary catheter develops fever, chills, suprapubic
(C) Is also common in northern latitudes pain, and difficulty voiding 48 hours after the
(D) Results in protective antitoxin levels in most catheter is removed. His bladder appears
children by obstructed, and he has white blood cells and
the time they are 6–8 years old bacteria on a urinalysis. Cystoscopy reveals a large
(E) Yields toxin-mediated cardiomyopathy bladder stone, and the urine culture grows greater
than 10,000 CFU/mL of a short, irregular gram-
10. A 45-year-old fisherman imbedded a fishhook positive rod. The most likely organism causing this
into his right forefinger. He removed it and did not infection is
seek immediate medical therapy. Five days later, he (A) Corynebacterium urealyticum
noted fever, severe pain, and nodular- type swelling (B) Nocardia brasiliensis
Jawetz Medical Microbiology 27th ed.
(C) Actinomadura (C) Erythromycin, clarithromycin, and azithromycin
(D) Erysipelothrix rhusiopathie (D) Vancomycin
(E) Lactobacillus acidophilus (E) Cefazolin and ceftriaxone

4. Seven days ago, a 27-year-old medical student


Chapter 13: The Staphylococci returned from Central America, where she had
spent the summer working in a clinic for indigenous
1. A 54-year-old woman develops a right shoulder people. Four days ago, she developed an
abscess with a strain of Staphylococcus aureus that erythematous sunburn-like rash. She also has had
is resistant to nafcillin. She was treated with a 2- headache, muscle aches, and abdominal cramps
week course of intravenous vancomycin with diarrhea. Her blood pressure is 70/40 mm Hg.
and improved. Three weeks later (week 5), the Pelvic examination shows she is having her
infection recurred, and she was given 2 more menstrual period with a tampon in place; otherwise,
weeks of intravenous vancomycin and again the pelvic examination is normal. Her kidney
improved. Four weeks later (week 11), the infection function test (serum urea nitrogen and creatinine)
recurred and the patient was again started on results are abnormal, indicating mild renal failure. A
intravenous vancomycin. The MICs for vancomycin blood smear for malaria is negative. Her illness is
for the S aureus isolates were as follows: initial likely to be caused by which of the following?
isolate (day 1), 1 μg/mL; week 5, 2 μg/mL; and (A) A toxin that results in greatly increased levels of
week 11, 8 μg/mL. The patient failed to improve intracellular cyclic adenosine monophosphate
with the third course of vancomycin, and alternative (cAMP)
therapy was used. The mechanism that best (B) A toxin that degrades sphingomyelin
explains the relative resistance of the patient’s (C) A toxin that binds to the class II major
strain of S aureus to vancomycin is histocompatibility
(A) Acquisition of the vanA gene from another complex (MHC) of an antigen-presenting cell and
microorganism the Vβ region of a T cell
(B) Active transport of vancomycin out of the (D) A two-component toxin that forms pores in
Staphylococcus white blood
aureus cell cells and increases cation permeability
(C) Action of β-lactamase (E) A toxin that blocks elongation factor 2 (EF2)
(D) Increased cell wall synthesis and alterations in
the cell wall 5. Over a period of 3 weeks, a total of five
structure newborns in the hospital nursery developed
(E) Phosphorylation and resultant inactivation of Staphylococcus aureus infections withS aureus
the vancomycin bacteremia. The isolates all had the same colony
morphology and hemolytic properties and identical
2. An 11-year-old boy develops a mild fever and antimicrobial susceptibility patterns, suggesting that
pain in his upper arm. A radiograph of his arm they were the same. (Later molecular methods
shows a lytic lesion (dissolution) in the upper part of showed the isolates were identical.) Which of the
the humerus with periosteal elevation over the following should be done now?
lesion. The patient is taken to surgery, where the (A) Prophylactic treatment of all newborns with
lesion is debrided (dead bone and pus removed). intravenous vancomycin
Culture from the lesion yields gram-positive cocci. A (B) Protective isolation of all newborns
test shows that the organism is a Staphylococcus (C) Closing the nursery and referring pregnant
and not a Streptococcus. Based on this information, women to
you know the organism is another hospital
(A) Susceptible to nafcillin (D) Hiring all new staff for the hospital nursery
(B) β-Lactamase positive (E) Culture using mannitol salt agar of the anterior
(C) A producer of protein A nares of the physicians, nurses, and others who
(D) Encapsulated cared for the infected babies
(E) Catalase positive
6. The exfoliative toxins, TSST-1, and the
3. A 36-year-old male patient has an abscess with a enterotoxins are all superantigens. The genes for
strain of Staphylococcus aureus that is β-lactamase these toxins are
positive. This indicates that the organism is (A) Present in all strains of Staphylococcus aureus
resistant to which of the following antibiotics? (B) Widely distributed on the staphylococcal
(A) Penicillin G, ampicillin, and piperacillin chromosome
(B) Trimethoprim–sulfamethoxazole
Jawetz Medical Microbiology 27th ed.
(C) On both the staphylococcal chromosome following microorganisms is a likely cause of the
(TSST-1 and exfoliative toxins) and on plasmids lesions?
(enterotoxins) (A) Escherichia coli
(D) On the staphylococcal chromosome in a (B) Chlamydia trachomatis
pathogenicity island (C) Staphylococcus aureus
(E) On plasmids (D) Streptococcus pneumoniae
(E) Bacillus anthracis
7. A 16-year-old bone marrow transplant patient
has a central venous line that has been in place for 11. Which of the following statements regarding the
2 weeks. He also has a urinary tract catheter, which role of protein A in the pathogenesis of infections
has been in place for 2 weeks as well. He develops caused by Staphylococcus aureus is correct?
fever while his white blood cell count is very low (A) It is responsible for the rash in toxic shock
and before the transplant has engrafted. Three syndrome.
blood cultures are done, and all grow (B) It converts hydrogen peroxide into water and
Staphylococcus epidermidis. Which one of the oxygen.
following statements is correct? (C) It is a potent enterotoxin.
(A) The Staphylococcus epidermidis organisms are (D) It is directly responsible for lysis of neutrophils.
likely to be susceptible to penicillin G. (E) It is a bacterial surface protein that binds to the
(B) The Staphylococcus epidermidis organisms are Fc portion of IgG1.
likely to be from the surface of the urinary tract
catheter. 12. Which of the following staphylococcal
(C) The Staphylococcus epidermidis organisms are organisms produces coagulase and has been
likely to be resistant to vancomycin. implicated in infections following a dog bite?
(D) The Staphylococcus epidermidis organisms are (A) Staphylococcus intermedius
likely to be from a skin source. (B) Staphylococcus epidermidis
(E) The Staphylococcus epidermidis organisms are (C) Staphylococcus saprophyticus
likely to be in a biofilm on the central venous (D) Staphylococcus hominis
catheter surface. (E) Staphylococcus hemolyticus

8. A 65-year-old man develops an abscess on the 13. All of the following statements regarding
back of his neck. Culture yields Staphylococcus Panton–Valentine
aureus. The isolate is tested and found to be leukocidin are correct except
positive for the mecA gene, which means that (A) It is a two-component toxin.
(A) The isolate is susceptible to vancomycin. (B) It is commonly produced by community-
(B) The isolate is resistant to vancomycin. associated MRSA strains.
(C) The isolate is susceptible to nafcillin. (C) It is an important virulence factor.
(D) The isolate is resistant to nafcillin (D) It is identical to one of the staphylococcal
(E) The isolate is susceptible to clindamycin. enterotoxins.
(F) The isolate is resistant to clindamycin. (E) It forms pores in the membranes of white blood
cells.
9. Antimicrobial resistance has become a significant
problem. 14. Which of the following statements best
Which one of the following is of major concern describes the function of the accessory gene
worldwide? regulator in Staphylococcus aureus?
(A) Nafcillin resistance in Staphylococcus aureus (A) It regulates production of β-hemolysins.
(B) Penicillin resistance in Streptococcus (B) It is influenced by environmental oxygen.
pneumoniae (C) It controls the preferential expression of surface
(C) Penicillin resistance in Neisseria gonorrhoeae adhesins.
(D) Vancomycin resistance in Staphylococcus (D) It is important in the control of autolysis.
aureus
(E) Tobramycin resistance in Escherichia coli 15. All of the following are important infection
control strategies in containing spread of MRSA in
10. A group of six children younger than 8 years of hospitals except
age live in a semitropical country. Each of the (A) Aggressive hand hygiene
children has several crusted weeping skin lesions of (B) Routine surveillance for nasal colonization
impetigo (pyoderma). The lesions are among high-
predominantly on the arms and faces. Which of the risk individuals
Jawetz Medical Microbiology 27th ed.
(C) Contact isolation for patients who are colonized 3. This infection (question 1) might have been
or infected prevented by
with MRSA (A) Prophylactic intramuscular benzathine penicillin
(D) Routine antimicrobial prophylaxis for all every 3 weeks
patients hospital- (B) A 23-valent capsular polysaccharide vaccine
ized for more than 48 hours (C) A vaccine against serogroups A, C, Y, and
(E) Aseptic management of skin lesions W135 capsular polysaccharide
(D) A vaccine of polyribosylribitol capsular
polysaccharide covalently linked to a protein
Chapter 14: The Streptococci, Enterococci, and (E) Oral penicillin twice daily
Related Genera
4. The pathogenesis of the organism causing the
1. A 48-year-old alcoholic man is admitted to a infection (question 1) includes which of the
hospital because of stupor. He is unkempt and following?
homeless and lives in an encampment with other (A) Invasion of cells lining the alveoli and entry into
homeless people, who called the authorities the pulmonary venule circulation
when he could not be easily aroused. His (B) Resistance to phagocytosis mediated by M
temperature is 38.5°C, and his blood pressure proteins
125/80 mm Hg. He moans when attempts are made (C) Migration to mediastinal lymph nodes where
to arouse him. He has positive Kernig and hemorrhage occurs
Brudzinski signs, suggesting meningeal irritation. (D) Lysis of the phagocytic vacuole and release
Physical examination and chest radiography show into the circulation
evidence of left lower lobe lung consolidation. An (E) Inhibition of phagocytosis by a polysaccharide
endotracheal aspirate yields rust-colored sputum. capsule
Examination of a Gram-stained sputum smear
shows numerous polymorphonuclear cells and 5. A 13-valent capsular polysaccharide protein
numerous gram-positive lancet-shaped diplococci. conjugate vaccine for the pathogen in question 1 is
On lumbar puncture, the cerebrospinal fluid is recommended
cloudy and has a white blood cell count of 570/μL (A) For children up to age 18 years and for
with 95% polymorphonuclear cells; Gram stain selected adults
shows numerous gram-positive diplococci. Based (B) Only on exposure to a patient with disease
on this information, the likely diagnosis is caused by the organism
(A) Pneumonia and meningitis caused by (C) For all children ages 2–23 months plus
Staphylococcus aureus selected older children and adults with
(B) Pneumonia and meningitis caused by immunocompromising conditions
Streptococcus pyogenes (D) For children ages 24–72 months
(C) Pneumonia and meningitis caused by (E) For all age groups older than age 2 months
Streptococcus
pneumoniae 6. An 8-year-old boy develops a severe sore throat.
(D) Pneumonia and meningitis caused by On examination, a grayish-white exudate is seen on
Enterococcus faecalis the tonsils and pharynx. The differential diagnosis
(E) Pneumonia and meningitis caused by Neisseria includes group A streptococcal infection, Epstein-
meningitidis Barr virus infection, severe adenovirus infection,
and diphtheria. (Neisseria gonorrhoeae pharyngitis
2. The patient in question 1 is started on antibiotic would also be included, but the patient has not
therapy to cover many possible microorganisms. been sexually abused.) The cause of the boy’s
Subsequently, culture of sputum and cerebrospinal pharyngitis is most likely
fluid yields gram-positive diplococci with a minimum (A) A catalase-negative gram-positive coccus that
inhibitory concentration to penicillin G of greater grows in chains
than 2 μg/mL. The drug of choice for this patient (B) A single-stranded positive-sense RNA virus
until further susceptibility testing can be done is (C) A catalase-positive gram-positive coccus that
(A) Penicillin G grows in
(B) Nafcillin clusters
(C) Trimethoprim–sulfamethoxazole (D) A catalase-negative gram-positive bacillus
(D) Gentamicin (E) A double-stranded RNA virus
(E) Vancomycin
7. A primary mechanism responsible for the
pathogenesis of the boy’s disease (question 6) is
Jawetz Medical Microbiology 27th ed.
(A) A net increase in intracellular cyclic adenosine (A) It is responsible for the mucoid appearance of
monophosphate the colonies in vitro.
(B) Action of M protein (B) It is antiphagocytic.
(C) Action of IgA1 protease (C) It binds to CD44 on human epithelial cells.
(D) Action of enterotoxin A (D) It is an important virulence factor.
(E) Inactivation of elongation factor 2 (E) A vaccine against the capsule is currently
available.
8. A 40-year-old woman develops severe headache
and fever. Her neurologic examination findings are 12. Enterococci can be distinguished from
normal. A brain scan shows a ring-enhancing lesion nonenterococcal group D streptococci on the basis
of the left hemisphere. During surgery, a brain of which of the following characteristics?
abscess is found. Culture of the abscess fluid grows (A) γ-Hemolysis
an anaerobic gram-negative bacillus (B) Esculin hydrolysis
(Fusobacterium nucleatum) and a catalase- (C) Growth in 6.5% NaCl
negative gram-positive coccus that on Gram stain is (D) Growth in the presence of bile
in pairs and chains. The organism is β-hemolytic (E) Gram stain morphology
and forms very small colonies (<0.5 mm in
diameter). One person thought it smelled like 13. Which of the following statements regarding the
butterscotch. It agglutinates with group F antisera. Streptococcus bovis group is correct?
The organism most likely is (A) They possess Lancefield group D antigen.
(A) Streptococcus pyogenes(group A) (B) Some strains are vancomycin resistant.
(B) Enterococcus faecalis (group D) (C) Infections caused by these organisms are
(C) Streptococcus agalactiae (group B) benign.
(D) Streptococcus anginosus group (D) All subspecies are PYR positive.
(E) Staphylococcus aureus (E) All subspecies are β-hemolytic.

9. Important methods for classifying and speciating 14. Which of the following genera requires pyridoxal
streptococci are for growth?
(A) Agglutination using antisera against the cell (A) Aerococcus
wall group- specific substance (B) Granulicatella
(B) Biochemical testing (C) Enterococcus
(C) Hemolytic properties (α-, β-, nonhemolytic) (D) Leuconostoc
(D) Capsular swelling (quellung) reaction (E) Pediococcus
(E) All of the above
15. Which of the following genera is typically
10. An 8-year-old girl develops Sydenham’s chorea resistant to vancomycin?
(“St. Vitus dance”) with rapid uncoordinated facial (A) Aerococcus
tics and involuntary purposeless movements of her (B) Gemella
extremities, strongly suggestive of acute rheumatic (C) Pediococcus
fever. She has no other major manifestations of (D) Streptococcus
rheumatic fever (carditis, arthritis, subcutaneous (E) Abiotrophia
nodules, skin rash). The patient’s throat culture is
negative for Streptococcus pyogenes (group A
streptococci). However, she, her brother, and her Chapter 15: Enteric Gram-Negative Rods
mother all had sore throats 2 months ago. A test (Enterobacteriaceae)
that if positive would indicate recent S pyogenes
infections is 1. A 20-year-old college student goes to the student
(A) Antistreptolysin S antibody titer health center because of dysuria, frequency, and
(B) Polymerase chain reaction for antibodies urgency on urination for 24 hours. She has recently
against M protein become sexually active. On urinalysis, many
(C) ASO antibody titer polymorphonuclear cells are seen. The most likely
(D) Esculin hydrolysis organism responsible for these symptoms and signs
(E) Antihyaluronic acid antibody titer is
(A) Staphylococcus aureus
11. All of the following statements regarding the (B) Streptococcus agalactiae
hyaluronic acid capsule of S pyogenes are correct (C) Gardnerella vaginalis
except (D) Lactobacillus species
(E) Escherichia coli
Jawetz Medical Microbiology 27th ed.
(E) Production of gas in the medium
2. A 27-year-old woman is admitted to the hospital
because of fever, with increasing anorexia, 6. An uncommon serotype of Salmonella enterica
headache, weakness, and altered mental status of subspecies enterica was found by laboratories in
2 days’ duration. She works for an airline as a cabin the health departments of adjacent states. The
attendant, flying between the Indian subcontinent isolates were all from a small geographic area on
and other places in Southeast Asia and the West either side of the border between the states,
Coast of the United States. Ten days before suggesting a common source for the isolates. (All of
admission, she had a diarrheal illness that lasted for the isolates were from otherwise healthy young
about 36 hours. She has been constipated for the adults who smoked marijuana; the same
past 3 days. Her temperature is 39°C, heart rate is Salmonella was isolated from a specimen of the
68 beats/ min, blood pressure is 120/80 mm Hg, marijuana.) By what method did the public health
and respirations are 18 breaths/min. She knows laboratories determine that these isolates were the
who she is and where she is but does not know the same?
date. She is picking at the bedclothes. Rose spots (A) Capsular (K antigen) typing
are seen on her trunk. The remainder of the (B) O antigen and H antigen typing
physical examination is normal. Blood cultures are (C) DNA sequencing
done, and an intravenous line is placed. The most (D) Sugar fermentation pattern determination
likely cause of her illness is (E) Decarboxylase reaction pattern determination
(A) Enterotoxigenic Escherichia coli(ETEC)
(B) Shigella sonnei 7. A 43-year-old man with diabetes has a 4-cm
(C) Salmonella enterica subspecies enterica nonhealing foot ulcer. Culture of the ulcer yields
serotype Typhimurium (Salmonella Typhimurium) Staphylococcus aureus, Bacteroides fragilis, and a
(D) Salmonella enterica subspecies enterica gram-negative bacillus that swarms across the
serotype Typhi (Salmonella Typhi) blood agar plate covering the entire surface of the
(E) Enteroinvasive Escherichia coli (EIEC) agar after 36 hours. The gram-negative bacillus is a
member of the genus
3. Blood cultures from the patient in question 2
grow a non– lactose-fermenting gram-negative (A) Escherichia
bacillus. Which of the following is likely to be a (B) Enterobacter
constituent of this organism? (C) Serratia
(A) O antigen 157, H antigen 7 (O157:H7) (D) Salmonella
(B) Vi antigen (capsule; virulence antigen) (E) Proteus
(C) O antigen 139 (O139)
(D) Urease 8. A 4-year-old boy from Kansas City who recently
(E) K1 (capsular type 1) started attending preschool and after-school
daycare is brought to his pediatrician for a diarrheal
4. A 37-year-old woman with a history of urinary illness characterized by fever to 38.2°C, severe
tract infections comes to the emergency department lower abdominal pain, and initially watery diarrhea.
with burning on urination along with frequency and His mother became concerned because the stools
urgency. She says her urine smells like ammonia. are now blood tinged 24 hours into the illness, and
The cause of her urinary tract infection is likely to be the child appears quite ill. The mother reports that
(A) Enterobacter aerogenes two other children who attend the same after-school
(B) Proteus mirabilis daycare have recently had diarrheal disease, one of
(C) Citrobacter freundii whom likewise had bloody stools. Which of the
(D) Escherichia coli following is the most likely pathogen causing the
(E) Serratia marcescens illness in these children?

5. An 18-year-old student has abdominal cramps (A) An enterotoxigenic strain of Escherichia coli
and diarrhea. A selective agar plate is inoculated (B) Salmonella enterica subspecies enterica
and grows suspicious gram- negative rods. Triple serotype Typhi
sugar iron agar is used to identify the isolates as (Salmonella Typhi)
salmonellae or shigellae. A result suggesting one of (C) Shigella sonnei
these two pathogens would be (D) Edwardsiella tarda
(A) Production of urease (E) Klebsiella oxytoca
(B) Motility in the medium
(C) Inability to ferment lactose and sucrose 9. A 5-year-old girl attended a birthday party at a
(D) Fermentation of glucose local fast food restaurant. About 48 hours later, she
Jawetz Medical Microbiology 27th ed.
developed cramping abdominal pain and a low-
grade fever and had five episodes of loose, bloody 13. An HIV-positive man recently traveled to the
stools. She is taken to a local emergency Caribbean for a 2-week vacation. He developed
department the next evening because the diarrhea acute watery diarrhea and abdominal pain without
has continued, and she now appears pale and fever during the second week of his vacation. Three
lethargic. On presentation, she has a temperature weeks later, he is seen in clinic for persistent
of 38°C, and she is hypotensive and tachycardic. symptoms, and he is concerned because he is
The abdominal examination reveals tenderness in beginning to lose weight. Given this history, you
the lower quadrants. Laboratory work is remarkable suspect:
for a serum creatinine of 2.0 mg/dL, a serum (A) Enteroinvasive Escherichia coli
hemoglobin of 8.0 mg/dL, thrombocytopenia, and (B) Salmonella typhi
evidence of hemolysis. What is the most likely (C) Enteropathogenic Escherichia coli
pathogen causing this child’s illness? (D) Shigella flexneri
(A) Escherichia coli O157:H7 (E) Enteroaggregative Escherichia coli
(B) Salmonella enterica subspecies enterica
serotype Typhimurium 14. Heat-labile toxin of ETEC acts by which of the
(C) Enteropathogenic Escherichia coli following
(D) Edwardsiella tarda mechanisms?
(E) Plesiomonas shigelloides (A) Attachment and effacement
(B) Activation of adenylyl cyclase
10. A 55-year-old homeless man with alcoholism (C) Aggregative adherence
presents with severe multilobar pneumonia. He (D) Ribosomal dysfunction
requires intubation and mechanical ventilation. A (E) None of the above
Gram stain of his sputum reveals numerous
polymorphonuclear leukocytes and gram-negative 15. A young woman presents with recurrent urinary
rods that appear to have a capsule. The organism tract infections caused by the same Proteus
is a lactose fermenter on MacConkey agar and is mirabilis strain. What is the major concern?
very mucoid. It is nonmotile and lysine (A) She does not take her medication.
decarboxylase positive. What is the most likely (B) She is pregnant because pregnant patients are
organism causing this man’s illness? more susceptible to UTIs.
(A) Serratia marcescens (C) She has a bladder or kidney stone.
(B) Enterobacter aerogenes (D) Her partner is infected.
(C) Proteus mirabilis (E) She has occult diabetes and should have a
(D) Klebsiella pneumoniae glucose tolerance test
(E) Morganella morganii

11. Which of the following statements regarding O Chapter 16: Pseudomonads and Acinetobacter
antigens is correct?
(A) All Enterobacteriaceae possess identical O 1. A sputum culture of a patient with cystic fibrosis
antigens. grows Pseudomonas aeruginosa that forms very
(B) They are found in the polysaccharide capsules mucoid colonies. The implication of this observation
of enteric is which one of the following?
bacteria. (A) The Pseudomonas aeruginosa is highly
(C) They are covalently linked to a polysaccharide susceptible to the
core. aminoglycoside antimicrobial tobramycin.
(D) They do not stimulate an immune response in (B) The Pseudomonas aeruginosa is infected with a
the host. pyocin (a bacteriocin).
(E) They are not important in the pathogenesis of (C) The colonies are mucoid because they have
infection caused by enteric bacteria. polysaccharide capsule of hyaluronic acid.
(D) The exotoxin A gene has been disabled and the
12. Which of the following test methods is the least Pseudomonas aeruginosa is no longer able to block
sensitive procedure for diagnosis of colitis caused host cell protein synthesis.
by Shiga toxin–producing Escherichia coli? (E) The Pseudomonas aeruginosa has formed a
(A) Culture on sorbitol MacConkey agar biofilm in the patient’s airway.
(B) Toxin testing using an enzyme immunoassay
(C) Cell culture cytotoxin assay using Vero cells 2. An environmental Gram-negative bacillus that is
(D) Polymerase chain reaction for detection of the resistant to cephalosporins, aminoglycosides, and
genes that encode Shiga toxin quinolones has become a very important
Jawetz Medical Microbiology 27th ed.
nosocomial pathogen largely because it is selected clear agar medium, they produce a green color in
by use of those antibiotics. This Gram-negative the agar. The agar where the green color is located
bacillus can take 2–3 days to grow and must be fluoresces when exposed to ultraviolet light. The
differentiated from Burkholderia cepacia. It is organism causing the patient’s infection is
(A) Pseudomonas aeruginosa (A) Pseudomonas aeruginosa
(B) Acinetobacter baumannii (B) Klebsiella pneumoniae
(C) Alcaligenes xylosoxidans (C) Escherichia coli
(D) Klebsiella pneumoniae (D) Burkholderia cepacia
(E) Stenotrophomonas maltophilia (E) Burkholderia pseudomallei

3. A 17-year-old girl with cystic fibrosis has a slight 7. The mechanism of action of exotoxin A of
increase in her frequent cough and production of Pseudomonas aeruginosa is
mucoid sputum. A sputum specimen is obtained (A) To activate acetylcholine esterase
and plated on routine culture media. The (B) To block elongation factor 2
predominant growths are Gram-negative bacilli that (C) To form pores in white blood cells and increase
form very mucoid colonies after 48 hours of cation permeability
incubation. These bacilli are oxidase positive, grow (D) To increase intracellular cyclic adenosine
at 42°C, and have a grape-like odor. These Gram- monophosphate
negative bacilli are which of the following? (E) To split lecithin into phosphorylcholine and
(A) Klebsiella pneumoniae diacylglycerol
(B) Pseudomonas aeruginosa
(C) Staphylococcus aureus 8. Patients deficient in these cells are at high risk
(D) Streptococcus pneumoniae for developing serious systemic infections with
(E) Burkholderia cepacia Pseudomonas aeruginosa:
(A) Eosinophils
4. The sputum from a 26-year-old patient with cystic (B) Neutrophils
fibrosis is plated on a colistin-containing agar. After (C) Macrophages
72 hours of incubation, the colistin-containing agar (D) Natural killer cells
grows Gram-negative bacilli that are oxidase (E) CD4+ T cells
positive but are otherwise difficult to identify. This
microorganism is of major concern. It is sent to a 9. A marine wounded in Afghanistan returns to her
reference laboratory so that molecular methods can home a paraplegic. Her past medical history
be used to identify or rule out which of the included surgery to amputate both her legs below
following? the knee and the placement of a suprapu- bic tube
(A) Pseudomonas aeruginosa to repair damage to her bladder. She is now at the
(B) Burkholderia cepacia VA outpatient clinic with a recurrent urinary tract
(C) Haemophilus influenzae infection that has not responded to conventional
(D) Pseudomonas putida antibiotic regimens for community-acquired cystitis.
(E) Burkholderia pseudomallei Her urine is positive for small, plump Gram-negative
coccobacilli. When cultured, this organism does not
5. Acinetobacter species: ferment carbohydrates, does not hydrolyze urea,
(A) Are only found in a hospital environment. does not reduce nitrates, and does not make
(B) May appear as Gram-positive rods. hydrogen sulfide. The organism most likely causing
(C) Can mimic the morphology of Hemophilus this marine’s infection is:
species in Gram stains of endocervical secretions. (A) Klebsiella oxytoca
(D) Can be a significant cause of ventilator- (B) Escherichia coli
associated pneumonia in intensive care unit (C) Staphylococcus saprophyticus
patients. (D) Proteus mirabilis
(E) Are susceptible to most antibiotics. (E) Acinetobacter baumanii

6. A 37-year-old firefighter sustains smoke 10. A 70-year-old neutropenic patient was


inhalation and is hospitalized for ventilatory support. diagnosed with ecthyma gangrenosum 3 days after
He has a severe cough and begins to expectorate he developed a fever of 39°C. Blood cultures drawn
purulent sputum. Gram stain of his sputum the day his fever started grew out overnight a
specimen shows numerous polymorphonuclear strictly aerobic, Gram-negative rod that was lactose
cells and numerous Gram-negative rods. Sputum negative and oxidase positive. Which of the
culture grows numerous Gram-negative rods that following antibiotic regimens would be most
are oxidase positive. They grow well at 42°C. On appropriate for treating this patient?
Jawetz Medical Microbiology 27th ed.
(A) Tobramycin+piperacillin/tazobactam (E) A medium containing vancomycin, polymyxin B,
(B) Vancomycin + metronidazole and trimethoprim incubated at 42°C in 5% oxygen
(C) Cefazolin and 10% CO2
(D) Tigecycline
(E) Oxacillin 4. Bacteremia associated with a gastrointestinal
infection is most likely to occur with which of the
following?
Chapter 17: Vibrio, Campylobacter,and (A) Salmonella typhi
Helicobacter (B) Vibrio cholerae
(C) Shigella boydii
1. Long-term carriage and shedding is most likely to (D) Vibrio parahaemolyticus
occur after gastrointestinal infection with which of (E) Campylobacter jejuni
the following species?
(A) Escherichia coli O157:H7 5. During the El Niño years in the mid- to late
(B) Shigella dysenteriae 1990s, the waters of Puget Sound between
(C) Vibrio cholerae Washington State and British Columbia warmed
(D) Campylobacter jejuni considerably. During this time, many people who
(E) Salmonella typhi ate clams and oysters from these waters became ill
with a disease characterized by explosive diarrhea
2. A 63-year-old man visited his favorite oyster and moderately severe abdominal cramps. The
restaurant in a small town on the eastern shore of diarrhea was usually watery, but in some patients, it
the Gulf Coast of Texas. He ate two dozen oysters. was bloody. The diarrhea usually had an onset
Two days later, he was admitted to the hospital within 24 hours after eating the shellfish. Stool
because of an abrupt onset of chills, fever, and cultures typically yielded a pathogenic gram-
light-headedness when he stood up. (In the negative bacillus. The microorganism of concern in
emergency department [ED], his blood pressure this setting is
was 60/40 mm Hg.) While in the ED, he developed (A) Enterotoxigenic Escherichia coli
erythematous skin lesions. These rapidly evolved (B) Vibrio cholerae
into hemorrhagic bullae, which then formed ulcers. (C) Enterohemorrhagic Escherichia coli
The man drank a six-pack of beer and one half- (D) Vibrio parahaemolyticus
bottle of whisky each day. A microorganism of (E) Shigella dysenteriae
major concern for this patient is
(A) Vibrio vulnificus 6. A patient presents to the emergency department
(B) Escherichia coli with nonbloody diarrhea for 12 hours. The patient
(C) Salmonella typhi lives in Washington, DC, and has not recently
(D) Clostridium perfringens traveled out of the area. Which one of the following
(E) Streptococcus pyogenes (group A streptococci) is unlikely to be the cause of your patient’s
diarrhea?
3. A family of four persons ate a meal that included (A) Salmonella typhimurium
undercooked chicken. Within 3 days, three (B) Campylobacter jejuni
members developed an illness characterized by (C) Shigella sonnei
fever, headache, myalgia, and malaise. Two of the (D) Vibrio cholerae
patients had concomitant diarrhea and abdominal (E) Escherichia coli
pain. The third person developed diarrhea after the
systemic symptoms had cleared. Stool cultures 7. An 18-year-old woman in rural Bangladesh
grew Campylobacter jejuni. Which of the following develops profuse (8 L/d) diarrhea. She has no
culture conditions was most likely used to isolate C symptoms other than the diarrhea and the
jejuni? manifestations of the fluid and electrolyte loss
(A) Thiosulfate-citrate-bile-sucrose medium caused by the diarrhea. The most likely cause of
incubated at 37°C in 5% oxygen and 10% CO2 her diarrhea is
(B) Salmonella-Shigella selective medium (A) Campylobacter jejuni
incubated at 37°C in ambient air (B) Enterotoxigenic Escherichia coli
(C) MacConkey agar and Hektoen enteric agar (C) Salmonella typhimurium
incubated at 42°C in 5% oxygen and 10% CO2 (D) Vibrio cholerae
(D) 5% sheep blood agar incubated at 37°C in (E) Shigella dysenteriae
ambient air
8. Age and geography are major factors in the
prevalence of colonization by Helicobacter pylori. In
Jawetz Medical Microbiology 27th ed.
developing countries, the prevalence of colonization 12. A 45-year-old man develops a gastric ulcer that
may be greater than 80% in adults. In the United can be visualized on a contrast medium–enhanced
States, the prevalence of colonization with this radiograph of his stomach. A biopsy specimen is
microorganism in adults older than age 60 years is taken from the gastric mucosa at the site of the
(A) 1–2% ulcer. A presumptive diagnosis can be reached
(B) 5–10% most rapidly by inoculating part of the specimen on
(C) 15–20% which of the following?
(D) 40–60% (A) A medium used to detect urease incubated at
(E) 80–95% 37°C
(B) A medium containing vancomycin, polymyxin B,
9. A 59-year-old man comes to the emergency and trimethoprim incubated at 42°C
department in the afternoon because of acute (C) MacConkey agar medium incubated at 37°C
swelling and pain in his right leg. Earlier that (D) Thiosulfate-citrate-bile-sucrose medium
morning, he had been working on a small sport incubated at 42°C
fishing boat in an estuary on the Gulf Coast of (E) Blood agar medium incubated at 37°C
Texas. While walking around the boat in shallow
water, he scratched his leg, breaking the skin at the
site of the current pain and swelling. He was not Chapter 18: Haemophilus,Bordetella, Brucella,
wearing boots. About 1 hour after the injury, the and Francisella
scratch became red and painful. Swelling
developed. Within 3 hours, the leg below the knee 1. A 68-year-old woman was seen in the clinic
had become markedly swollen. because she had felt feverish and had been
The skin was red and tender. There was serous experiencing increasing pain and swelling in her left
drainage from the wound, which had ulcerated and knee during the past 3 weeks. Four years earlier, a
was now much enlarged. Near the wound, bullae prosthetic joint had been placed in her left knee. On
were forming—the largest approximately 2.5 cm in examination, the knee was swollen, and fluid could
diameter. The most likely cause of this medical be detected. An aspirate of the fluid was obtained.
emergency is There were 15,000 polymorphonuclear cells/mL in
(A) Staphylococcus aureus the fluid. No organisms were seen on Gram stain.
(B) Streptococcus pyogenes A routine culture was done. On the fourth day of
(C) Clostridium perfringens incubation, colorless colonies smaller than 1 mm in
(D) Escherichia coli diameter were seen on the blood and chocolate
(E) Vibrio vulnificus agar plates. The organism was a tiny gram-negative
coccobacillus that was catalase positive and
10. The Vibrio cholerae factor responsible for oxidase positive. A urea slant was inoculated and
diarrhea is a toxin that was positive for urease activity after overnight
(A) BlocksEF-2 incubation. The patient was probably infected with
(B) Yields increased intracellular levels of cAMP which of the following microorganisms?
(C) Cleaves SNARE (A) Haemophilus influenzae
(D) Blocks EF-1-dependent binding of aminoacyl- (B) Haemophilus ducreyi
tRNA to ribosomes (C) Francisella tularensis
(E) Cleaves VAMP (D) Brucella species
(E) Staphylococcus aureus
11. In September 1854, a severe epidemic of
cholera occurred in the Soho/Golden Square area 2. After the culture (Question 1) turned positive,
of London. Dr. John Snow, a father of additional history was obtained. Approximately 4
epidemiology, studied the epidemic and helped stop weeks before the onset of her knee pain, the patient
it by which of the following actions? had visited relatives in Israel and traveled to other
(A) Banning the sale of apples at the local markets countries in the Mediterranean area. She had a
(B) Removing the handle of the Broad Street water particular fondness for one food product that was
pump the probable vehicle for her infection. The product
(C) Stopping the sale of shellfish imported from most likely was
Normandy (A) Bananas
(D) Pasteurizing milk (B) Unpasteurized goat’s cheese
(E) Promoted washing vegetables that were (C) Rare hamburger
consumed raw (D) Fresh orange juice
(E) Green tea
Jawetz Medical Microbiology 27th ed.
3. A 55-year-old game warden in Vermont found a both negative. Subcultures were done. Two days
dead muskrat on the bank of a stream. He picked later, very small colonies were seen on the sheep
up the animal, thinking it might have been illegally blood agar plate. The organism was a tiny gram
trapped or shot; it was not, and the game warden negative
buried it. Four days later, he developed a 1.5-cm coccobacillus that was catalase positive and
painful ulcer on the index finger of his right hand, a oxidase positive. It showed urease activity after 2
1-cm ulcer on his right forehead, and pain in his hours of incubation on urea-containing medium.
right axilla. Physical examination also revealed right This child had infection with
axillary lymphadenopathy. This patient is most likely (A) Brucella species
infected with (B) Mycobacterium tuberculosis
(A) Brucella species (C) Francisella tularensis
(B) Rickettsia rickettsii (D) Haemophilus influenzae
(C) Salmonella Typhi (E) Moraxella catarrhalis
(D) Haemophilus ducreyi
(E) Francisella tularensis 6. A 3-year-old child develops Haemophilus
influenzae meningitis. Therapy is begun with
4. An 18-month-old boy has been playing with a cefotaxime. Why is this third generation
child who develops Haemophilus influenzae cephalosporin used rather than ampicillin?
meningitis. The boy’s parents consult his (A) About 80% of Haemophilus influenzae
pediatrician, who says she is comfortable that the organisms have modified penicillin-binding proteins
child will be fine because he has been fully that confer resistance to ampicillin.
immunized with the polyribitol ribose phosphate (B) The drug of choice, trimethoprim–
(PRP)–protein conjugate vaccine. For what reason sulfamethoxazole, cannot be used because the
is it necessary to immunize infants of 2 months to child is allergic to sulfonamides.
2 years of age with polysaccharide–protein (C) It is easier to administer intravenous cefotaxime
conjugate vaccines? than intravenous ampicillin.
(A) The conjugate protein is diphtheria toxoid, and (D) There is concern that the child will rapidly
the goal is for the infant to develop simultaneous develop a penicillin (ampicillin) allergy.
immunity to diphtheria. (E) About 20% of Haemophilus influenzae
(B) Infants 2 months to 2 years of age do not organisms have a plasmid that encodes for β-
immunologically respond to polysaccharide lactamase.
vaccines that are not conjugated to a protein.
(C) The conjugate vaccine is designed for older 7. A 55-year-old man with severe dental caries
children and adults as well as infants. presented with 1 month of fever, malaise, and back
(D) Maternal (transplacental) antibodies against pain and now presents with moderately severe
Haemophilus influenzae are gone from the infant’s shortness of breath. The examination reveals a
circulation by 2 months of age. febrile man who appears pale and dyspneic. Other
(E) None of the above. physical findings include conjunctival petechiae, a
grade III/VI systolic murmur, and an enlarged
5. An 11-year-old boy from Peru was referred to the spleen. Blood cultures grow a pleomorphic
Brain Tumor Institute. Three months earlier he had gram-negative rod that is not hemolytic and that
developed headaches and then slowly progressive when tested is X and V factor negative. The most
right-sided weakness. A CT scan showed a mass likely causative pathogen is
lesion in the left hemisphere. He was thought to (A) Haemophilus influenzae
have a brain tumor. A lumbar puncture was not (B) Haemophilus ducreyi
done because of concern about increased (C) Aggregatibacter aphrophilus
intracranial pressure and brain herniation through (D) Actinobacillus hominis
the tentorium cerebelli. During surgery, a mass (E) Haemophilus parainfluenzae
lesion in the left hemisphere was found. Frozen
sections of the tissue were done while the patient 8. All of the following statements regarding acellular
was in the operating room. Microscopy of the pertussis vaccines are correct except
sections showed a granulomatous inflammatory (A) All formulations of the vaccine contain at least
reaction. No tumor was seen. Tissue was submitted two antigens.
for culture for Mycobacterium tuberculosis. (B) The acellular vaccine has replaced the whole
Middlebrook 7H9 broth medium was used. Six days cell vaccine in the childhood vaccine series.
after the culture was set up, the automated machine (C) All children should receive five doses of the
detected that the culture result was positive. vaccine before school entry.
Results of an acid-fast stain and a Gram stain were
Jawetz Medical Microbiology 27th ed.
(D) The vaccine is approved only for young children (D) Pertussis toxin
and adolescents. (E) Dermonecrotic toxin
(E) The vaccine is safer than and as immunogenic
as whole cell vaccines. 15. Which of the following pathogens discussed in
this chapter is on the select agent list?
9. Which of the following subspecies of Francisella (A) Haemophilus influenzae
tularensis is the most virulent for humans? (B) Aggregatibacter aphrophilus
(A) tularensis (C) Bordetella pertussis
(B) holarctica (D) Francisella tularensis
(C) mediasiatica (E) All of the above
(D) novicida
Chapter 19: Yersinia and Pasteurella
10. All of the following statements regarding the
etiologic agent of chancroid are correct except 1. An 18-year-old male resident of Arizona came to
(A) The organism is a small gram-negative rod. the emergency department (ED) complaining of
(B) The organism requires X factor but not V factor. fever, pain in his left groin, and diarrhea for the past
(C) The organism grows well on standard chocolate 2 days. On examination, he was afebrile, had a
agar. pulse rate of 126 beats/min, a respiratory rate of 20
(D) On Gram stain of lesions, the organism occurs breaths/min, and a blood pressure of 130/80 mm
in strands. Hg. Left groin swelling and tenderness were noted.
(E) The organism is susceptible to erythromycin. A groin muscle strain was diagnosed, attributed to a
fall 2 days earlier. He was treated with nonsteroidal
11. A 3-month-old infant is brought to the pediatric anti-inflammatory drugs and released. The next
emergency department in severe respiratory day, the patient reported feeling weak, had difficulty
distress. The child appears dehydrated, and there is breathing, and collapsed while taking a shower. He
a prominent peripheral lymphocytosis. The chest was transported to a hospital ED and pronounced
radiograph reveals perihilar infiltrates. The child’s dead shortly after arrival. Cultures of blood samples
grandmother, who watches the infant now that the obtained in the ED were positive for Yersinia pestis.
mother has returned to work, has had a dry hacking An epidemiologic investigation indicated that the
cough for about 2 weeks. The most likely causative patient most likely became infected as a result of
agent is bites by Y pestis–infected fleas while walking
(A) Haemophilus influenzae type b through a prairie dog colony (see Chapter 48).
(B) Bordetella pertussis Which of the following statements about the
(C) Streptococcus agalactiae pathogenesis of plague is correct?
(D) Chlamydia pneumoniae (A) Yersinia pestis produces a coagulase when
(E) Bordetella bronchiseptica incubated at 28°C.
(B) There is no risk for pneumonia caused by
12. In Question 11, the factor responsible for the person-to-person transmission of Yersinia pestis.
profound lymphocytosis is (C) Yersinia pestis organisms multiply in
(A) A hemagglutinin polymorphonuclear cells.
(B) A polysaccharide capsule (D) After the bite of an infected flea, Yersinia pestis
(C) An A/B structured toxin infection seldom, if ever, disseminates beyond the
(D) A heat-labile toxin site of the flea bite and the regional lymph nodes.
(E) A neuraminidase (E) Yersinia pestis is transmitted to animals (and
humans) in flea feces excreted when the flea is
13. All of the following cause zoonotic infections feeding.
except
(A) Francisella tularensis 2. The drug of choice to treat the patient in
(B) Brucella melitensis Question 1 would have been
(C) Bordetella pertussis (A) Ampicillin
(D) Bacillus anthracis (B) Cefotaxime
(E) Leptospira interrogans (C) Levofloxacin
(D) Erythromycin
14. Which of the following is not a recognized (E) Streptomycin
virulence factor of Bordetella pertussis?
(A) Heat-labile toxin 3. Yersinia pestis entered North America through
(B) Filamentous hemagglutinin San Francisco in the 1890s, carried by rats on ships
(C) Tracheal cytotoxin that had sailed from Hong Kong, where a plague
Jawetz Medical Microbiology 27th ed.
epidemic occurred. The current reservoir for Y (D) It is unique to Yersinia pestis.
pestis in the United States is (E) It encodes factors that are important for survival
(A) Urban feral cats in both the flea and the human.
(B) Urban rats
(C) Domestic cows 9. All of the following statements regarding the
(D) Coyotes epidemiology of infections caused by Yersinia
(E) Rural wild rodents enterocolitica are correct except
(A) Most human infections are caused by serotype
4. Which of the following is generally not considered O:1.
a potential agent of bioterrorism and biologic (B) Humans acquire the infection from ingestion of
warfare? food or drinks contaminated by animals or animal
(A) Yersinia pestis products.
(B) Botulinum toxin (C) Person-to-person spread is quite common.
(C) Streptococcus pyogenes (D) A large inoculum is required to cause infection.
(D) Brucella species (E) Infection is more prevalent in persons with
(E) Bacillus anthracis histocompatibility antigen HLA-B27.

5. An 8-year-old boy was bitten by a stray cat. Two 10. Optimum recovery of Yersinia enterocolitica
days later, the wound was red and swollen and from the stools of patients with gastroenteritis
drained purulent fluid. Pasteurella multocida was requires which of the following specialized media?
cultured from the wound. The drug of choice to treat (A) Cefsulodin-irgasan-novobiocin agar
this infection is (B) Xylose-lysine decarboxylase agar
(A) Amikacin (C) Hektoen-enteric agar
(B) Erythromycin (D) Regan-Lowe medium
(C) Gentamicin (E) MacConkey agar
(D) Penicillin G
(E) Clindamycin 11. An organism suspected of being Yersinia pestis
is recovered from a patient with sepsis. The isolate
6. Intimate contacts of patients with suspected has bipolar staining and is catalase positive but is
plague pneumonia should receive which of the oxidase and urease negative and is nonmotile. At
following agents as chemoprophylaxis? this point, what should be done?
(A) Gentamicin (A) Nothing; the laboratory has confirmed the
(B) Cefazolin diagnosis.
(C) Rifampin (B) Inoculate the isolate to an identification kit or
(D) Penicillin automated system for confirmation.
(E) Doxycycline (C) Call the police because there is a possible
bioterrorism event.
7. In a patient who has the bubonic form of plague, (D) Send the isolate to the nearest public health
all of the following specimens are acceptable for laboratory for confirmation.
diagnosis except (E) Send the isolate to the hospital across town for
(A) Stool culture on hektoen enteric agar sequencing.
(B) Blood culture using routine laboratory media
(C) Culture of a lymph node aspirate on blood and Chapter 20: The NeIsseriae
MacConkey agars
(D) Acute and convalescent serology 1. The inhabitants of a group of small villages in
(E) Immunohistochemical staining of lymph node rural sub-Saharan Africa experienced an epidemic
tissue of meningitis. Ten percent of the people died, most
of them younger than the age of 15 years. The
8. All of the following statements regarding the microorganism that most likely caused this epidemic
pFra/pMT plasmid of Yersinia pestis are true except was
(A) It encodes the capsular protein (fraction FI) that (A) Streptococcus agalactiae (group B)
confers antiphagocytic properties. (B) Escherichia coli K1 (capsular type 1)
(B) It contains genes that yield plasminogen- (C) Haemophilus influenzae serotype b
activating protease that has temperature-dependent (D) Neisseria meningitidis serogroup A
coagulase activity. (E) West Nile virus
(C) It contains genes that encode phospholipase D,
which is required for organism survival in the flea
midgut.
Jawetz Medical Microbiology 27th ed.
2. A 9-year-old boy presented to the clinic with a 5. A 38-year-old vice squad police officer comes to
the emergency department with a chief complaint
urethral discharge for the past 24 hours. Neisseria expressed as follows: “I have disseminated
gonorrhoeae was cultured from the specimen and gonococcal infection again.” He is correct. Cultures
of his urethra and knee fluid yield Neisseria
found to be β-lactamase positive and resistant to gonorrhoeae. He has previously had five episodes
high levels (≥32 μg/mL) of tetracycline. Which of the of disseminated gonococcal infection. The patient
should be evaluated for
following statements about these antimicrobial (A) Selective IgA deficiency
resistance factors is correct? (B) A polymorphonuclear cell chemotactic defect
(C) Deficiency of a late-acting complement
(A) β-lactamase production and high-level
component C5, C6, C7, or C8
resistance to tetracycline are both mediated by
(D) Absent lymphocyte adenosine deaminase
genes on plasmids.
activity
(B) Whereas β-lactamase production is mediated by
(E) Myeloperoxidase deficiency
a gene on the bacterial chromosome, high-level
tetracycline resistance is mediated by a gene on a
6. Which of the following individuals should
plasmid.
routinely receive vaccination with the conjugate
(C) Whereas β-lactamase production is mediated
meningococcal vaccine?
by a gene on a plasmid, high-level tetracycline
(A) A healthy young adolescent entering high
resistance is mediated by a gene on the bacterial
school
chromosome.
(B) A healthy child entering kindergarten
(D) β-lactamase production and high-level
(C) A 60-year-old man with insulin-dependent
resistance to tetracycline are both mediated by
diabetes
genes on the bacterial chromosome.
(D) A healthy 40-year-old technician who works in a
cancer research laboratory
3. A 6-year-old boy develops a fever and headache.
(E) A 65-year-old woman with coronary artery
He is taken to the emergency department, where he
disease
is noted to have a stiff neck, suggesting meningeal
irritation. A lumbar puncture is done, and culture of
7. A 25-year-old sexually active woman presents
the cerebrospinal fluid grows Neisseria meningitidis
with purulent vaginal discharge and dysuria 7 days
serogroup B. Which of the following should be
after having unprotected sexual intercourse with a
considered for his family (household) members?
new partner. Of the choices below, what is the most
(A) No prophylaxis or other steps are necessary.
sensitive diagnostic method for determining the
(B) They should be given Neisseria meningitidis
likely etiologic agent?
pilin vaccine.
(A) Gram stain
(C) They should be given Neisseria meningitidis
(B) An enzyme immunoassay
serogroup B polysaccharide capsule vaccine.
(C) Bacterial culture on selective media
(D) They should be given rifampin prophylaxis.
(D) A nucleic acid amplification test
(E) They should be given sulfonamide prophylaxis.
(E) Serology

4. An 18-year-old woman who reports unprotected


8. What is the currently recommended treatment for
sex with a new partner 2 weeks previously develops
gonococcal urethritis in men who have sex with
fever and left lower quadrant abdominal pain with
men in the United States?
onset in association with her menstrual period. On
(A) Single dose of an oral fluoroquinolone
pelvic examination in the emergency department,
(B) Seven days of oral doxycycline
she has bilateral tenderness when the uterus is
(C) Ceftriaxone given intramuscularly as a single
palpated. A mass 2–3 cm in diameter is felt on the
dose
left, suggestive of tubo-ovarian abscess.
(D) Spectinomycin given intramuscularly as a single
Subsequently, Neisseria gonorrhoeae is cultured
dose
from her endocervix. The diagnosis is gonococcal
(E) Seven days of oral amoxicillin
pelvic inflammatory disease. A common sequela of
this infection is:
9. Which of the following cell components produced
(A) Cancer of the cervix
by Neisseria gonorrhoeae is responsible for
(B) Urethral stricture
attachment to host cells?
(C) Uterine fibroid tumors
(A) Lipooligosaccharide
(D) Infertility
(B) Pili (fimbriae)
(E) Vaginal-rectal fistula
(C) IgA1 protease
Jawetz Medical Microbiology 27th ed.
(D) Outer membrane porin protein (A) Butyrate esterase
(E) Iron-binding protein (B) Gram stain
(C) Growth on 5% sheep blood agar
10. A 60-year-old man with severe chronic lung (D) PYR
disease presents with fever, cough productive of (E) Oxidase
purulent sputum, and worsening hypoxemia. A
sputum sample is collected, and the specimen Chapter 21: Infections caused by Anaerobic
is sent promptly to the laboratory. Microscopic Bacteria
examination of a Gram stain reveals numerous
polymorphonuclear leukocytes and predominately 1. A 55-year-old man visits his physician
gram-negative diplococci that are both intracellular complaining of a severe cough and production of
and extracellular. The organism grows well on 5% purulent sputum. His breath has a very unpleasant
SBA and chocolate agar and is positive for butyrate fetid odor. Chest radiography shows a large amount
esterase. What is the most likely organism causing of fluid in the left pleural space and a 5-cm lung
this man’s illness? cavity with an air-fluid level. A needle is inserted
(A) Neisseria gonorrhoeae through the chest wall, and some of the fluid in the
(B) Neisseria lactamica pleural space is removed; it is thick, yellow-gray in
(C) Moraxella catarrhalis color, and malodorous. Which of the following
(D) Haemophilus influenzae organisms or sets of organisms are most likely to be
(E) Neisseria meningitidis cultured from the pleural fluid?
(A) Bacteroides fragilis, Escherichia coli, and
11. One major advantage of the conjugate enterococci
meningococcal vaccines compared with the (B) Prevotella bivia, peptostreptococci, and
polysaccharide vaccine is Staphylococcus epidermidis
(A) Stimulation of mucosal secretory IgA (C) Prevotella melaninogenica, Fusobacterium
(B) Fewer side effects species, and viridans streptococci
(C) A T cell–dependent response to vaccine is (D) Propionibacterium species, peptostreptococci,
induced and Staphylococcus aureus
(D) Inclusion of serogroup B (E) Streptococcus pneumoniae

12. A 25-year-old woman presents with septic 2. An 18-year-old man develops fever with pain in
arthritis of the knee. The fluid aspirate grows a the right lower quadrant of his abdomen. After initial
gram-negative diplococcus on chocolate agar after evaluation, he is taken to the operating room.
48 hours of incubation. The isolate is oxidase During surgery, a ruptured appendix with an
positive and oxidizes glucose but not maltose, abscess is found. Bacteroides fragilis is cultured
lactose, or sucrose. You suspect infection with: from the abscess fluid. Which of the following
(A) Neisseria meningitidis factors promote abscess formation by B fragilis?
(B) Neisseria lactamica (A) Lipopolysaccharide
(C) Moraxella catarrhalis (B) Capsule
(D) Neisseria gonorrhoeae (C) Superoxide dismutase
(E) None of the above (D) Pili
(E) Leukocidin toxin
13. All of the following are virulence factors
associated with N gonorrhoeae except 3. Infections caused by Bacteroides species can be
(A) Pili treated with all of the following antibiotics except
(B) Por (A) Ampicillin–sulbactam
(C) Lipooligosaccharide (B) Clindamycin
(D) Opa proteins (C) Metronidazole
(E) A thick polysaccharide capsule (D) Penicillin
(E) Cefoxitin
14. The prevalence of gonococcal infections
increased between 2009 and 2012. 4. A 17-year-old high school senior develops
(A) True infectious mononucleosis. About 2 weeks later, he
(B) False develops a significantly higher fever, a worsening
sore throat, an inability to swallow, and severe
15. A useful test to differentiate Moraxella neck and chest pain. Upon admission, he has signs
catarrhalis from saprophytic neisseriae in of sepsis and respiratory distress. What is the most
respiratory samples is: likely organism causing this complication?
Jawetz Medical Microbiology 27th ed.
(A) Fusobacterium necrophorum (E) Staphylococcus aureus
(B) Bacteroides ovatus
(C) Prevotella melaninogenica Chapter 22: Legionella, Bartonella, and Unusual
(D) Clostridium tetani Bacterial Pathogens
(E) Actinomyces israelii
1. Humans become infected with Legionella
5. The drug of choice for treatment of infections pneumophila by
caused by Actinomyces species is (A) Kissing a person who is a legionella carrier
(A) Tigecycline (B) Breathing aerosols from environmental water
(B) Cefoxitin sources
(C) Metronidazole (C) Receiving a mosquito bite
(D) Imipenem (D) Consuming undercooked pork
(E) Penicillin
2. An 11-year-old girl developed an acute onset of
6. All of the following statements regarding fever, chills, headache, vomiting, and severe
anaerobes are true except migratory arthralgias (joint pain) and myalgias
(A) They possess the enzyme cytochrome oxidase (muscle pain). Two days later, she developed a
(B) Many species are part of the normal human maculopapular rash over her palms, soles, and
microbiota extremities. At the same time, her left knee became
(C) They are often found along with aerobes in extremely painful and swollen. On examination,
complicated infections fluid was demonstrated in the knee. Further history
(D) Special techniques are required to ensure their disclosed that the patient had a pet rat. Culture
recovery from clinical specimens of the fluid from her knee on 5% sheep blood agar
(E) Some species are more tolerant of exposure to showed 2-mm colonies after 3 days of incubation.
oxygen than others Broth culture showed small puffball-like growth.
Gram staining showed a gram-negative bacillus 0.5
7. Lemierre’s disease is a serious infection of the μm wide and 1–4 μm long. Some extremely long
head and neck associated with which of the forms (up to 150 μm) with bead-like chains, fusiform
following anaerobes? swellings, and large round bodies were seen. The
(A) Prevotella melaninogenica microbiologist who observed the Gram-stained
(B) Bacteroides thetaiotamicron smear immediately knew the cause of the girl’s
(C) Porphyromonas gingivalis infection to be
(D) Peptococcus niger (A) Pasteurella multocida
(E) Fusobacterium necrophorum (B) Streptobacillus moniliformis
(C) Francisella tularensis
8. Definitive identification of an anaerobe is likely (D) Bartonella bacilliformis
best accomplished by (E) Yersinia pestis
(A) Colony morphology on anaerobic media
(B) The presence of pigment 3. A 70-year-old man presents with bilateral
(C) Susceptibility to a variety of antimicrobial disks pneumonia. His Legionella urinary antigen test
(D) Cell wall fatty acid analysis using gas liquid result is positive. Which of the following is the likely
chromatography cause of his pneumonia?
(E) Gram stain morphology (A) Legionella pneumophila serogroup 1
(B) Legionella micdadei serogroup 4
9. A patient who has not maintained good dentition (C) Legionella bozemanii serogroup 2
presents with induration and swelling of the (D) Legionella longbeachae serogroup 2
mandibular area. On examination, you note (E) All of the above because the urinary antigen test
purulent material draining from a small opening. is genus specific and not species or serotype
The material appears yellowish, and there are some specific.
visible granules. You perform a Gram stain, and
pleomorphic gram-positive rods with short branches 4. A 65-year-old man comes to the emergency
are noted along with cells suggestive of acute and department feeling feverish and “really tired.” He
chronic inflammation. You suspect which of the has a chronic cigarette cough, but this has
following organisms? dramatically increased in the past week and he
(A) Bacteroides fragilis has been producing whitish sputum. The previous
(B) Lactobacillus acidophilus day he had a temperature of 38°C and watery
(C) Clostridium perfringens diarrhea. Physical examination reveals inspiratory
(D) Actinomyces israelii
Jawetz Medical Microbiology 27th ed.
and expiratory wheezes and rales over the right (E) The disease rapidly leads to sepsis even in
lower lung field. Chest radiography shows a patchy immunocompetent people.
right lower lobe infiltrate. The differential diagnosis
of this patient’s disease is 8. Which of the following statements about bacillary
(A) Streptococcus pneumoniae pneumonia angiomatosis is most correct?
(B) Legionella pneumophila pneumonia (A) It is caused by Bartonella bacilliformis.
(C) Haemophilus influenzae pneumonia (B) It is typically confined to the skin.
(D) Mycoplasma pneumoniae pneumonia (C) The major differential diagnosis is Kaposi
(E) All of the above sarcoma.
(D) The etiologic agent can be grown in 1–2 days in
5. Routine sputum cultures for the patient in routine culture on sheep blood agar.
Question 4 grow normal microbiota. Treatment with (E) Dogs are the reservoir for the etiologic agent.
ampicillin for 2 days yields no improvement. A
diagnosis of Legionnaires’ disease is considered, 9. An important factor in the pathogenesis of
and bronchoscopy is done to obtain bronchial Legionnaires’ disease is that
alveolar lavage fluid and deep airway specimens. (A) Legionella pneumophila kills polymorphonuclear
Which of the following would suggest a diagnosis of cells.
disease caused by Legionella pneumophila (B) Alveolar macrophages phagocytose Legionella
serotype 1? pneumophila using coiled pseudopods.
(A) Legionella urinary antigen assay (C) Legionella pneumophila invades pulmonary
(B) Direct fluorescent antibody on the bronchial capillaries, leading to dissemination and systemic
alveolar lavage fluid illness.
(C) Culture of the bronchial alveolar lavage on (D) Legionella pneumophila induces alveolar
charcoal yeast extract medium with antibiotics macrophage phagosomes to fuse with lysosomes.
(D) Antibody assay on paired (acute phase and (E) Legionella pneumophila outer surface protein A
convalescent phase) sera (OspA) is important for invasion of alveolar
(E) All of the above macrophages.

6. Charcoal is present in buffered charcoal yeast 10. True statements regarding Tropheryma whipplei
extract agar used to isolate Legionella pneumophila include all of the following except
to (A) It is easy to cultivate on chocolate agar after 3
(A) Provide the growth factors ordinarily provided by days of incubation.
free-living amebas present in environmental water (B) It is a gram-positive actinomycete.
(B) Serve as a carbon source for the growth of (C) It causes fever, abdominal pain, diarrhea,
Legionella pneumophila weight loss, and migratory polyarthralgia.
(C) Prevent hemolysis of the red blood cells in the (D) It stains with PAS.
medium
(D) Provide a dark background 11. All of the statements below regarding infections
(E) Act as a detoxifying agent with Legionella are correct except
(A) Hospitals that care for patients at risk for
7. A 23-year-old, otherwise healthy woman Legionella infections should know if their potable
presents with a 3-day history of low-grade fever and water systems contain Legionella.
headache. Examination reveals enlarged and (B) Human-to-human transmission is the major
slightly tender lymph nodes near her left elbow mechanism of transmission of Legionella infection.
and in the left axilla. Approximately 2 weeks earlier, (C) Legionella species can be visualized with Gram
she had visited a friend whose cat had scratched stain if carbolfuchsin is used for the counter stain.
her on the left arm; the site later developed a (D) The chest radiograph of a patient who has
reddish papule. Which of the following statements Legionella pneumonia is indistinguishable from that
about this disease is correct? of patients with pneumonia caused by other
(A) Characteristic histopathology in response to pathogens.
infection is acute, neutrophilic inflammation. (E) A macrolide or quinolone are the drugs of first
(B) The diagnosis is based on a suggestive history choice for treatment of Legionella infections.
and physical examination.
(C) β-Lactam/β-lactamase inhibitor combinations 12. Which of the following best represents the role
are the agents of choice for treatment. of the Mip protein in Legionella pathogenesis?
(D) The diagnosis is based on negative routine (A) It prevents phagosome–lysosome fusion.
bacterial cultures of pus aspirated from involved (B) It acts as a siderophore to capture iron.
lymph nodes. (C) It prevents phagocytosis.
Jawetz Medical Microbiology 27th ed.
(D) It facilitates adherence to the macrophage and 3. A 47-year-old woman presents with a 3-month
stimulates cellular invasion. history of progressive cough, weight loss, and fever.
(E) None of the above. Chest radiography shows bilateral cavitary disease
suggestive of tuberculosis. Sputum culture grows
13. Pontiac fever is a severe form of pneumonia an acid-fast bacillus that is a photochromogen
caused by Legionella pneumophila serotypes 1 and (makes an orange pigment when exposed to light).
6. The organism most likely is
(A) True (A) Mycobacterium tuberculosis
(B) False (B) Mycobacterium kansasii
(C) Mycobacterium gordonae
14. All of the following statements regarding (D) Mycobacterium avium complex
Streptobacillus moniliformis are correct except (E) Mycobacterium fortuitum
(A) It is susceptible to penicillin.
(B) It causes the disease rat-bite fever. 4. A 31-year-old Asian woman is admitted to the
(C) It causes Haverhill fever from ingestion of hospital with a 7-week history of increasing malaise,
contaminated food. myalgia, nonproductive cough, and shortness of
(D) The morphology of the organism is spiral breath. She has daily fevers of 38–39°C and a
shaped. recent 5-kg weight loss. She had a negative chest
radiograph when she entered the United States 7
15. The diagnosis of Whipple’s disease is best years ago. The patient’s grandmother died of
made by tuberculosis when the patient was an infant. A
(A) Paired serum obtained 8 weeks apart current chest radiograph is normal; results of other
(B) Prolonged culture on mycobacterial media tests show a decreased hematocrit and liver
(C) Nucleic acid amplification testing performed on function test abnormalities. Liver and bone marrow
tissue biopsies show granulomas with giant cells and acid-
(D) Histopathology fast bacilli. She is probably infected with
(E) None of the above (A) Mycobacterium leprae
(B) Mycobacterium fortuitum
(C) Mycobacterium ulcerans
Chapter 23: Mycobacteria (D) Mycobacterium gordonae
(E) Mycobacterium tuberculosis
1. A 60-year-old man has a 5-month history of
progressive weakness and a weight loss of 13 kg 5. It is very important that the patient in question 4
along with intermittent fever, chills, and a chronic also be evaluated for
cough productive of yellow sputum, occasionally (A) HIV/AIDS
streaked with blood. A sputum specimen is (B) Typhoid fever
obtained, and numerous acid-fast bacteria are seen (C) Liver abscess
on the smear. Culture of the sputum is positive for (D) Lymphoma
M tuberculosis. Which treatment regimen is most (E) Malaria
appropriate for initial therapy?
(A) Isoniazid and rifampin 6. Of concern regarding the patient in question 4 is
(B) Sulfamethoxazole–trimethoprim and that she could be infected with a Mycobacterium
streptomycin that is
(C) Isoniazid, rifampin, pyrazinamide, and (A) Susceptible only to isoniazid
ethambutol (B) Resistant to streptomycin
(D) Isoniazid, cycloserine, and ciprofloxacin (C) Resistant to clarithromycin
(E) Rifampin and streptomycin (D) Susceptible only to ciprofloxacin
(E) Resistant to isoniazid and rifampin
2. If the patient’s M tuberculosis isolate (question 1)
proves to be resistant to isoniazid, the likely 7. You observe a 40-year-old man begging on a
mechanism for resistance is street in a town in India. He has clawing of the
(A) β-Lactamase fourth and fifth digits with loss of distal parts of the
(B) Mutations in the catalase-peroxidase gene digits of both hands, strongly suggesting leprosy.
(C) Alterations in the β subunit of RNA polymerase The causative agent of this disease
(D) Mutations in the DNA gyrase gene (A) Is susceptible to isoniazid and rifampin
(E) Mutations in the genes encoding the S12 (B) Grows in parts of the body that are cooler than
protein and 16S rRNA 37°C
Jawetz Medical Microbiology 27th ed.
(C) Can be cultured in the laboratory using 11. Which of the following statements regarding
Middlebrook 7H11 medium interferon-γ release assays (IGRAs) is correct?
(D) Is seen in high numbers in biopsies of (A) They are useful for evaluating
tuberculoid leprosy lesions immunocompromised patients for active
(E) Commonly infects people in Texas because tuberculosis.
armadillos are hosts of M leprae (B) They detect antigens present in all
8. Which of the following statements about the Mycobacterium species.
purified protein derivative (PPD) and the tuberculin (C) They are not available yet for testing in the
skin test is most correct? United States.
(A) It is strongly recommended that medical and (D) They are performed using molecular probes that
other health science students have PPD skin tests detect organism DNA.
every 5 years. (E) They are used as alternatives to the tuberculin
(B) Persons immunized with BCG rarely, if ever, skin test to evaluate for latent tuberculosis.
convert to positive PPD skin test results.
(C) The intradermal skin test is usually read 4 hours 12. M abscessus most often causes pulmonary
after being applied. disease among which group of individuals?
(D) A positive tuberculin test result indicates that an (A) Young children exposed to dirt
individual has been infected with M tuberculosis in (B) African American smokers
the past and may continue to carry viable (C) Elderly, nonsmoking white females
mycobacteria. (D) Hispanic men who work outdoors
(E) A positive PPD skin test result implies that a (E) Persons living in the Northwestern United States
person is immune to active tuberculosis.
13. A newly characterized rapidly growing
9. A 72-year-old woman has an artificial hip joint Mycobacterium that has emerged as an important
placed because of degenerative joint disease. One cause of central venous catheter associated
week after the procedure, she has fever and joint infections is
pain. The hip is aspirated, and the fluid is submitted (A) Mycobacterium phlei
for routine culture and for culture for acid-fast (B) Mycobacterium mucogenicum
organisms. After 2 days of incubation, there is no (C) Mycobacterium xenopi
growth on any of the media. After 4 days, however, (D) Mycobacterium smegmatis
bacilli are seen growing on the sheep blood agar (E) Mycobacterium terrae
plate, and similar-appearing acid fast bacilli are
growing on the culture for acid-fast bacteria. The 14. The definition of extensively drug-resistant
patient is most likely infected with (XDR) tuberculosis includes
(A) Mycobacterium tuberculosis (A) Resistance to isoniazid
(B) Mycobacterium chelonae (B) Resistance to a fluoroquinolone
(C) Mycobacterium leprae (C) Resistance to capreomycin, amikacin or
(D) Mycobacterium kansasii kanamycin
(E) Mycobacterium avium complex (D) Resistance to rifampin
(E) All of the above
10. A 10-year-old child has a primary pulmonary M
tuberculosis infection. Which of the following 15. All of the following organisms are rapidly
features of tuberculosis is most correct? growing mycobacteria except
(A) In primary tuberculosis, an active exudative (A) Mycobacterium fortuitum
lesion develops and rapidly spreads to lymphatics (B) Mycobacterium abscessus
and regional lymph nodes. (C) Mycobacterium mucogenicum
(B) The exudative lesion of primary tuberculosis (D) Mycobacterium nonchromogenicum
often heals slowly. (E) Mycobacterium chelonae
(C) If tuberculosis develops years later, it is a result
of another exposure to M tuberculosis.
(D) In primary tuberculosis, all of the infecting M Chapter 24: Spirochetes and Other Spiral
tuberculosis organisms are killed by the patient’s Microorganisms
immune response.
(E) In primary tuberculosis, the immune system is 1. A 28-year-old woman who is 10 weeks pregnant
primed, but the PPD skin test result remains presents to the obstetrics clinic for prenatal care.
negative until there is a second exposure to M She has a history of treatment for syphilis 7 years
tuberculosis. previously. The results of serologic tests for syphilis
are as follows: nontreponemal test, RPR,
Jawetz Medical Microbiology 27th ed.
nonreactive; treponemal test (TP-PA), reactive. (C) Borreliae do not pass transovarially from one
Which of the following statements is most correct? generation to the next in ticks.
(A) The mother’s previous treatment for syphilis was (D) The main reservoir for the Borrelia is deer.
effective. (E) Borrelia is resistant to penicillin and tetracycline.
(B) The baby is at high risk for congenital syphilis.
(C) The mother needs to be treated again for 5. A 23-year-old man presented with a
syphilis. maculopapular rash over much of his trunk but not
(D) The mother needs a lumbar puncture and a in his mouth or on his palms. Because secondary
VDRL test of her CSF for neurosyphilis. syphilis was considered in the differential diagnosis,
a RPR test was done, and the result was positive at
2. A 12-year-old Boy Scout went to summer camp a 1:2 dilution. However, the TP-PA test result was
for 2 weeks in late August at a site located just negative. Which of the following diseases can be
outside Mystic, Connecticut. When he returned ruled out?
home, his mother noticed a bull’s-eyeshaped rash (A) Secondary syphilis
on the back of her son’s left calf. Shortly after Labor (B) Atypical measles
Day, the boy developed a flu-like illness that (C) Coxsackie virus infection
resolved after 4 days of bed rest. Three weeks (D) Acute HIV 1 infection
later, the boy complained to his mother that his (E) Allergic drug reaction
body hurt all over whenever he moved. This
prompted a visit to the pediatrician, who ordered an 6. Which of the following animals is the source of
infectious disease workup. What is the most likely Leptospira interrogans?
source of the boy’s infection? (A) Alligators
(A) Respiratory transmission from another sick (B) Ducks
camper (C) Frogs
(B) Ingestion of urine-contaminated water from a (D) Catfish
stream (E) Swine
(C) The bite of a mosquito harboring a parasite
(D) Ingestion of fecally contaminated food 7. A 27-year-old medical resident was admitted to
(E) The bite of a spirochete-infected tick the hospital because of sudden onset of fever to
39°C and headache. Two weeks previously, he had
3. Nontreponemal serological tests: vacationed in rural Oregon, where he had
(A) Are useful in definitively identifying a frequently gone swimming in an irrigation canal that
Treponema pallidum infection. bordered
(B) Measure antibodies against Treponema land where cows were pastured. Blood tests done
pallidum. shortly after admission indicated renal function
(C) Can be used to monitor antibiotic treatment of abnormality and elevated bilirubin and other liver
primary or secondary syphilis. function test results. Routine blood, urine, and CSF
(D) Measure antibodies against lipids released from culture results were negative. Leptospirosis is
damaged cells. suspected. Which of the following would be most
(E) Are useful in diagnosing a disseminated likely to confirm this diagnosis?
gonococcal infection. (A) Testing acute and convalescent phase sera
using the RPR test
4. A 42-year-old woman went camping in the Sierra (B) Culture of urine on human diploid fibroblast cells
Nevada Mountains, where she slept for two nights (C) Testing serum by dark-field examination for the
in an abandoned log cabin. After the second night, presence of leptospires
a tick was found on her shoulder. Six days later, (D) Testing acute and convalescent phase sera for
she developed fever to 38°C, which lasted for 4 antileptospiral antibodies
days. Ten days later, she had another similar (E) Culture of CSF on blood and chocolate agar
episode of (F) Gram stain of CSF and blood
fever. Examination of a blood smear stained with
Wright stain showed spirochetes suggestive of 8. A 47-year-old man presents with slowly
Borrelia species. Which of the following statements progressive arthritis in his knees. He enjoys hiking
about relapsing fever is correct? in the coastal areas of Northern California, where
(A) Each relapse is associated with an antigenically the prevalence of Borrelia burgdorferi in the Ixodes
distinct variant. ticks is known to be 1–3% (considered low). The
(B) Blood smears should be made when the patient patient is concerned about Lyme disease. He never
is afebrile. noticed a tick on his body and did not see an
Jawetz Medical Microbiology 27th ed.
expanding red rash. The result of an EIA for Lyme (D) Mycoplasma pneumoniae
borreliosis is positive. What should be done now? (E) Chlamydia trachomatis
(A) A biopsy specimen of the synovium of a knee
joint should be examined for Borrelia burgdorferi. 3. Which of the following is important in the
(B) The patient should be given an antibiotic to treat pathogenesis of mycoplasmal infections?
Lyme disease. (A) The peptidoglycan in the mycoplasmal cell wall
(C) PCR on the patient’s plasma should be done to (B) The presence of lacto-N-neotetraose with a
detect Borrelia burgdorferi. terminal galactosamine as the host cell receptor
(D) A serum specimen should be submitted for (C) The structures and the interactive proteins that
immunoblot assay to detect antibodies reactive with mediate adhesion to host cells
Borrelia burgdorferi antigens. (D) The absence of cilia on the surface of the host
(E) Culture of synovial fluid on blood and chocolate cells
agar. (E) Growth in an anatomic site where anaerobic
organisms thrive
9. Which of the following organisms principally
infects the liver and kidneys? 4. A 25-year-old woman is referred to the sexually
(A) Leptospira interrogans transmitted diseases clinic because of contact with
(B) Staphylococcus aureus a male partner with gonorrhea. The woman has had
(C) Escherichia coli 15 male sex partners since becoming sexually
(D) Enterococcus faecalis active. The likelihood that she also has genital
(E) Treponema pallidum Mycoplasma hominis infection is
(A) 1%
10. All of the following statements regarding (B) 5%
relapsing fever are correct except (C) 15%
(A) Epidemic disease carries a higher mortality rate (D) 40%
than endemic disease. (E) 90%
(B) Endemic disease in North America is caused by
Borrelia recurrentis. 5. A 25-year-old medical student has contact with a
(C) The recurrent febrile episodes are caused by patient who has pneumonia with fever and cough.
antigenic variation among the spirochetes. Four days later, the medical student develops fever
(D) Penicillin is the drug of choice. and cough, and chest radiographs show
(E) Crushing a tick could transmit the spirochetes. consolidation of the right lower lobe. Routine
bacterial sputum culture results are negative.
Pneumonia caused by Mycoplasma pneumoniae is
considered. All of the following are methods to
Chapter 25: Mycoplasmas and Cell Wall- confirm the clinical suspicion except
Defective Bacteria (A) PCR amplification of Mycoplasma pneumoniae
DNA in sputum
1. Ureaplasma urealyticum is so named because (B) Culture of sputum for Mycoplasma pneumoniae
(A) It thrives in the upper urinary tract. (C) Gram stain of sputum smear
(B) It requires urea as a growth substrate. (D) Culture of a lung aspirate for Mycoplasma
(C) It is a frequent cause of symptomatic urinary pneumoniae
bladder infections in young women. (E) Enzyme immunoassay test of acute and
(D) It causes chronic urinary tract infections in convalescent sera
premature babies born to mothers with
ureaplasmas as part of the genital flora. 6. Which type of test is most readily used to obtain
laboratory confirmation of Mycoplasma pneumoniae
2. An 18-year-old sexually active woman develops infection?
left lower quadrant pain and fever. On pelvic (A) Culture in broth containing serum, glucose, and
examination, she has tenderness in the left adnexa, a penicillin (to inhibit other flora)
and a mass suggestive of a uterine tube abscess is (B) PCR
palpated. The patient is diagnosed with pelvic (C) Electron microscopy
inflammatory disease. Which of the following (D) EIA tests on acute and convalescent phase
bacteria is considered to be a common cause of sera
pelvic inflammatory disease?
(A) Bacillus cereus 7. A 13-year-old boy develops infection with
(B) Haemophilus influenzae Mycoplasma pneumoniae. What is the risk for
(C) Neisseria subflava infection in other members of his household?
Jawetz Medical Microbiology 27th ed.
(A) None; it is sexually transmitted (C) Mice are the reservoir.
(B) 1–3% (D) Historically, the disease occurs in times of
(C) 10–15% prosperity.
(D) 20–40% (E) Recrudescence can occur many years after the
(E) 50–90% initial infection.

8. A 19-year-old man develops cough and fever. A 3. The most useful drug to treat ehrlichiosis is
chest radiograph shows consolidation of the left (A) Doxycycline
lower lobe. A diagnosis of pneumonia is made. (B) Penicillin G
Which of the following bacteria is a frequent (C) Trimethoprim–sulfamethoxazole
cause of community-acquired pneumonia? (D) Gentamicin
(A) Legionella pneumophila (E) Nitrofurantoin
(B) Chlamydia pneumoniae
(C) Streptococcus pneumoniae 4. A disease characterized by malaise, headache,
(D) Mycoplasma pneumoniae rigors, and fever developed in members of several
(E) Klebsiella pneumoniae families living in an unheated war-damaged house
in an Eastern European country. Erythematous 2–6
9. Initiation of infection by Mycoplasma pneumoniae mm macular red rashes appeared on the peoples’
begins with trunks and later on their extremities. Some of the
(A) Elaboration of a polysaccharide capsule that people had coughs. One elderly person, although
inhibits phagocytosis sick, was much less sick than other adults. The
(B) Secretion of a potent exotoxin people huddled together to keep warm; body lice
(C) Endocytosis by ciliated respiratory epithelial were common. Which of the following statements
cells is most correct?
(D) Adherence to respiratory epithelial cells (A) The disease that these people had is common
mediated by P1 adhesin in the Rocky Mountain states.
(E) Phagocytic uptake by alveolar macrophages (B) The elderly person may have had acute
epidemic typhus many years ago and recrudescent
10. Infection with Mycoplasma genitalium: typhus now.
(A) is not restricted to the genitourinary tract. (C) Fleas from rodents in the house were spreading
(B) results in inflammation causing urethritis in Rickettsia typhi.
males and cervicitis in females. (D) The primary host of the body louse infecting the
(C) is best treated with a first-generation people is the rat.
cephalosporin. (E) Epidemic typhus can be prevented by a
(D) is associated only with nongonococcal urethritis vaccine.
in males.
(E) is asymptomatic unless a co-infection with 5. Which of the following statements about
Chlamydia trachomatis is present. Ehrlichiae and ehrlichiosis is most correct?
(A) Dogs and mice are reservoirs.
(B) Mosquitoes are the vectors.
Chapter 26: Rickettsia and Related Genera (C) Ampicillin is the treatment of choice.
(D) Culture is a good method to confirm the
1. Morulae (intracellular inclusions in leukocytes) diagnosis.
are characteristic of which of the following (E) Ehrlichiae are typically found in lymphocytes.
diseases?
(A) Malaria caused by Plasmodium falciparum 6. A group of urban teenagers visited a sheep ranch
infection but not Plasmodium malariae infection in a large Western state for a 2-week experience.
(B) Dengue While they were there, many of the pregnant ewes
(C) Babesia infection delivered lambs to the delight of the closely
(D) Ehrlichiae infection observing teenagers. About 10 days later, three of
(E) Loa loa the teenagers developed flulike illnesses
characterized by malaise, cough, and fever. One
2. Which of the following statements about had an infiltrate on chest radiography, indicating
epidemic typhus (Rickettsia prowazekii disease) is pneumonia. The three teenagers had different
most correct? doctors, but the physicians each drew a blood
(A) The disease occurs primarily in sub-Saharan specimen and submitted it to the city health
Africa. department for serologic testing. All three
(B) It is transmitted by ticks. specimens were positive for Q fever. Public health
Jawetz Medical Microbiology 27th ed.
investigators determined that all of the teenagers palms, and soles. Initially the rash was macular, but
had been to the sheep ranch. When the it quickly evolved into maculopapules, some with
investigators contacted the ranch, they were told central petechiae. Rocky Mountain spotted fever
that there was no Q fever there and that no one caused by Rickettsia rickettsii was diagnosed.
who lived at the ranch had been sick. The most Which of the following statements about Rocky
likely explanation for the teenagers’ illnesses and Mountain spotted fever is correct?
the lack of illness at the ranch is (A) The vectors of Rickettsia rickettsii are ticks of
(A) There was no Q fever at the ranch, and it was the genus Ixodes.
acquired elsewhere. (B) A rash consistently appears by day 4 of illness.
(B) The people at the ranch had been previously (C) Rickettsia rickettsii forms inclusions in
immunized against Q fever. monocytes.
(C) The teenagers acquired Q fever at the ranch, (D) The patient’s antibody response may not occur
and the people who lived there had all previously until after the second week of illness.
had Q fever and were now immune to it. (E) The highest incidence of this disease is in the
(D) The teenagers had other illnesses, and the Rocky Mountain states.
positive Q fever serology result was unrelated.
(E) The public health laboratory had errors in the Q 11. The recommended treatment for Q-fever
fever serologic tests. endocarditis is
(A) Emergent surgery; antibiotics are not effective
7. A middle-aged sportsman, a resident of (B) Levofloxacin monotherapy for 6 weeks
Oklahoma, took a hike through a rural wooded and (C) 18 months of combination therapy with
brushy area near his home. The next morning, he doxycycline and hydroxychloroquine
noticed and removed a large (>1 cm) tick from (D) Penicillin and gentamicin combination therapy
his upper arm. About 1 week later, he experienced using IgG titers to determine duration
a gradual onset of fever and malaise. He now seeks
medical attention because he is concerned about a 12. Coxiella burnetii can be transmitted by milk
possible infection transmitted by the tick. Which of when animals such as goats and cows are infected.
the following diseases is most likely to be acquired The presently recommended conditions of “high-
from a tick? temperature, short-time” pasteurization
(A) Dengue are adequate to destroy viable Coxiella organisms.
(B) Rocky Mountain spotted fever (A) True
(C) Typhus (B) False
(D) Yellow fever
(E) Malaria 13. The histopathological hallmark of infection
caused by Rickettsia rickettsiae is
8. Which of the following drugs should not be used (A) Morulae within granulocytes
to treat Rocky Mountain spotted fever (Rickettsia (B) Morulae within monocytes
rickettsii infection)? (C) Granulomatous inflammation
(A) Trimethoprim–sulfamethoxazole (D) Intracellular vacuoles
(B) Chloramphenicol (E) Perivascular lymphocytes
(C) Doxycycline
14. All of the following statements regarding
9. Which of the following should be used to prevent Rickettsialpox are correct except
Rocky Mountain spotted fever (Rickettsia rickettsii (A) The cause of the disease is R akari.
infection)? (B) Ticks of the genus Amblyomma are responsible
(A) Attenuated Rickettsia rickettsii vaccine for transmission.
(B) Prophylactic doxycycline (C) The disease is mild.
(C) Preventing tick bites by wearing protective (D) Disease is more common in urban than rural
clothing areas.
(D) Delousing with insecticide
15. Reasons why C burnetii could be a potential
10. One week after deer hunting in a wooded area, agent of bioterrorism include
a 33-year-old man developed fever to 39°C with (A) It is acquired by the inhalation.
headache and malaise. Over the subsequent 24 (B) It is highly infectious.
hours, he developed nausea, vomiting, abdominal (C) It can be difficult to treat depending on the
pain, and diarrhea. On day 4, he developed a rash, phase of infection.
initially around the wrists and ankles, which then (D) Pneumonia may be severe.
progressively evolved, involving the arms, trunk, (E) All of the above.
Jawetz Medical Microbiology 27th ed.
Neisseria gonorrhoeae, Chlamydia trachomatis, or
both.
Chapter 27: Chlamydia spp. 351 (D) The endocervical specimen should be analyzed
for herpes simplex.
1. Which of the following statements about (E) Initial treatment is with ampicillin.
chlamydial antigens is correct? 5. The following statements about trachoma are
(A) Chlamydiae have shared group or genus- correct except
specific antigens. (A) It follows chronic or recurrent eye infection with
(B) There is no cross-reaction between Chlamydia Chlamydia trachomatis.
trachomatis and Chlamydia pneumoniae antigens. (B) Millions of people worldwide have trachoma.
(C) All five serovars of Chlamydia pneumoniae (C) Trachoma is readily prevented by a chlamydial
cross-react with Chlamydia psittaci. vaccine.
(D) One serovar of Chlamydia trachomatis causes (D) Progression of trachoma can be slowed by
eye infections, and the second serovar causes intermittent treatment with azithromycin.
genital infections. (E) Trachoma involves scarring of the conjunctiva,
eyelid deformities, and eyelash injury to the cornea.
2. The following are part of the control of Chlamydia 6. Elimination of blinding trachoma involves all of
psittaci and psittacosis in birds except the following except
(A) Quarantine of psittacine birds imported into the (A) Periodic administration of azithromycin
United States (B) Face washing and hygiene
(B) Only allowing sale of psittacine birds hatched in (C) Periodic culture screening of conjunctiva swab
the United States specimens for Chlamydia trachomatis
(C) Testing of birds for C psittaci infection (D) Environment improvements to sewage systems
(D) Controlling the shipment of psittacine birds to decrease the number of flies
(E) Putting tetracycline in the feed of psittacine (E) Surgery on deformed eyelids
birds
7. Which one of the following statements about
3. All of the following statements about perinatal Chlamydia pneumoniae is most correct?
Chlamydia trachomatis infections are correct except (A) Transmission from person to person is by the
(A) Between 15% and 40% of infants born to airborne route.
infected women develop inclusion conjunctivitis. (B) It makes glycogen-rich inclusions that stain with
(B) Between 10% and 20% of infants born to iodine.
infected women develop infant pneumonia. (C) There are multiple serovars, including three that
(C) The incubation period for Chlamydia cause a systemic illness.
trachomatis inclusion conjunctivitis is 1–2 days. (D) They are resistant to macrolides.
(D) The incubation period for infant pneumonia is (E) The reservoir is house cats.
typically 2–12 weeks.
(E) Ocular prophylaxis with erythromycin or 8. The serovars of Chlamydia trachomatis generally
tetracycline for neonatal Neisseria gonorrhoeae can be divided into groups representing their clinical
infection is generally not effective against neonatal infections and anatomic site infected. Which of the
Chlamydia trachomatis infection. following statements about the C trachomatis
(F) Infant pneumonia caused by Chlamydia serovars is most correct?
trachomatis often presents with a staccato cough. (A) There is no immunologic cross-reaction
between Chlamydia trachomatis serovars A, B, Ba,
4. An adolescent girl came to the clinic because of and D and the Chlamydia pneumoniae serovar.
a new and unusual vaginal discharge. She had (B) Serovars L1, L2, and L3 are associated with
recently become sexually active and had two new lymphogranuloma venereum.
partners during the previous month. On pelvic (C) The same Chlamydia trachomatis serovars are
examination, a purulent discharge was seen at the associated with blinding trachoma and sexually
opening of her endocervical canal. Which of the transmitted infections.
following statements about this patient is most (D) The antibody titer rise seen beginning around
correct? 6–8 years follows infections with Chlamydia
(A) A serologic test for syphilis is not indicated trachomatis serovars D–K.
because her symptoms are not those of syphilis.
(B) A Gram stain of her endocervical specimen 9. In the United States, it has long been known that
would show Chlamydia trachomatis inside the positive seroprevalence for Chlamydia
polymorphonuclear cells. trachomatis infection increases greatly during the
(C) The differential diagnosis includes infection with
Jawetz Medical Microbiology 27th ed.
primary school years (ages 6–10 years). A likely (E) Rhinovirus
reason for this is
(A) Frequent adenovirus infections 14. Inclusion conjunctivitis of the newborn
(B) Increased incidence of infections with (A) Is a mucopurulent conjunctivitis that occurs 7–
Chlamydia trachomatis 12 days after delivery
(C) Cross-reactive antibodies with M protein of (B) Is caused by Chlamydia psittaci
group A streptococci (Streptococcus pyogenes) (C) Is a result of exposure to pet birds in the home
(D) Children often have psittacosis (D) Is treated with systemic penicillin because it
(E) Frequent infections with Chlamydia pneumoniae may progress to pneumonia
(E) None of the above
10. All of the following statements about
lymphogranuloma venereum (LGV) are correct 15. The diagnostic method of choice for Chlamydia
except trachomatis pneumonia in the newborn is
(A) Chronic LGV proctitis can lead to rectal (A) A nucleic acid amplification test that targets the
strictures and fistula formation. ompA gene
(B) The disease is more common in northern (B) Culture of respiratory secretions in McCoy cells
latitudes. or other cell lines
(C) There may be marked systemic symptoms, (C) Enzyme immunoassay testing of respiratory
including fever, nausea, vomiting, headache, and secretions
meningismus. (D) IgG antibodies detected by complement fixation
(D) Chronic inflammation with LGV can lead to
lymphatic obstruction.
(E) Inguinal lymph nodes may become enlarged
and matted, draining pus through the skin. Chapter 28: Antimicrobial Chemotherapy
(F) A few days or weeks after exposure, the disease
manifests itself as a genital papule or vesicle. 1. The antimicrobial agent whose structure is shown
below is considered the drug of choice to treat
11. Which of the following methods are considered infections caused by which one of the following
the diagnostic tests of choice for urogenital microorganisms?
infections caused by Chlamydia trachomatis?
(A) Serology using complement fixation
(B) Cell culture using cycloheximide containing
McCoy cells
(C) Direct fluorescent antibody testing on urethral
and cervical specimens (A) Bacteroides fragilis
(D) Nucleic acid amplification methods (B) Pseudomonas aeruginosa
(E) Enzyme immunoassays performed on genital (C) Herpes simplex virus
tract specimens (D) Streptococcus pyogenes (group A streptococci)
(E) Mycobacterium tuberculosis
12. Nucleic acid amplification tests that are currently
available in the United States for diagnosing 2. Resistance of Staphylococcus aureus to the drug
chlamydial infections are approved for testing all of shown in Question 1 is caused by
the following specimens except (A) The action of acetyltransferase
(A) Self-collected vaginal swabs in women (B) The action of β-lactamase
(B) First void urine samples obtained from men (C) Substitution of the d-Ala-d-Ala dipeptide with the
(C) Rectal swabs obtained from children 12 years of d-Alad-Lac dipeptide in the cell wall peptidoglycan
age or younger (D) Decreased permeability of the bacterial cell wall
(D) Urethral swab samples obtained from adult men to the drug
(E) Cervical swab samples obtained from (E) Staphylococcus aureus being an intracellular
adolescent girls pathogen

13. Chlamydia pneumoniae pneumonia most 3. Streptococcus pneumoniae resistance to the


resembles infection caused which of the following drug shown in Question 1 is caused by
organisms? (A) The action of acetyltransferase
(A) Streptococcus pneumoniae (B) The action of β-lactamase
(B) Mycoplasma pneumoniae (C) Substitution of the d-Ala-d-Ala dipeptide with d-
(C) Haemophilus influenzae Alad-Lac dipeptide in the cell wall peptidoglycan
(D) Chlamydia trachomatis (D) Decreased permeability of the bacterial cell wall
Jawetz Medical Microbiology 27th ed.
(E) Genetically modified binding proteins in the (E) Vancomycin resistance in Staphylococcus
bacterial cell wall aureus

4. All of the following statements about antimicrobial 9. Which of the following factors is not generally
resistance of enterococci are correct except considered when selecting initial antimicrobial
(A) Enterococci are resistant to sulfamethoxazole– therapy for an infection?
trimethoprim in vivo. (A) Age of the patient
(B) Cephalosporins are not active against (B) Anatomic site of the infection (eg, meningitis or
enterococci. urinary tract infection)
(C) Resistance to the streptogramins (quinupristin– (C) Whether or not the patient is
dalfopristin) has emerged. immunocompromised
(D) Vancomycin-resistant enterococci are rare in (D) Whether or not the patient has implanted
Europe and the United States. devices in place (eg, artificial hip joint, artificial heart
(E) Vancomycin-resistant enterococci once valve, urinary catheter)
consistently clonal are now heterogeneous. (E) Waiting for culture and susceptibility test results

5. A 20-year-old Asian woman, a recent immigrant 10. All of the following agents have good activity
to the United States, develops fever and a cough against gram-positive organisms except
productive of blood-streaked sputum. She has lost (A) Daptomycin
6 kg of body weight in the past 6 weeks. Her chest (B) Vancomycin
radiograph shows bilateral upper lobe infiltrates with (C) Aztreonam
cavities. Given the history and chest radiography (D) Quinupristin–dalfopristin
findings, which of the following drug regimens (E) Tigecycline
would be the best appropriate initial therapy while
awaiting culture results? 11. Tigecycline, a new glycylcycline antibiotic with
(A) Isoniazid, rifampin, pyrazinamide, and good activity against a variety of pathogens, is best
ethambutol used for treatment of which of the following
(B) Penicillin G and rifampin infections?
(C) Cefotaxime, clindamycin, and trimethoprim– (A) Meningitis
sulfamethoxazole (B) Intra-abdominal infections caused by mixed
(D) Ampicillin–sulbactam aerobic and
(E) Vancomycin, gentamicin, and clindamycin anaerobic bacteria
(C) Neonatal sepsis
6. Aminoglycoside antibiotics typically cause which (D) Urethritis caused by Chlamydia trachomatis
of the following adverse events? (E) As monotherapy for bacteremia caused by
(A) They cause aplastic anemia. Acinetobacter baumannii
(B) They cause nonspecific stimulation of B cells. 12. Which of the following carbapenem antibiotics
(C) They cause ototoxicity and nephrotoxicity. has no activity against Pseudomonas aeruginosa?
(D) They cause photosensitivity. (A) Imipenem
(B) Meropenem
7. Which one of the following groups of (C) Doripenem
antimicrobial agents acts on microorganisms by (D) Ertapenem
inhibiting protein synthesis?
(A) Fluoroquinolones 13. Which of the following agents would not be
(B) Aminoglycosides expected to demonstrate postantibiotic affect
(C) Penicillins against gram-negative bacilli?
(D) Glycopeptides (eg, vancomycin) (A) Imipenem
(E) Polymyxins (B) Ciprofloxacin
(C) Gentamicin
8. There are many bacterial–antimicrobial (D) Ampicillin
resistance combinations. Which one of the following
is of major international concern? 14. All of the following are common mechanisms of
(A) Sulfonamide resistance in Neisseria meningitidis resistance to the penicillins except
(B) Penicillin G resistance in Neisseria gonorrhoeae (A) Production of β-lactamases
(C) Ampicillin resistance in Haemophilus influenzae (B) Alterations in target receptors (PBPs)
(D) Erythromycin resistance in Streptococcus (C) Inability to activate autolytic enzymes
pyogenes (group A streptococci) (D) Failure to synthesize peptidoglycans
(E) Methylation of ribosomal RNA
Jawetz Medical Microbiology 27th ed.
(C) Hemagglutination
15. The drug of first choice for the treatment of (D) Polymerase chain reaction
serious anaerobic infections caused by Bacteroides (E) Enzyme immunoassay
fragilis is
(A) Clindamycin 5. Which one of the following states a principle
(B) Ampicillin regarding viral nucleic acid?
(C) Cefoxitin (A) Viruses contain both RNA and DNA.
(D) Metronidazole (B) Some viruses contain a segmented genome.
(E) Amoxicillin–clavulanate (C) Purified viral nucleic acid from any virus is
usually infectious.
(D) Viral genome sizes are similar among known
human viruses.
Section IV: Virology
6. Two mutants of poliovirus have been isolated,
Chapter 29: General Properties of Viruses one (MutX) with a mutation in gene X and the
second (MutY) with a mutation in gene Y. If cells
1. Some viruses are characterized by helical are infected with each mutant alone, no virus is
symmetry of the viral nucleocapsid. Which of the produced. If a cell is coinfected with both MutX and
following statements about viruses with helical MutY, which one of the following is most likely to
symmetry is most accurate? occur?
(A) All enveloped viruses with helical symmetry are (A) Reassortment of genome segments may occur
classified into the same virus family. and give rise to a viable wild-type virus.
(B) Helical nucleocapsids are found primarily in (B) The genomes may be reverse transcribed to
DNA-containing viruses. DNA and both MutX and MutY viruses produced.
(C) All human viruses with helical nucleocapsids (C) Complementation between the mutant gene
possess an envelope. products may occur and both MutX and MutY
(D) Excess empty helical particles containing no viruses produced.
nucleic acid are commonly produced in infected (D) The cells will transform at high frequency
cells. because they will not be killed by the poliovirus
mutants.
2. Virus-infected cells often develop morphologic
changes referred to as cytopathic effects. Which of 7. Which one of the following viruses possesses an
the following statements about virus-induced RNA genome that is infectious when purified?
cytopathic changes is most accurate? (A) Influenza virus
(A) They are pathognomonic for an infecting virus. (B) Poliovirus
(B) They are rarely associated with cell death. (C) Papillomavirus
(C) They may include giant cell formation. (D) Measles virus
(D) They can only be seen with an electron (E) Rotavirus
microscope. 8. Viruses belonging to which of the following
groups are likely to establish latent infections?
3. Viruses usually initiate infection by first (A) Poxviruses
interacting with receptors on the surface of cells. (B) Filoviruses
Which of the following statements is most accurate (C) Herpesviruses
about cellular receptors for viruses? (D) Influenza viruses
(A) Cellular receptors for viruses have no known (E) Caliciviruses
cellular function.
(B) All viruses within a given family use the same 9. Some viruses encode for a viral RNA-dependent
cellular receptor. RNA polymerase. Which of the following states a
(C) All cells in a susceptible host express the viral principle about viral RNA polymerases?
receptor. (A) All RNA viruses carry RNA polymerase
(D) Successful infection of a cell by a virus may molecules inside virus particles because they are
involve interaction with more than one type of needed to initiate the next infectious cycle.
receptor. (B) Antibodies against the viral RNA polymerase
neutralize virus infectivity.
4. Which of the following can be used to quantitate (C) Negative-strand RNA viruses supply their own
the titer of infectious viruses? RNA-dependent RNA polymerase because
(A) Plaque assay eukaryotic cells lack such enzymes.
(B) Electron microscopy
Jawetz Medical Microbiology 27th ed.
(D) The viral RNA polymerase protein also serves 15. Arboviruses are classified into several different
as a major core structural protein in the virus virus families but are grouped together based on
particle. which of the following common characteristics?
(A) Replicate only in humans
10. Which of the following statements regarding (B) Contain both RNA and DNA
virus morphology is true? (C) Are transmitted by vectors
(A) All RNA viruses are spherical in shape. (D) Cause hemorrhagic fevers
(B) Some viruses contain flagella. (E) Cause encephalitis
(C) Some viruses with DNA genomes contain a
primitive nucleus.
(D) Viral surface proteins protect the viral genome Chapter 30: Pathogenesis and Control of Viral
from nucleases. Diseases
(E) Helical nucleocapsids are found with single-
stranded DNA viruses. 1. Interferons are an important part of the host
defense against viral infections. What is interferon’s
11. Many viruses can be grown in the laboratory. principal mode of action?
Which of the following statements about virus (A) It is present in the serum of healthy individuals,
propagation is not true? providing a viral surveillance role.
(A) Some viruses can be propagated in cell-free (B) It coats viral particles and blocks their
media. attachment to cells.
(B) Some mammalian viruses can be cultivated in (C) It induces synthesis of one or more cellular
hen’s eggs. proteins that inhibit translation or transcription.
(C) Some viruses with broad host ranges can (D) It protects the virus-infected cell that produced it
multiply in many types of cells. from cell death.
(D) Some human viruses can be grown in mice.
(E) Most virus preparations have particle-to- 2. A 9-month-old girl is taken to the emergency
infectious unit ratios greater than 1. room because of fever and persistent cough. Rales
are heard in her left chest on physical examination.
12. Laboratory infections can be acquired when An infiltrate in her left lung is seen on
working with viruses unless good laboratory safety the chest radiograph. Pneumonia is diagnosed.
practices are followed. Which of the following is not Which of the following is the most likely cause?
a good biosafety practice? (A) Rotavirus
(A) Use of biosafety hoods (B) Rhinovirus
(B) Use of laboratory coats and gloves (C) Adenovirus
(C) Avoidance of pipetting by mouth (D) Respiratory syncytial virus
(D) Flushing experimental waste down laboratory (E) Coxsackievirus
sink
(E) Not eating or drinking in the laboratory 3. Which one of the following is a fundamental
principle of viral disease causation?
13. Small viruses are in the same size range as (A) One virus type induces a single disease
which of the following? syndrome.
(A) Staphylococcus species (B) Many viral infections are subclinical and do not
(B) Serum globulin produce clinical disease.
(C) Red blood cells (C) The type of disease produced by a virus can be
(D) Eukaryotic ribosomes predicted by the morphology of that virus.
(E) Mitochondria (D) A particular disease syndrome has a single viral
cause.
14. Which of the following is not an important factor
contributing to the phenomenon of emerging viral 4. The skin is a formidable barrier to virus entry, but
diseases? a few viruses are able to breach this barrier and
(A) International air travel initiate infection of the host. Which of the following
(B) Antibiotic resistance is an example of a virus that enters through skin
(C) Deforestation abrasions?
(D) War (A) Adenovirus
(E) Organ and tissue transplantation (B) Rotavirus
(C) Rhinovirus
(D) Papilloma virus
(E) Influenza virus
Jawetz Medical Microbiology 27th ed.
9. What type of hepatitis B vaccine is currently in
5. A 40-year-old man has HIV/AIDS characterized use in the United States?
by a low CD4 count and a high viral load. Highly (A) Synthetic peptide vaccine
active antiretroviral therapy (HAART) will be (B) Killed-virus vaccine
initiated. One of the drugs under consideration (C) Attenuated live-virus vaccine
is a nucleoside analog that inhibits viral reverse (D) Subunit vaccine produced using recombinant
transcriptase and is active against both HIV and DNA
hepatitis B virus. That drug is:
(A) Acyclovir 10. Which one of the following phrases accurately
(B) Amantadine describes viral neutralizing antibodies?
(C) Ribavirin (A) Directed against viral protein determinants on
(D) Saquinavir the outside of the virus particle
(E) Lamivudine (B) Appear in the host sooner after viral infection
(F) Fuzeon than interferon
(C) Directed against viral nucleic acid sequences
6. Regarding the HIV/AIDS patient in Question 5, a (D) Induced only by disease-causing viruses
peptidomimetic agent that blocks virus-mediated (E) Of little importance to immunity to viral infection
cleavage of viral structural protein precursors is
chosen as a second drug. That drug is: 11. Many viruses use the respiratory tract as the
(A) Acyclovir route of entry to initiate infections. Which of the
(B) Amantadine following virus groups does not?
(C) Ribavirin (A) Adenovirus
(D) Saquinavir (B) Coronavirus
(E) Lamivudine (C) Hepadnavirus
(F) Fuzeon (D) Paramyxovirus
(E) Poxvirus
7. A 63-year-old woman is hospitalized for
treatment of leukemia. One day after admission she 12. Which of the following licensed virus vaccines is
develops chills, fever, cough, headache, and a subunit vaccine prepared using recombinant DNA
myalgia. She states that her husband had a similar technology?
illness a few days earlier. There is major concern (A) Measles–mumps–rubella
about a respiratory virus outbreak in the staff of the (B) Varicella
chemotherapy ward and in the patients on that (C) Hepatitis A
ward. A synthetic amine that inhibits influenza A (D) Papilloma
virus by blocking viral uncoating is chosen for (E) Rotavirus
prophylactic treatment of the staff and patients. (F) Rabies
That drug is:
(A) Acyclovir 13. Which of the following viruses is the most
(B) Amantadine common cause of neonatal infections in the United
(C) Ribavirin States?
(D) Saquinavir (A) Rubella
(E) Lamivudine (B) Parvovirus B19
(F) Fuzeon (C) Hepatitis B
(D) Cytomegalovirus
8. Which one of the following statements describes (E) Varicella
an advantage of killed-virus vaccines over (F) Human immunodeficiency virus
attenuated live-virus vaccines?
(A) Killed-virus vaccines induce a broader range of 14. Which one of the following statements
immune responses than do attenuated live-virus concerning interferonsis least accurate?
vaccines. (A) Interferons are proteins that influence host
(B) Killed-virus vaccines more closely mimic natural defenses in many ways, one of which is the
infections than do attenuated live-virus vaccines. induction of an antiviral state.
(C) Killed-virus vaccines pose no risk that vaccine (B) Interferons are synthesized only by virus-
virus might be transmitted to susceptible contacts. infected cells.
(D) Killed-virus vaccines are efficacious against (C) Interferons inhibit a broad range of viruses, not
respiratory virus infections because they induce just the virus that induced the interferon.
good mucosal immunity. (D) Interferons induce the synthesis of a
ribonuclease that degrades viral mRNA.
Jawetz Medical Microbiology 27th ed.
5. Which one of the following is a disease in which
15. Each of the following statements concerning the role of parvovirus B19 has not been
viral vaccines is correct except: established?
(A) In live attenuated vaccines, the virus has lost its (A) Erythema infectiosum (fifth disease)
ability to cause disease but has retained its ability to (B) Transient aplastic crisis
induce neutralizing antibody. (C) Hydrops fetalis
(B) In live attenuated vaccines, the possibility of (D) Fulminant hepatitis
reversion to virulence is of concern.
(C) With inactivated vaccines, IgA mucosal 6. Which one of the following best describes the
immunity is usually induced. replication of human parvovirus B19?
(D) With inactivated vaccines, protective immunity is (A) Stimulates resting cells to proliferate
mainly caused by the production of IgG. (B) Uses blood group antigen P as cellular receptor
(C) Readily establishes persistent infections
(D) Entire replication cycle occurs in cytoplasm
(E) Production of infectious progeny requires the
presence of a helper virus
Chapter 31: Parvoviruses 7. Which one of the following statements is most
accurate concerning human infections by
1. Which one of the following best describes a parvovirus B19?
physicochemical property of parvoviruses? (A) Parvovirus B19 is transmitted readily by sexual
(A) Enveloped virus particle. intercourse.
(B) Single-stranded DNA genome. (B) Patients with disseminated disease caused by
(C) Infectivity is inactivated by ether treatment. parvovirus
(D) Virion exhibits helical symmetry. B19 should be treated with acyclovir.
(E) Virion is about the same size as herpesviruses. (C) Parvovirus B19 does not cause any human
disease.
2. An 8-year-old child recently had erythema (D) There is no vaccine for human parvovirus.
infectiosum. Her 33-year-old mother subsequently
developed arthralgia followed by painful arthritis 8. Human bocavirus is a newly discovered
with swelling in the small joints of both hands. In parvovirus. It has been detected most frequently in
addition to the apparent tropism for joints, human which type of sample?
parvovirus B19 is highly tropic for which cell type? (A) Urine
(A) CD4 T lymphocytes (B) Cord blood
(B) Renal tubule cells (C) Respiratory secretions
(C) Erythroid cells (D) Fetal liver
(D) Glial cells (E) Bone marrow
(E) Peyer patches
9. Which of the following is available as a treatment
3. The 8-year-old child in Question 2 had an illness or preventive for parvovirus B19 infections?
with more than one phase. Which symptoms (A) Commercial immunoglobulin
coincide with the second phase of the illness? (B) Vaccine containing recombinant VP2 viral
(A) Sore throat antigen
(B) Skin rash (C) Bone marrow transplantation
(C) Headache (D) Antiviral drug that blocks virus–receptor
(D) Diarrhea interaction
(E) Cough
10. Human erythroviruses and bocaviruses share
4. A 42-year-old man with HIV/AIDS presented with the following properties except for which one?
aplastic anemia. Using the PCR, parvovirus B19 (A) Small, nonenveloped virus particles.
was detected in his serum. The patient presumably (B) Difficult to culture.
acquired his parvovirus B19 infection from another (C) Cause anemia.
person. The most likely route of transmission is: (D) Global distribution.
(A) By contact with respiratory secretions or (E) No vaccine exists.
droplets
(B) By contact with a skin rash Chapter 32: Adenoviruses
(C) Through sexual activity
(D) Through a recent blood transfusion
Jawetz Medical Microbiology 27th ed.
1. What adenovirus protein or proteins regulate (A) Types 40 and 41
early transcription of the viral genes and modulate (B) Types 8, 19, and 37
the cell cycle? (C) Types 1, 2, 5, and 6
(A) Fiber (D) Types 3, 4, and 7
(B) Hexon (E) Types 21, 22, 34, and 35
(C) Penton
(D) Terminal protein 7. Which adenovirus types are frequent causes of
(E) E1 region protein acute respiratory disease among military recruits?
(F) Cysteine proteinase (A) Types 40 and 41
(G) E3 region protein (B) Types 8, 19, and 37
(C) Types 1, 2, 5, and 6
2. What adenovirus protein serves as primer for (D) Types 3, 4, and 7
initiation of viral DNA synthesis? (E) Types 21, 22, 34, and 35
(A) Fiber
(B) Hexon 8. Which of the following events led to
(C) Penton reappearance of acute respiratory disease
(D) Terminal protein outbreaks among U.S. military recruits in the late
(E) E1 region protein 1990s?
(F) Cysteine proteinase (A) Emergence of a new virulent strain of
(G) E3 region protein adenovirus
(B) Cessation of adenovirus vaccination program for
3. What adenovirus protein comprises the majority recruits
of capsomeres making up the virus capsid? (C) Change in military housing and training
(A) Fiber conditions for recruits
(B) Hexon (D) Cessation of adenovirus antiviral drug therapy
(C) Penton program for recruits
(D) Terminal protein
(E) E1 region protein 9. Your summer research project is to study the
(F) Cysteine proteinase viruses that cause gastroenteritis. You recover a
(G) E3 region protein virus from a stool sample and notice that the growth
medium on the infected cultures is highly acidic.
4. A 3-month-old infant had watery diarrhea and You find that the viral genome is double-stranded
fever for 10 days. Rotavirus or adenovirus types 40 DNA. Of the following, which one is the most
and 41 are the suspected agents. What type of appropriate conclusion you could draw?
specimen would be most appropriate for detection (A) There is a high likelihood that the agent is a
of adenovirus types 40 and 41 infection in this rotavirus.
patient? (B) You need to determine the viral serotype to
(A) Blood establish whether the virus was important in
(B) Urine causing the disease.
(C) Conjunctival swab (C) The patient should have been treated with the
(D) Stool antiviral drug amantadine to shorten the duration of
(E) Throat swab symptoms.
(F) Cerebrospinal fluid (D) The virus particle would contain a reverse
transcriptase enzyme.
5. Which of the following human diseases has not
been associated with adenoviruses? 10. Which of the following groups of individuals is at
(A) Cancer the lowest risk of adenovirus disease?
(B) Common colds (A) Healthy adults
(C) Acute respiratory diseases (B) Young children
(D) Keratoconjunctivitis (C) Bone marrow transplant recipients
(E) Gastroenteritis (D) Military recruits
(F) Hemorrhagic cystitis (E) AIDS patients

6. A 2½-year-old child attending nursery school 11. Adenoviruses can cause eye infections that are
acquires a mild respiratory infection. Other children highly contagious. Which of the following is least
in the nursery school have similar illnesses. Which likely to be a means of transmission during an
adenovirus types are the most likely causes of the outbreak of epidemic keratoconjunctivitis?
illnesses? (A) Swimming pools
Jawetz Medical Microbiology 27th ed.
(B) Hand towels (C) Herpes simplex virus type 2
(C) Mosquito bites (D) Kaposi sarcoma herpesvirus
(D) Hand-to-eye (E) Varicella-zoster virus
(E) Contaminated ophthalmic equipment
4. A 19-year-old female college student has a fever,
12. There are 57 known sero types of human sore throat, and lymphadenopathy accompanied by
adenoviruses. Which of the following statements is lymphocytosis with atypical cells and an increase in
most accurate? sheep cell agglutinins. The diagnosis is most likely
(A) Types cannot be distinguished serologically. (A) Infectious hepatitis
(B) All cause respiratory infections in children. (B) Infectious mononucleosis
(C) Most types replicate well in T lymphocytes. (C) Chickenpox
(D) Two types can cause gastroenteritis. (D) Herpes simplex infection
(E) Viral meningitis
13. Each of the following statements concerning
adenoviruses is correct except 5. A Tzanck smear of a scraping obtained from a
(A) Adenoviruses are composed of a double- vesicle on the skin demonstrates multinucleated
stranded DNA giant cells. Multinucleated giant cells are associated
genome and a capsid without an envelope. with which of the following viruses?
(B) Adenoviruses cause both sore throat and (A) Varicella-zoster
pneumonia. (B) Variola major
(C) Adenoviruses have only one serologic type. (C) Coxsackievirus
(D) Adenoviruses are implicated as a cause of (D) Molluscum contagiosum
tumors in animals but not humans.
6. Which of the following statements about beta-
herpesviruses is not true?
14. Which of the following conditions is least likely (A) They establish latent infections and persist
to be caused by adenoviruses? indefinitely in infected hosts.
(A) Conjunctivitis (B) They are reactivated in immunocompromised
(B) Pneumonia patients.
(C) Pharyngitis (C) Most infections are subclinical.
(D) Glomerulonephritis (D) They can infect lymphoid cells.
(E) They have short, cytolytic growth cycles in
Chapter 33: Herpesvirus cultured cells.

1. A previously healthy 3-year-old boy develops a 7. A 28-year-old woman has recurrent genital
classic viral childhood illness. Which of the following herpes. Which of the following statements about
primary viral infections of childhood is usually genital herpes infections is true?
symptomatic? (A) Reactivation of latent virus during pregnancy
(A) Cytomegalovirus poses no threat to the newborn.
(B) Epstein-Barr virus (B) Virus cannot be transmitted in the absence of
(C) Hepatitis B virus apparent lesions.
(D) Varicella-zoster virus (C) Recurrent episodes caused by reactivation of
(E) Parvovirus B19 latent virus tend to be more severe than the primary
infection.
2. Which one of the following is a recommended (D) They can be caused by either herpes simplex
therapy for herpes simplex virus genital infection? virus type 1 or type 2.
(A) Acyclovir (E) Latent herpes simplex virus can be found in
(B) Attenuated live virus vaccine dendritic cells.
(C) Herpes immune globulin
(D) Interferon-α 8. Which of the following viruses causes a
(E) Ribavirin mononucleosis-like syndrome and is excreted in the
urine?
3. Most herpesvirus infections are endemic (A) Cytomegalovirus
worldwide. Which one of the following viruses (B) Epstein-Barr virus
shows marked geographic differences in (C) Human herpesvirus 6
seroprevalence? (D) Varicella-zoster virus
(A) Cytomegalovirus (E) Herpes simplex virus type 2
(B) Epstein-Barr virus
Jawetz Medical Microbiology 27th ed.
9. A 53-year-old woman develops fever and focal 15. Which one of the following is the best
neurologic signs. Magnetic resonance imaging explanation for the selective action of acyclovir
shows a left temporal lobe lesion. Which of the (acycloguanosine) in herpes simplex virus (HSV)-
following tests would be most appropriate to confirm infected cells?
a diagnosis of herpes simplex encephalitis in this (A) Acyclovir binds specifically to viral receptors
patient? only on the surface of the HSV-infected cell.
(A) Brain biopsy (B) Acyclovir is phosphorylated by a virus-encoded
(B) Tzanck smear phosphokinase only within HSV-infected cells.
(C) Polymerase chain reaction assay for viral DNA (C) Acyclovir selectively inhibits the RNA
in cerebrospinal fluid polymerase in the HSV virion.
(D) Serologic test for viral IgM antibody (D) Acyclovir specifically blocks the matrix protein of
HSV, thereby preventing release of progeny HSV.
10. Which of the following tumors is caused by a
virus other than Epstein-Barr virus? 16. Each of the following statements concerning
(A) Posttransplant lymphomas herpesvirus latency is correct except
(B) Hodgkin disease (A) Exogenous stimuli can cause reactivation of
(C) Kaposi sarcoma latent infection, with induction of symptomatic
(D) AIDS-related central nervous system non- disease.
Hodgkin lymphomas (B) During latency, antiviral antibody is not
(E) Burkitt lymphoma demonstrable in the sera of infected individuals.
(C) Reactivation of latent herpesviruses is more
11. An outbreak of a rash called “mat herpes” common in patients with impaired cell-mediated
occurred among high school students who had immunity than in immunocompetent patients.
competed in a wrestling tournament. Which of the (D) Virus can be recovered from latently infected
following statements is most accurate? cells by cocultivation with susceptible cells.
(A) The rash is not contagious among wrestlers.
(B) The causative agent is herpes simplex virus 17. Vaccines have been demonstrated to be
type 1. efficacious in preventing herpesvirus disease in
(C) The causative agent is varicella-zoster virus. which one of the following situations?
(D) Lesions typically last 1 month or longer. (A) Herpes simplex virus type 1 primary infection
(E) Students should be vaccinated before (B) Herpes simplex virus type 2 reactivation
participating in wrestling tournaments. (C) Varicella-zoster reactivation
(D) Cytomegalovirus primary infection
12. The shingles vaccine is recommended for which (E) Epstein-Barr virus reactivation
of the following groups?
(A) Healthy adolescents 18. Herpes simplex virus and cytomegalovirus
(B) Individuals older than age 60 years share many features. Which one of the following
(C) Pregnant women features is least likely to be shared?
(D) Those who never had chickenpox (A) Important cause of morbidity and mortality in the
newborn
13. The most common congenital infection is (B) Congenital abnormalities caused by
caused by: transplacental passage
(A) Varicella-zoster virus (C) Important cause of serious disease in
(B) Herpes simplex virus type 2 immunosuppressed individuals
(C) Human herpesvirus 8 (Kaposi sarcoma (D) Mild or inapparent infection
herpesvirus)
(D) Cytomegalovirus 19. Herpes simplex virus type 1 (HSV-1) is distinct
(E) Parvovirus from herpes simplex virus type 2 (HSV-2) in several
different ways. Which one of the following is the
14. Which of the following groups are at increased least accurate statement?
risk for herpes zoster? (A) HSV-1 causes lesions above the umbilicus more
(A) Persons at advanced age frequently than HSV-2 does.
(B) Patients with atopic dermatitis (B) Infection by HSV-1 is not associated with any
(C) Pregnant women tumors in humans.
(D) Persons who have been vaccinated with (C) Antiserum to HSV-1 neutralizes HSV-1 much
varicella vaccine more effectively than HSV-2.
(E) Infants with congenital infections (D) Whereas HSV-1 causes frequent recurrences,
HSV-2 infection rarely recurs.
Jawetz Medical Microbiology 27th ed.

20. Each of the following statements concerning 5. Which of the following best describes the
Epstein-Barr virus is correct except currently licensed smallpox vaccines?
(A) Many infections are mild or inapparent. (A) Live attenuated smallpox virus
(B) The earlier in life primary infection is acquired, (B) Inactivated smallpox virus
the more likely the typical picture of infectious (C) Live vaccinia virus
mononucleosis will be manifest. (D) Inactivated vaccinia virus
(C) Latently infected lymphocytes regularly persist (E) Reassortant vaccine containing both vaccinia
after an acute episode of infection. and smallpox viruses
(D) Infection confers immunity against second
episodes of infectious mononucleosis. 6. Which of the following does not apply to vaccinia
virus replication in cultured cells?
Chapter 34: Poxvirus (A) Viral replication cycle takes place in the
cytoplasm of infected cells.
1. A patient presents to the emergency room with (B) The uncoating step leading to release of the
vesicular lesions on both hands potentially viral genome requires a newly synthesized viral
resembling smallpox. A public health investigation protein.
is begun to rule out smallpox. The patient is an (C) Early transcription of more than 50 viral genes
immigrant working as a shepherd in several states. occurs within viral cores and precedes viral DNA
What is the most likely cause of his skin lesions? replication.
(A) Vaccinia virus (D) Newly formed virus particles mature by budding
(B) Variola virus through the nuclear membrane.
(C) Monkeypox virus
(D) Tanapox virus 7. Which feature of the variola virus makes it an
(E) Orf virus extreme bioterrorism threat?
(A) Wide availability of the virus
2. An emergency services worker is considering (B) Weaponized strains present in several
smallpox vaccination because of the potential for laboratories
bioterrorism. Which one of the following conditions (C) Limited immunity in present population
is not a contraindication for the use of vaccinia (D) Low stockpiles of effective drugs for treatment
(smallpox) vaccine under routine nonemergency (E) Potential emergence from animal reservoir
conditions?
(A) Immunosuppression 8. A patient presents with skin lesions similar in
(B) Severe allergy to a component of the vaccine appearance to
(C) Household contact with a person with eczema molluscum contagiosum. How is diagnosis of this
(D) Pregnancy condition
(E) Previous smallpox vaccination typically made?
(A) Viral culture
3. Which of the following poxviruses infects only (B) Rapid antigen test
humans? (C) PCR for viral DNA
(A) Monkeypox (D) Clinical appearance
(B) Molluscum contagiosum (E) Inoculation of chorioallantoic membrane of chick
(C) Tanapox Embryos
(D) Cowpox
(E) Yaba tumor virus 9. Which of the following does not fulfill the criteria
for exposure to vaccinia?
4. A 7-year-old boy has pox-like lesions on his left (A) Smallpox vaccination
hand and arm. He has a pet rodent imported from (B) Close contact with a recent smallpox vaccine
West Africa. Monkeypox is diagnosed in the boy (C) Intrauterine exposure
and the rodent. Which of the following statements (D) Injection of vaccinia immune globulin
about monkeypox virus is most correct? 10. A researcher wishes to obtain a full-length
(A) Clinical disease resembles smallpox. genome of variola virus for vaccine studies. Which
(B) Human infections are never fatal. of the following is the appropriate source of the viral
(C) Smallpox vaccination is not protective. DNA?
(D) Infections are readily transmitted among family (A) The Centers for Disease Control and
members. Prevention.
(E) Virus particles can be distinguished from (B) A World Health Organization collaborating
smallpox virus by electron microscopy. center.
Jawetz Medical Microbiology 27th ed.
(C) The American Type Culture Collection. 2. Which of the following exposures poses a risk for
(D) A colleague with a variola virus clone. hepatitis infection?
(E) Distribution of a full-length viral genome is (A) A nurse sustains a needlestick while drawing up
prohibited. insulin to administer to an HBV-infected patient with
diabetes.
11. Laboratory scientists who work with vaccinia (B) While cleaning the bathroom, a housekeeper’s
virus–infected cultures or animals are at risk of intact skin has contact with feces.
accidental exposure to the virus. Which of the (C) An operating room technician with chapped and
following procedures by the laboratory worker is of abraded hands notices blood under his gloves after
least benefit in protecting against inadvertent assisting in an operation on a patient with HCV
Infection with vaccinia virus? infection.
(A) Proper use of personal protective equipment (D) A child drinks out of the same cup as her
such as gloves and goggles mother, who has an HAV infection.
(B) Cleaning of laboratory work space before (E) A shopper eats a sandwich prepared by a
experimentation worker with an asymptomatic HBV infection.
(C) Smallpox vaccination 3. An epidemic of jaundice caused by HEV
(D) Safe needle-handling practices occurred in New Delhi. HEV is:
(E) Use of biosafety hoods (A) Found in rodents and pigs.
(B) A major cause of bloodborne hepatitis.
12. Vaccinia virus has all of the following attributes (C) The cause of a disease that resembles hepatitis
except C.
(A) Can cause severe localized or disseminated (D) Capable of establishing chronic infections.
disease. (E) Associated with an increased risk of liver
(B) Is a live, attenuated smallpox virus. cancer.
(C) Can induce immunity that lasts only a few years.
(D) Has been in use for more than 200 years. 4. HDV (delta agent) is found only in patients who
(E) Gene sequences coding for other viral proteins have either acute or chronic infection with HBV.
can be inserted into its genome. Which of the following is most correct?
13. The eradication of smallpox was facilitated by (A) HDV is a defective mutant of HBV.
several features of the virus. Which one of the (B) HDV depends on HBV surface antigen for virion
following contributed least to eradication? formation.
(A) It has one antigenic type. (C) HDV induces an immune response
(B) Inapparent infection is rare. indistinguishable from that induced by HBV.
(C) Administration of live vaccine reliably induces (D) HDV is related to HCV.
immunity. (E) HDV contains a circular DNA genome.
(D) It multiplies in the cytoplasm of infected cells.
5. A 23-year-old woman is planning a 1-year trip
14. Vaccination with the vaccinia (smallpox) vaccine through Europe, Egypt, and the Indian subcontinent
protects against infections by the following and receives a vaccine for hepatitis A. The current
poxviruses except: hepatitis A vaccine is
(A) Molluscum contagiosum (A) A live attenuated virus vaccine
(B) Variola (B) A recombinant DNA vaccine
(C) Cowpox (C) A formalin-inactivated virus vaccine
(D) Monkeypox (D) An envelope glycoprotein subunit vaccine
(E) A chimeric poliovirus that expresses HAV
Chapter 35: Hepatitis Viruses neutralizing epitopes

1. A 24-year-old woman in New York City is 6. The following statements about HCV infection
admitted to the hospital because of jaundice. On and associated chronic liver disease in the United
workup, she is found to have HCV infection. The States are correct except
major risk factor for HCV infection in the United (A) HCV is responsible for 40% of chronic liver
States is: disease.
(A) Tattoos (B) Chronic infection develops in most (70–90%)
(B) Injecting drug use HCV-infected persons.
(C) Blood transfusion (C) HCV-associated liver disease is the major
(D) Sexual activity cause for liver transplantation.
(E) Working in health care occupations (D) HCV viremia occurs transiently during early
stages of infection.
Jawetz Medical Microbiology 27th ed.
(E) HCV-infected patients are at high risk (5–20%) (E) Was previously immunized with HBV vaccine
for liver cancer. prepared from healthy HBsAg-positive carriers.
7. A middle-aged man complained of acute onset of
fever, nausea, and pain in the right upper 11. The following persons are at increased risk for
abdominal quadrant. There was jaundice, and dark HAV infection and should be routinely vaccinated
urine had been observed several days earlier. A except for which group?
laboratory test was positive for HAV IgM antibody. (A) Persons traveling to or working in countries that
The physician can tell the patient that have high levels of HAV infection
(A) He probably acquired the infection from a recent (B) Men who have sex with men
blood transfusion. (C) Users of illegal drugs (both injecting and
(B) He will probably develop chronic hepatitis. noninjecting)
(C) He will be at high risk of developing (D) Persons who have an occupational risk for
hepatocellular carcinoma. infection
(D) He will be resistant to infection with hepatitis E. (E) Persons who have a clotting factor disorder
(E) He may transmit the infection to family members (F) Susceptible persons who have chronic liver
by person-to-person spread for up to 2 weeks. disease
(G) Teachers in elementary schools
8. Several different viruses can cause hepatitis.
One of the following statements applies to all four 12. There is global variation in the prevalence of
viruses: HAV, HCV, HDV, and HEV. HBV infection. Which of the following geographic
(A) It contains a single-stranded RNA genome. areas has low endemicity (HBsAg prevalence of <
(B) It is transmitted primarily by the parenteral route. 2%)?
(C) It is transmitted primarily by the fecal–oral route. (A) Southeast Asia
(D) It is associated with fulminant hepatitis. (B) The Pacific Islands
(E) It undergoes sequence variation during chronic (C) Eastern Europe
infection. (D) Australia
(E) Sub-Saharan Africa
9. A 30-year-old student goes to the emergency
room because of fever and anorexia for the past 3 13. Which of the following persons are not
days. She appears jaundiced. Her liver is enlarged recommended to receive hepatitis B vaccine
and tender. A laboratory test shows elevated because they have a risk factor for HBV infection?
aminotransferases. She reports a history of having (A) Sexually active persons who are not in long-
received hepatitis B vaccine 2 years ago but has term, mutually
not had hepatitis A vaccine. The results of her monogamous relationships
hepatitis serologic tests are as follows: (B) Injection drug users
HAV IgM-negative, HAV IgG-positive, (C) Pregnant women
HBsAgnegative, HBsAb-positive, HBcAb-negative, (D) Persons who live in a household with a person
HCV Ab-positive. The most accurate conclusion is who is HBsAg positive
that she probably (E) Persons seeking treatment for a sexually
(A) Has hepatitis A now, has not been infected with transmitted disease
HBV, and had hepatitis C in the past.
(B) Has hepatitis A now and has been infected with 14. Which of the following statements regarding
both HBV and HCV in the past. HBIG is not true?
(C) Has been infected with HAV and HCV in the (A) HBIG provides temporary protection when
past and has hepatitis B now. administered in standard doses.
(D) Has been infected with HAV in the past, has not (B) HBIG typically is used instead of hepatitis B
been infected with HBV, and has hepatitis C now. vaccine for postexposure immunoprophylaxis to
(E) Has been infected with HAV and HCV in the prevent HBV infection.
past, has not been infected with HBV, and has (C) No evidence exists that HBV, HCV, or HIV have
hepatitis E now. ever been transmitted by HBIG in the United States.
(D) MHBIG is not used as protection against HCV
10. A 36-year-old nurse is found to be both HBsAg infection.
positive and HBeAg positive. The nurse most likely
(A) Has acute hepatitis and is infectious. 15. Each of the following statements concerning
(B) Has both HBV and HEV infections. HAV is correct except
(C) Has a chronic HBV infection. (A) The hepatitis A vaccine contains inactivated
(D) Has cleared a past HBV infection. HAV as the immunogen.
Jawetz Medical Microbiology 27th ed.
(B) HAV commonly causes asymptomatic infection (C) Transcription inhibitors and interferons
in children. (D) Protease inhibitors, polymerase inhibitors, and
(C) The diagnosis of hepatitis A is usually made by interferons
isolating HAV in cell culture. (E) Reverse transcriptase inhibitors and interferons
(D) γ-Globulin is used to prevent hepatitis A in
exposed persons. Chapter 36: Picornavirus (enterovirus and
Rhinovirus Groups)
16. Which of the following serologic patterns is
suggestive of a patient with chronic hepatitis B with 1. Which of the following statements about
a pre-core mutation? rhinoviruses is correct?
(A) HBsAg positive, HBsAb negative, anti-HBc (A) There are three antigenic types.
positive, HBeAg positive, HBV DNA positive (B) Amantadine protects against infection.
(B) HBsAg positive, HBsAb negative, anti-HBc (C) They do not survive on environmental surfaces.
positive, HBeAg positive, HBV DNA positive (D) They are the most frequent causative agent of
(C) HBsAg positive, HBsAb positive, anti-HBc the common cold.
positive, HBeAg negative, HBV DNA positive (E) They share physicochemical similarities with
(D) HBsAg negative, HBsAb positive, anti-HBc coronaviruses.
positive, HBeAg negative, HBV DNA negative
2. A 26-year-old man develops myopericarditis with
17. A 35-year-old man addicted to intravenous mild congestive heart failure that increases over
drugs has been a carrier of HBsAg for 10 years. He several weeks. Coxsackievirus B5 infection is
suddenly develops acute fulminant hepatitis and diagnosed. Which of the following clinical
dies within 10 days. Which of the following syndromes is not associated with coxsackievirus
laboratory tests would contribute most to diagnosis? infections?
(A) Anti-HBs antibody (A) Herpangina
(B) HBeAg (B) Myocarditis or pericarditis
(C) Anti-HBc antibody (C) Aseptic meningitis
(D) Anti-delta virus antibody (D) Acute hemorrhagic conjunctivitis
(E) Progressive postpolio muscle atrophy
18. Each of the following statements concerning
HCV and HDV is correct except 3. A 3-month-old child develops fever, restlessness,
(A) HCV is an RNA virus. and unusual crying. These are followed by apparent
(B) HDV is transmitted primarily by the fecal–oral lethargy. Physical examination shows a normal-
route. appearing infant who is minimally
(C) HDV is a defective virus that can replicate only responsive to stimuli. A lumbar puncture yields
in a cell that is also infected with HBV. cerebrospinal fluid with 200 white blood cells per
(D) People infected with HCV commonly become microliter, predominantly lymphocytes. Acute
chronic carriers of HCV and are predisposed to aseptic meningitis is diagnosed, probably
hepatocellular carcinoma. caused by an enterovirus. Enteroviruses are
characterized by
19. Which of the following statements about HBV is (A) Latency in sensory ganglia and reactivation
false? primarily in immunocompromised patients
(A) Replication involves reverse transcriptase. (B) Transmission primarily by the fecal–oral route
(B) Infected persons may have large numbers of (C) The presence of a DNA polymerase enzyme
noninfectious viral particles circulating in their (D) The entry of cells following binding to the
bloodstream. intercellular adhesion molecule-1 (ICAM-1) receptor
(C) Infection can result in cirrhosis. (E) Undergoing antigenic shift and drift
(D) Asymptomatic infections can last for years.
(E) In the United States, the incidence of infection 4. Picornavirus vaccines have been used for
has been steadily increasing over the past few several decades in the prevention of human
years. disease. Which of the following statements
is correct?
20. Treatment of hepatitis C can involve drugs of (A) The live, attenuated poliovirus vaccine produces
which of the following classes? gastrointestinal tract resistance.
(A) Protease inhibitors, polymerase inhibitors, and (B) There is an effective killed vaccine against the
interleukins three major types of rhinoviruses.
(B) Non-nucleoside polymerase inhibitors, protease
inhibitors, and interferons
Jawetz Medical Microbiology 27th ed.
(C) The live, attenuated poliovirus vaccine induces 9. Outbreaks of hand-foot-and-mouth disease,
protective immunity against the closely related characterized by oral ulcerations and vesicular
coxsackie B viruses. rashes, occur and may result in infant deaths. The
(D) None of the available echovirus vaccines should disease is caused by
be given to immunocompromised patients. (A) Foot-and-mouth disease virus
(E) Only the live attenuated poliovirus vaccine is (B) Chickenpox virus
currently recommended for use in the United (C) Nonpolio enteroviruses
States. (D) Rhinoviruses
(E) Rubella virus
5. One month after school has been let out for the
summer, a 16-year-old girl develops fever, myalgia, 10. Epidemiologic studies indicate that a core group
and headache. An outbreak of an illness with of enteroviruses is consistently circulating in the
similar symptoms caused by an echovirus is known United States. Which of the following statements is
to be occurring in the community. The primary most accurate?
anatomic site of echovirus multiplication in the (A) Members of the core group all display an
human host is epidemic pattern of outbreaks of disease.
(A) The muscular system (B) The group includes about half of the known
(B) The central nervous system enteroviruses.
(C) The alimentary tract (C) Disease occurs predominantly in adolescents
(D) The blood and lymph system and adults.
(E) The respiratory system (D) Members of the group are all classified as
coxsackie A and B viruses.
6. Which of the following properties of enteroviruses (E) This core group determines the majority of
is not shared by rhinoviruses? enterovirus disease.
(A) Single-stranded RNA genome
(B) Production by cleavage of viral proteins from a 11. Each of the following statements concerning
polyprotein precursor rhinoviruses is correct except
(C) Resistance to lipid solvents (A) Rhinoviruses are one of the most frequent
(D) Stability at acid pH (pH 3.0) causes of the common cold.
(E) Icosahedral symmetry (B) Rhinoviruses grow better at 33°C than at 37°C;
thus, they tend to cause disease in the upper
7. A person with asthma has an acute exacerbation respiratory tract rather than the lower respiratory
with increased lower respiratory illness. A virus is tract.
recovered. The isolate is most likely to be which of (C) Rhinoviruses are members of the picornavirus
the following virus types? family and resemble poliovirus in their structure and
(A) Parainfluenza virus replication.
(B) Parechovirus (D) The immunity provided by the rhinovirus vaccine
(C) Rhinovirus is excellent because there is only one serotype.
(D) Respiratory syncytial virus
(E) Echovirus 12. Live, attenuated oral polio vaccine (OPV) and
inactivated polio vaccine (IPV) are both available. In
8. The use of live oral polio vaccine has been which one of the following situations is the use of
replaced by inactivated polio vaccine in many OPV preferred?
countries. Which of the following is the primary (A) Routine infant vaccination
reason? (B) Mass immunization programs in areas of high
(A) It is more cost effective to use the inactivated poliomyelitis endemicity
vaccine. (C) Adult immunization
(B) There is a greater risk of vaccine-induced (D) Patients who are receiving immunosuppressive
disease than wild virus–induced disease in areas therapy
where poliovirus has been eradicated. (E) Family contacts of immunocompromised
(C) Only a single dose of inactivated vaccine is patients
necessary compared with multiple doses of the oral
vaccine. 13. Which of the following statements about
(D) Circulating poliovirus strains have changed and enteroviral meningitis is true?
the live vaccine is no longer effective in many (A) Vaccines are generally available to protect
countries. against the disease.
(B) The main symptom is muscle paralysis.
(C) Transmission is usually by the fecal–oral route.
Jawetz Medical Microbiology 27th ed.
(D) The causative agents do not survive well in the 4. This viral gastroenteritis agent has a segmented,
environment. doublestranded RNA genome and a double-shelled
(E) Recovery is rarely complete. capsid. It is a member of which virus family?
(A) Adenoviridae
14. The major barrier to the control of rhinovirus (B) Astroviridae
upper respiratory infections by immunization is (C) Caliciviridae
(A) The poor local and systemic immune response (D) Reoviridae
to these viruses (E) Coronaviridae
(B) The large number of rhinovirus serotypes
(C) The side effects of the vaccine 5. Rotavirus and Norwalk virus are distinctly
(D) The inability to grow the viruses in cell culture different viruses. However, they share which one of
the following characteristics?
15. Each of the following clinical syndromes is (A) Fecal–oral mode of transmission.
associated with infection by picornaviruses except (B) They mainly cause disease in infants and young
(A) Myocarditis or pericarditis children.
(B) Hepatitis (C) They induce generally mild disease in young
(C) Mononucleosis children.
(D) Meningitis (D) Infection patterns show no seasonal variation.
(E) A double-stranded RNA genome
Chapter 37: Reoviruses, Rotaviruses and
Caliciviruses 6. Because rotavirus infections can be serious, a
vaccine would be beneficial. Which of the following
1. A 36-year-old man enjoyed a meal of raw is most correct regarding a rotavirus vaccine?
oysters. Twenty-four hours later, he became ill, with (A) A killed human rotavirus group A vaccine is
a sudden onset of vomiting, diarrhea, and licensed for use in the United States.
headache. The most likely cause of his (B) Live attenuated vaccines are licensed for use in
gastroenteritis is the United States.
(A) Astrovirus (C) Vaccine development is complicated by rapid
(B) Hepatitis A virus antigenic variation by the virus.
(C) Norwalk virus (D) Available antiviral drugs make a vaccine
(D) Rotavirus, group A unnecessary.
(E) Echovirus (E) Vaccine development is complicated because
the virus cannot be grown in cell culture.
2. This virus is the most important cause of
gastroenteritis in infants and young children. It 7. Rotaviruses and astroviruses share a number of
causes infections that are often severe and may be characteristics. Which of the following is not
life threatening, especially in infants. shared?
(A) Echovirus (A) Multiple serotypes exist
(B) Norwalk virus (B) Can cause gastroenteritis in infants and children
(C) Rotavirus, group A (C) Can cause gastroenteritis in elderly
(D) Orbivirus institutionalized patients
(E) Parvovirus (D) Live vaccine available
(E) Fecal–oral route of transmission
3. An outbreak of epidemic gastroenteritis occurred
at a wooded summer camp 24 hours after a party 8. A 20-year-old man was on a 3-week tour of Italy
for visiting families. Some of the visiting parents with other college students. One day he abruptly
also became ill. Samples taken 2 weeks later from became ill with nausea and vomiting followed 5
the well that was the source of drinking water at the hours later by abdominal cramps and watery
camp were negative for fecal coliforms. The most diarrhea. No fever was noted. Which of these
likely source of the outbreak was viruses is the most likely cause of the man’s illness?
(A) Mosquitoes or ticks, present in high numbers in (A) Calicivirus
the area (B) Rotavirus
(B) Contaminated food served at the party (D) Adenovirus
(C) A nearby stream used for fishing (E) Astrovirus
(D) A visiting parent who was developing
pneumonia 9. Which statement about rotaviral gastroenteritis is
(E) The swimming pool false?
Jawetz Medical Microbiology 27th ed.
(A) The name of the causative agent was 1999. By 2002 the virus had spread throughout the
suggested by its appearance. continental United States. This arbovirus, a member
(B) Most of the estimated 600,000 deaths occurring of the Japanese B encephalitis antigenic complex,
worldwide from this disease are from dehydration. is which of the following?
(C) Most cases of the disease occur in infants and (A) Japanese B encephalitis virus
children. (B) Tick-borne encephalitis virus
(D) The causative agent infects mainly the stomach. (C) West Nile virus
(E) The disease is transmitted by the fecal–oral (D) Dengue virus
route. (E) Rift Valley fever virus
10. Norwalk virus illness might be prevented by any
of the following except 3. Which of the following descriptions of or
(A) Avoidance of raw fruits statements about Lassa fever is correct?
(B) Live, reassortant vaccine (A) It is found in eastern Africa.
(C) Careful handwashing (B) Human-to-human transmission does not occur.
(D) Avoidance of local drinking water (C) It seldom causes death or complications.
(E) Avoidance of raw oysters (D) It occurs from contact with the house rat
Mastomys natalensis.
11. Which of the following statements about (E) There is no drug that is effective in treating
noroviruses is false? Lassa fever.
(A) They cause almost half the cases of viral
gastroenteritis in the United States. 4. Arboviruses are transmitted by bloodsucking
(B) They can be responsible for epidemics of arthropods from one vertebrate host to another.
gastroenteritis. Arboviruses are found in the following virus families
(C) They generally produce an illness lasting 1–2 except which of the following?
weeks. (A) Togaviridae
(D) Similar viruses are widespread among marine (B) Flaviviridae
animals. (C) Bunyaviridae
(E) They typically cause disease in children and (D) Reoviridae
adults rather than infants. (E) Arenaviridae

12. Each of the following statements regarding 5. A 27-year-old man develops fever, chills,
rotaviruses is correct except headache, and backache. Four days later he
(A) The rotavirus vaccine contains recombinant develops a high fever and jaundice. Yellow fever is
RNA polymerase as the immunogen. diagnosed. Which of the following statements
(B) Rotaviruses are a leading cause of diarrhea in concerning yellow fever is correct?
young children. (A) The virus is transmitted by culicine mosquitoes
(C) Rotaviruses are transmitted primarily by the in the urban form of disease.
fecal–oral route. (B) Monkeys in the jungle are a major reservoir of
(D) Rotaviruses belong to the reovirus family, which yellow fever virus.
has a double-stranded segmented RNA genome. (C) Yellow fever often has long-term complications.
(D) All infections lead to apparent disease.
Chapter 38: Arthropod-Borne and Rodent-Borne (E) Ribavirin is specific therapy.
Viral Diseases
6. Regarding the patient in Question 5, yellow fever
1. A 74-year-old man develops fever, malaise, and occurs in which region or regions of the world?
a sore throat followed shortly thereafter by nausea, (A) Asia
vomiting, and then stupor. Eastern equine (B) Africa and South America
encephalitis is diagnosed. Control of this disease in (D) Africa and Middle East
humans could be accomplished by eradication of (E) Throughout the world
which of the following?
(A) Horses 7. African hemorrhagic fevers, Marburg and Ebola,
(B) Birds are severe diseases often ending in death. Which of
(C) Sandflies the following is most accurate about Ebola virus?
(D) Mosquitoes (A) It is spread by contact with blood or other body
(E) Ticks fluids.
(B) It is transmitted by mosquitoes.
2. An arbovirus common in the Middle East, Africa, (C) It is a flavivirus.
and Southwest Asia first appeared in New York in
Jawetz Medical Microbiology 27th ed.
(D) It causes infections but no disease in nonhuman (A) It is the most important mosquito-borne viral
primates. disease affecting humans.
(E) It is antigenically related to Lassa fever virus. (B) It is distributed worldwide in tropical regions.
(C) It can cause a severe hemorrhagic fever.
8. Which of the following groups can be vaccinated (D) There is a single antigenic type.
routinely with yellow fever vaccine without special (E) One form of disease is characterized by
safety considerations? increased vascular permeability.
(A) Children younger than 9 months old 12. Which of the following diseases occurring in the
(B) Pregnant women United States lacks a known insect vector?
(C) Persons with compromised immune systems (A) Hantavirus pulmonary syndrome
(D) All of the above (B) West Nile fever
(E) None of the above (C) La Crosse encephalitis
(D) Colorado tick fever
9. Hantaviruses, which are emerging pathogens in (E) St. Louis encephalitis
the United States, can be described by which of the
following? 13. Each of the following statements concerning
(A) They are arenaviruses. arboviruses is correct except
(B) They are readily transmitted human to human. (A) The pathogenesis of dengue hemorrhagic shock
(C) They cause influenza-like symptoms followed syndrome is associated with the heterotypic
rapidly by acute respiratory failure. anamnestic response.
(D) They are acquired by inhalation of aerosols of (B) Wild birds are the reservoir for encephalitis
deer urine. viruses but not for yellow fever virus.
(E) They show a high frequency of antigenic (C) Ticks are the main mode of transmission for
variation. both encephalitis viruses and yellow fever virus.
(D) There is a live, attenuated vaccine that
10. A microbiologist was performing a necropsy in a effectively prevents yellow fever.
laminar flow biosafety cabinet on a blue-jay
submitted as part of a state’s arbovirus surveillance 14. Which of the following statements about yellow
program. He lacerated his thumb while using a fever is false?
scalpel to remove the bird’s brain. Four days later, (A) There is no animal reservoir.
he developed a headache, myalgia, and malaise (B) The name “yellow” comes from the fact that
followed by chills, sweats, and lymph node swelling. many victims have jaundice.
Two days later, a rash began on his face and (C) Certain mosquitoes are biological hosts for the
spread to the trunk, arms, and legs, persisting causative agent.
for about 3 days. He sought medical care and (D) Outbreaks of the disease could occur in the
reported a history of dengue fever and vaccinations United States because a suitable vector is present.
with yellow fever and Japanese B encephalitis (E) An attenuated vaccine is widely used to prevent
vaccines. A serum sample taken the day of the the disease.
injury contained anti-flavivirus immunoglobulin G
(IgG) antibody by enzyme-linked immunosorbent 15. Which of the following statements about
assay. A serum sample collected 13 days after the hantaviruses in the United States is correct?
onset of illness showed an increased titer of (A) They are limited in range to southwestern
antiflavivirus IgG antibody and the presence of states.
West Nile virus IgM antibody. The physician could (B) They are carried only by deer mice.
conclude that the most likely cause of the (C) They infect human beings with a fatality rate
microbiologist’s illness was which that can be above 30%.
virus? (D) They were first identified in the early 1970s.
(A) Dengue virus (E) They are contracted mainly in bat caves.
(B) Yellow fever virus
(C) West Nile virus Chapter 39: Orthomyxoviruses (Influenza virus)
(D) St. Louis encephalitis
(E) Not identifiable until neutralizing antibody titers 1. Which of the following statements regarding the
from paired sera could be assessed against a panel prevention and treatment of influenza is correct?
of arboviruses (A) Booster doses of vaccine are not
recommended.
11. Which of the following statements about dengue (B) Drugs that inhibit neuraminidase are active only
virus is not true? against influenza A.
Jawetz Medical Microbiology 27th ed.
(C) As with some other live vaccines, the attenuated (D) Cytomegalovirus
influenza vaccine should not be given to pregnant (E) Listeria
women.
(D) The influenza vaccine contains several '. Which of the following statements concerning
serotypes of virus. antigenic drift in influenza viruses is correct?
(E) The virus strains in the influenza vaccine do not (A) It results in major antigenic changes.
vary from year to year. (B) It is exhibited only by influenza A viruses.
(C) It is caused by frameshift mutations in viral
2. Which of the following statements about the genes.
neuraminidase of influenza virus is not correct? (D) It results in new subtypes over time.
(A) Is embedded in the outer surface of the viral (E) It affects predominantly the matrix protein.
envelope
(B) Forms a spike structure composed of four 8. A 32-year-old male physician developed a
identical monomers, each with enzyme activity “flulike” syndrome with fever, sore throat, headache,
(C) Facilitates release of virus particles from and myalgia. To provide laboratory
infected cells confirmation of influenza, a culture for the virus was
(D) Lowers the viscosity of the mucous film in the ordered. Which of the following would be the best
respiratory tract specimen for isolating the virus responsible for this
(E) Is antigenically similar among all mammalian infection?
influenza viruses (A) Stool
(B) Nasopharyngeal swab
3. Which of the following statements reflects the (C) Vesicle fluid
pathogenesis of influenza? (D) Blood
(A) The virus enters the host in airborne droplets. (E) Saliva
(B) Viremia is common.
(C) The virus frequently establishes persistent 9. Which of the following statements about isolation
infections in the lung. of influenza viruses is correct?
(D) Pneumonia is not associated with secondary (A) Diagnosis of an influenza virus infection can
bacterial infections. only be made by isolating the virus.
(E) Viral infection does not kill cells in the (B) Isolation of influenza virus is done using
respiratory tract. newborn mice.
(C) Isolation of virus can help determine the
4. Which of the following symptoms is not typical of epidemiology of the disease.
influenza? (D) Primary influenza virus isolates grow readily in
(A) Fever cell culture.
(B) Muscular aches
(C) Malaise 10. The principal reservoir for the antigenic shift
(D) Dry cough variants of influenza virus appears to be which of
(E) Rash the following?
(A) Chronic human carriers of the virus
5. The type-specific antigen (A, B, or C) of influenza (B) Sewage
viruses is found on which viral constituent? (C) Pigs, horses, and fowl
(A) Hemagglutinin (D) Mosquitoes
(B) Neuraminidase (E) Rodents
(C) Nucleocapsid
(D) Polymerase complex 11. Highly pathogenic H5N1 avian influenza (HPAI)
(E) Major nonstructural protein can infect humans with a high mortality rate, but it
(F) Lipid in the viral envelope has not yet resulted in a pandemic. The following
are characteristics of HPAI, except for one. Which
6. A 70-year-old nursing home patient refused the one is not?
influenza vaccine and subsequently developed (A) Efficient human-to-human transmission
influenza. She died of acute pneumonia 1 week (B) Presence of avian influenza genes
after contracting the flu. The most common cause (C) Efficient infection of domestic poultry
of acute postinfluenza pneumonia is which of the (D) Contains segmented RNA genome
following?
(A) Legionella 12. Which of the following statements about
(B) Staphylococcus aureus diagnostic testing for influenza is true?
(C) Measles
Jawetz Medical Microbiology 27th ed.
(A) Clinical symptoms reliably distinguish influenza organ most commonly exhibiting signs of mumps is
from other respiratory illnesses. the
(B) Viral culture is the “gold standard” diagnostic (A) Lungs
test because it is the most rapid assay. (B) Ovary
(C) Patient antibody responses are highly specific (C) Parotid glands
for the strain of infecting influenza virus. (D) Skin
(D) Reverse transcription polymerase chain reaction (E) Testes
is preferred for its speed, sensitivity, and specificity.
2. The paramyxoviruses include the most important
13. The mechanism of “antigenic drift” in influenza causes of respiratory infections in infants and young
viruses includes all but one of the following children. Which of the following is not characteristic
(A) Can involve either hemagglutinin or of paramyxoviruses?
neuraminidase antigens (A) Genome is negative-sense RNA.
(B) Mutations caused by viral RNA polymerase (B) Envelope contains a glycoprotein with fusion
(C) Can predominate under selective host activity.
population immune pressures (C) Paramyxoviruses do not undergo genetic
(D) Reassortment between human and animal or reassortment.
avian reservoirs (D) Replication cycle occurs in cytoplasm of
(E) Can involve genes encoding structural or susceptible cells.
nonstructural proteins (E) Genome is segmented.

14. Each of the following statements concerning the 3. A 2-month-old infant developed a respiratory
prevention and treatment of influenza is correct illness that the pediatrician diagnosed as
except bronchiolitis. The most likely cause
(A) The inactivated influenza vaccine contains of the disease is
H1N1 virus but the live, attenuated influenza (A) Parainfluenza virus type 4
vaccine contains H3N2 virus. (B) Respiratory syncytial virus
(B) The vaccine is recommended to be given each (C) Influenza virus
year because the antigenicity of the virus drifts. (D) Metapneumovirus
(C) Oseltamivir is effective against both influenza A (E) Measles virus
and influenza B viruses.
(D) The main antigen in the vaccine that induces 4. Several paramyxoviruses can cause pneumonia
protective antibody is the hemagglutinin. in infants or children. For which of the following
paramyxoviruses is there an effective vaccine
15. Each of the following statements concerning the available that would prevent pneumonia?
antigenicity of influenza A virus is correct except (A) Parainfluenza virus type 1
(A) Antigenic shifts, which represent major changes (B) Measles virus
in antigenicity, occur infrequently and are caused by (C) Respiratory syncytial virus
the reassortment of segments of the viral genome. (D) Mumps virus
(B) Antigenic shifts affect both the hemagglutinin (E) Metapneumovirus
and the neuraminidase.
(C) The worldwide epidemics caused by influenza A 5. A 27-year-old woman who is 2 months’ pregnant
virus are caused by antigenic shifts. develops fever, malaise, and arthralgia. A fine
(D) The protein involved in antigenic drift is primarily maculopapular rash appears on her face, trunk, and
the internal ribonucleoprotein. extremities. Rubella is diagnosed, and there is
concern that the fetus will be infected, resulting in
16. Which of the following infectious agents is most the congenital rubella syndrome. Which of the
likely to cause a pandemic? following statements about this syndrome is
(A) Influenza A virus correct?
(B) Streptococcus pyogenes (A) The disease can be prevented by vaccination of
(C) Influenza B virus school-age children with measles vaccine.
(D) Respiratory syncytial virus (B) Congenital abnormalities occur when a
(E) Influenza C virus nonimmune pregnant woman is infected at any time
during pregnancy.
Chapter 40: Paramyxoviruses and Rubella virus (C) Deafness is a common defect associated with
congenital rubella syndrome.
1. A 4-year-old boy develops an acute febrile (D) Only rare strains of rubella virus are teratogenic.
illness. His pediatrician diagnoses mumps. The (E) None of the above
Jawetz Medical Microbiology 27th ed.
10. A 3-year-old girl develops an acute respiratory
6. A 5-year-old child develops a low-grade fever, virus infection that requires hospitalization. Ribavirin
coryza, conjunctivitis, and Koplik spots. The therapy is considered. Ribavirin is approved for
physician can conclude that treatment of which of the following situations?
(A) The child has probably not been successfully (A) Lower respiratory tract disease caused by
vaccinated with the MMR vaccine. respiratory syncytial virus in infants
(B) The child’s pregnant mother is at risk of (B) Congenital rubella syndrome
becoming infected and her unborn child developing (C) Aseptic meningitis caused by mumps infection
congenital abnormalities, including mental (D) Pneumonia caused by measles virus in adults
retardation. (E) Encephalitis related to Nipah virus
(C) A rash will soon develop on the child’s face and (F) All of the above
will last only 2–3 days.
(D) Treatment of the child with the antiviral drug 11. Reverse transcription polymerase chain
ribavirin should be initiated immediately to minimize reaction (RT-PCR) assays are useful in diagnosis of
the chance of development of acute encephalitis. paramyxovirus infections. Which of the following
statements about RT-PCR is not accurate?
7. Parainfluenza viruses are ubiquitous and cause (A) More sensitive assay than virus isolation
respiratory illnesses in people of all ages. However, (B) Can identify virus strains
reinfections with parainfluenza viruses are common (C) More rapid assay than antigen detection
because (D) Can provide data about genetic variation for
(A) Many antigenic types of parainfluenza viruses molecular epidemiology studies
exist, and exposure to new strains results in new (E) More specific assay for parainfluenza viruses
infections. than serology
(B) Infections in the respiratory tract do not elicit a
systemic immune response. 12. Each of the following statements concerning
(C) Limited virus replication occurs, which fails to measles vaccine is correct except
stimulate antibody production. (A) The vaccine contains live, attenuated virus.
(D) Secretory immunoglobulin A antibody in the (B) The vaccine should not be given at the same
nose is short lived, disappearing a few months after time as the mumps vaccine because the immune
infection. system cannot respond to two viral antigens given
8. A 20-month-old boy had an illness characterized simultaneously.
by fever, irritability, conjunctivitis, and a brick-red (C) Virus in the vaccine contains only one serotype.
rash initially on the face but spreading downward (D) The vaccine should not be given before 15
and outward. At age 9 years, the boy had a gradual months of age because maternal antibodies can
onset of severe, generalized neurologic prevent an immune response.
deterioration. Subacute sclerosing panencephalitis
(SSPE) was diagnosed. Which of the following 13. Each of the following statements concerning
statements about SSPE is correct? rubella is correct except
(A) Defective varicella-zoster virus is present in (A) Congenital abnormalities occur primarily when a
brain cells. pregnant woman is infected during the first
(B) High titers of measles antibody are found in trimester.
cerebrospinal fluid. (B) Women who say that they have never had
(C) The incidence of the disease is rising since the rubella can, nevertheless, have neutralizing
introduction of MMR vaccine. antibody in their serum.
(D) Rapidly progressive deterioration of brain (C) In a 6-year-old child, rubella is a mild, self-
function occurs. limited disease with few complications.
(E) The disease is a rare, late complication of (D) Acyclovir is effective in the treatment of
rubella infection. congenital rubella syndrome.

9. Which of the following paramyxoviruses has an 14. Each of the following statements concerning
HN surface glycoprotein lacking hemagglutinin rubella vaccine is correct except
activity? (A) The vaccine prevents reinfection, thereby
(A) Measles virus limiting the spread of virulent virus.
(B) Mumps virus (B) The immunogen in the vaccine is killed rubella
(C) Parainfluenza virus type 1 virus.
(D) Respiratory syncytial virus (C) The vaccine induces antibodies that prevent
(E) Rubella virus dissemination of the virus by neutralizing it during
the viremic stage.
Jawetz Medical Microbiology 27th ed.
(D) The incidence of both childhood rubella and
congenital rubella syndrome has decreased Chapter 41: Coronaviruses
significantly since the advent of the vaccine.
1. A 63-year-old woman develops fever, headache,
15. Each of the following statements concerning malaise, myalgia, and cough. It is early in the winter
mumps is correct except respiratory virus season, and the patient’s physician
(A) Mumps virus is a paramyxovirus and hence has does not know what viruses are present in the
a single stranded RNA genome. community. Which of the following viruses is not a
(B) Meningitis is a recognized complication of cause of acute respiratory disease?
mumps. (A) Influenza virus
(C) Mumps orchitis in children before puberty often (B) Adenovirus
causes sterility. (C) Respiratory syncytial virus
(D) During mumps, the virus spreads through the (D) Coronavirus
bloodstream (viremia) to various internal organs. (E) Rotavirus

16. Each of the following statements concerning 2. Based on sequence analysis and serologic
subacute sclerosing panencephalitis is correct assays, the most likely origin of the SARS
except coronavirus is which of the following?
(A) Immunosuppression is a frequent predisposing (A) Recombination between a human and an
factor. animal coronavirus that created a new virus
(B) Aggregates of helical nucleocapsids are found (B) Jump of an animal coronavirus into humans
in infected cells. (C) Mutation of a human coronavirus that resulted
(C) High titers of measles antibody are found in in increased virulence
cerebrospinal fluid. (D) Acquisition of human cellular genes by a human
(D) Slowly progressive deterioration of brain coronavirus via recombination that allowed viral
function occurs. evasion of the host immune response
3. The coronavirus SARS epidemic of 2002–2003
17. Which one of the following is the best evidence resulted in many cases and deaths. What is the
on which to base a decisive diagnosis of acute primary route of transmission of human
mumps disease? coronaviruses?
(A) A positive skin test result (A) Fecal–oral
(B) A fourfold rise in antibody titer to mumps antigen (B) Respiratory
(C) A history of exposure to a child with mumps (C) Blood
(D) Orchitis in young adult man (D) Perinatal mother-to-infant
18. Which one of the following statements (E) Sexual activity
concerning mumps is correct?
(A) Although the salivary glands are the most 4. Coronavirus infections in humans usually cause
obvious sites of infection, the testes, ovaries, and a common cold syndrome. However, a recent
pancreas can be involved as well. outbreak of SARS was characterized by pneumonia
(B) Because there is no vaccine against mumps, and progressive respiratory failure. The prevention
passive immunization is the only means of or treatment of these diseases can be
preventing the disease. accomplished by
(C) The diagnosis of mumps is made on clinical (A) A subunit vaccine
grounds because the virus cannot be grown in cell (B) A cold-adapted live-attenuated vaccine
culture and serologic tests are inaccurate. (C) The antiviral drug amantadine
(D) Second episodes of mumps can occur because (D) Infection control measures, including isolation
there are two serotypes of the virus, and protection and wearing of protective gear
is type specific. (E) The antiviral drug acyclovir

19. Which of the following statements is more likely 5. An epidemic of acute respiratory virus infections
to be true of measles (rubeola) than German occurred among the elderly residents of a nursing
measles (rubella)? home. Influenza viruses and coronaviruses, which
(A) Koplik spots are present. can cause serious respiratory disease in the elderly,
(B) It causes birth defects. are suspected. Which of the following
(C) It causes only a mild illness. characteristics is shared by these viruses?
(D) Human beings are the only natural host. (A) Segmented genome
(E) Attenuated virus vaccine is available for (B) Infectious RNA genome
prevention.
Jawetz Medical Microbiology 27th ed.
(C) High frequency of recombination during characteristic of which of the following central
replication nervous system infections?
(D) Single serotype infects humans (A) Borna disease
(E) Negative-sense genome (B) Rabies
(C) Subacute sclerosing panencephalitis
6. The following are common characteristics of (D) New variant Creutzfeldt-Jakob disease
coronaviruses, except for one. Which is not correct? (E) Postvaccinal encephalitis
(A) Possess cross-reactive antigens with influenza
viruses 4. Which of the following statements about rabies
(B) Contain the largest genomes among RNA vaccines for human use is true?
viruses (A) Contain live, attenuated rabies virus
(C) Can cause gastroenteritis (B) Contain multiple antigenic types of rabies virus
(D) Are distributed worldwide (C) Can treat clinical cases of rabies
(D) Can be used for postexposure prophylaxis
7. SARS coronavirus shares some characteristics, (E) They are associated with Guillain-Barre
but not all, with human coronavirus HCoV-OC43. syndrome
Which of the following statements is true for SARS
coronavirus? 5. A 22-year-old man is a resident of a small town
(A) Causes annual outbreaks during the winter near London. He likes to eat beefsteak. He
(B) Is distributed worldwide develops a severe progressive neurologic disease
(C) Populations at high risk of disease included characterized by psychiatric symptoms, cerebellar
health care workers signs, and dementia. Probable bovine spongiform
(D) Natural hosts are palm civets encephalopathy (BSE) is diagnosed. New variant
Creutzfeldt-Jakob disease in humans and BSE
8. A traveler returning from Mecca presents with appear to be caused by the same agent. Which of
pneumonia, fever, and cough. What is the best test the following statements is true of both diseases?
to diagnose MERS coronavirus? (A) Immunosuppression of the host is a
(A) Coronavirus antigen assay predisposing factor.
(B) Human coronavirus PCR (B) It is an immune-mediated degenerative
(C) MERS-CoV PCR neurologic disorder.
(D) Respiratory viral culture (C) There is a long incubation period (months to
years) from time of exposure to appearance of
9. Risk factors for severe MERS coronavirus symptoms.
infection include which of the following? (D) The agent is recoverable only from the central
(A) Recent camel exposure nervous system of an infected host.
(B) Prior coronavirus infection (E) The interferon response persists throughout the
(C) Seasonal allergies incubation period.
(D) Chronic obstructive pulmonary disease (F) There is a high-titer antibody response toward
PrPSc protein of the agent.
Chapter 42 Rabies, Slow Virus Infections, and
Prion Disease 6. Rabies virus has a wide host range and the
ability to infect all warm-blooded animals, including
1. Rabies virus is rapidly destroyed by humans. Which statement about the epidemiology
(A) Ultraviolet radiation of human rabies is true?
(B) Heating at 56°C for 1 hour (A) Africa accounts for the majority of rabies
(C) Ether treatment fatalities.
(D) Trypsin treatment (B) Dog bites cause most cases of human rabies in
(E) All of the above England.
(C) Domestic animals are the source of most
2. Prions are readily destroyed by human rabies in the United States.
(A) Ionizing radiation (D) Human-to-human rabies transmission places
(B) Formaldehyde medical personnel at serious risk.
(C) Boiling (E) Bat rabies has caused most human rabies
(D) Proteases cases in the United States since the 1990s.
(E) None of the above
7. Infectious scrapie agent can be detected in
3. The presence in neurons of eosinophilic amyloid plaques in infected brains of sheep and
cytoplasmic inclusion bodies, called Negri bodies, is
Jawetz Medical Microbiology 27th ed.
hamsters. The genome of the infectious agent is (C) Immediately euthanize the dog.
characterized by which of the following nucleic (D) Because canine rabies has been eliminated in
acid types? the United States, dog bites are no longer an
(A) Negative-sense, single-stranded RNA indication for postexposure prophylaxis, and no
(B) Small interfering RNA, smallest known further action is needed.
infectious RNA (E) Test the dog for rabies antibody.
(C) DNA copy of RNA genome, integrated in
mitochondrial DNA 11. The slow virus disease that most clearly has
(D) Single-stranded, circular DNA immunosuppression as an important factor in its
(E) No detectable nucleic acid pathogenesis is
(A) Progressive multifocal leukoencephalopathy
8. A 49-year-old man visited a neurologist after 2 (B) Subacute sclerosing panencephalitis
days of increasing right arm pain and paresthesias. (C) Creutzfeldt-Jakob disease
The neurologist diagnosed an atypical neuropathy. (D) Scrapie
The symptoms increased and were accompanied
by hand spasms and sweating on the right side 12. Scrapie and kuru possess all of the following
of the face and trunk. The patient was admitted to characteristics except
the hospital the day after developing dysphagia, (A) A histologic picture of spongiform
hypersalivation, agitation, and generalized muscle encephalopathy
twitching. Vital signs and blood tests were normal, (B) Transmissibility to animals associated with a
but within hours the patient became confused. The long incubation period
consulting neurologist suspected rabies. Rabies (C) Slowly progressive deterioration of brain
immune globulin, vaccine, and acyclovir were function
administered. The patient was placed on (D) Prominent intranuclear inclusions in
mechanical ventilation the following day. Renal oligodendrocytes
failure developed, and the patient died 3 days later.
Rabies test results were positive. The patient’s wife 13. A 5-year-old boy in San Francisco reaches into
reported the patient had suffered no bites by dogs a car to pet another family’s dog and is bitten on the
or wild animals. The most likely explanation finger. Six weeks after the bite, the child develops
for treatment failure is fever, headache, and a seizure. He becomes
(A) The rabies test results were falsely positive and combative and hallucinates. What is the best
the patient did not have rabies. diagnostic test to perform on the patient to rule in
(B) Treatment was initiated after the onset of clinical rabies as a cause of his illness?
symptoms of rabies. (A) Detection of serum antirabies antibody
(C) The vaccine was directed against dog rabies (B) Culture of cerebrospinal fluid for virus
and the patient was infected with bat rabies. (C) Direct fluorescent antibody stain of a biopsy
(D) The rabies immune globulin should not have from the nape of the neck
been administered as it interfered with the vaccine. (D) Brain biopsy
(E) Interferons—and not the treatment regimen (E) Cerebrospinal fluid antirabies antibody
administered—are the treatment of choice once
rabies symptoms develop. 14. Each of the following statements concerning
rabies and rabies virus is correct except
9. Which of the following animals is most commonly (A) The virus has a lipoprotein envelope and single-
reported rabid in the United States? stranded RNA as its genome.
(A) Squirrels (B) The virus has a single antigenic type (serotype).
(B) Raccoons (C) In the United States, dogs are the most
(C) Rabbits common reservoir.
(D) Swine (D) The incubation period is usually long (several
(E) Rats weeks) rather than short (several days).

10. A runner reports an “unprovoked bite” from a 15. A 20-year-old man, who for many years had
neighborhood dog. The dog was captured by local received daily injections of growth hormone
animal control authorities, and it appears healthy. prepared from human pituitary glands, develops
What is the appropriate action? ataxia, slurred speech, and dementia. At autopsy
(A) Confine and observe the dog for 10 days for the brain shows widespread neuronal degeneration,
signs suggestive of rabies. a spongy appearance due to many vacuoles
(B) Begin postexposure prophylaxis of the bitten between the cells, no inflammation, and no
person.
Jawetz Medical Microbiology 27th ed.
evidence of virus particles. The most likely (A) Polyomavirus BK
diagnosis is (B) Human papillomavirus, all types
(A) Herpes encephalitis (C) Human papillomavirus, low-risk types
(B) Creutzfeldt-Jakob disease (D) Hepatitis C virus
(C) Subacute sclerosing panencephalitis (E) Human cytomegalovirus
(D) Progressive multifocal leukoencephalopathy
(E) Rabies 6. Human papillomavirus can cause cancer in
humans and is most commonly associated with
Chapter 43: Human cancer Viruses (A) Rectal polyps
(B) Breast cancer
1. Viruses can cause cancer in animals and (C) Prostate cancer
humans. A principle of viral carcinogenesis is that (D) Anogenital cancers
(A) Retroviruses cause most types of human (E) Mesotheliomas
cancer.
(B) Not all infections with a human cancer virus lead 7. A virus that causes human cancer is also
to tumor formation. associated with a nervous system disorder called
(C) Short latent periods elapse between time of tropical spastic paraparesis. That virus is
virus infection and tumor appearance. (A) Polyomavirus JC
(D) Animal models seldom predict cellular (B) Polyomavirus SV40
mechanisms in human cancer. (C) Herpes simplex virus
(E) Host factors are insignificant in influencing the (D) Human T-lymphotropic virus
development of virus-induced human cancer. (E) Human immunodeficiency virus

2. Cellular oncogenes represent activated genes 8. The polyomaviruses encode oncoproteins called
involved in cancer. A second class of cancer genes T antigens. These viral gene products
is involved in cancer development only when both (A) Are not needed for virus replication
alleles of such a gene are inactivated. The second (B) Interact with cellular tumor suppressor proteins
class of genes is called (C) Function to integrate the viral provirus into the
(A) Proto-oncogenes cellular chromosome
(B) T antigen genes (D) Mutate rapidly to allow the virus to escape
(C) Tumor suppressor genes immune clearance by the host
(D) Transduced genes (E) Are not able to transform cells in culture
(E) Silent genes
9. Cancer viruses are classified in several virus
3. A 38-year-old woman is diagnosed with cervical families. Which of the following virus families
cancer. This cancer is common worldwide and has contains a human cancer virus with an RNA
a sexually transmitted viral etiology. The causative genome?
agent of human cervical cancer is (A) Adenoviridae
(A) Hepatitis C virus (B) Herpesviridae
(B) Hepatitis B virus (C) Hepadnaviridae
(C) Human papillomaviruses, high-risk types (D) Papillomaviridae
(D) Polyomaviruses (E) Flaviviridae
(E) Herpesviruses
10. Laryngeal papillomas in children are generally
4. Retroviruses encode an enzyme called reverse caused by the same viruses that cause benign
transcriptase. The function of the reverse genital condylomas. These viruses are
transcriptase enzyme is (A) Papillomaviruses, types 6 and 11
(A) DNase activity (B) Polyomavirus JC
(B) RNA-dependent DNA polymerase activity (C) Epstein-Barr virus
(C) DNA-dependent RNA polymerase activity (D) Molluscum contagiosum virus
(D) RNA-dependent RNA polymerase activity (E) Papillomaviruses, types 16 and 18
(E) Topoisomerase activity
11. Vaccines against the most common HPV types
5. Two months after a kidney transplant, a 47-year- that cause genital infections were approved in 2006
old man developed nephropathy. Up to 5% of renal and 2007. They are aimed for use in which of the
allograft recipients develop nephropathy. A viral following population(s)?
cause of some of the nephropathy cases has been (A) All adults, both men and women
identified as (B) All female adults
Jawetz Medical Microbiology 27th ed.
(C) Women with precancerous cervical lesions 4. The typical course of an untreated HIV infection
(D) All adolescents and young adults, both boys extends over 10 or more years. There is usually a
and girls long period (clinical latency) between the time of
(E) Adolescent and young adult females primary HIV infection and the development
of AIDS. During this period of clinical latency
12. Which of the following best describes available (A) HIV is not detectable in the plasma
HPV vaccines? (B) CD4 cell counts remain unchanged
(A) Live attenuated virus (C) Virus replicates at a very low rate
(B) Live recombinant virus (D) Virus is present in lymphoid organs
(C) Noninfectious subunit (E) Neutralizing antibodies are not elicited
(D) Toxoid
5. Viral coinfections occur in HIV-1-infected
13. Many of the oncogenic retroviruses carry individuals and may contribute to morbidity and
oncogenes closely related to normal cellular genes, mortality. The most common coinfection in HIV-1-
called proto-oncogenes. Which one of the following positive persons in the United States involves
statements concerning proto-oncogenes is (A) Hepatitis C virus
incorrect? (B) Hepatitis D virus
(A) Several proto-oncogenes have been found in (C) HIV type 2
mutant form in human cancers that lack evidence (D) Human T-lymphotropic virus
for viral etiology. (E) Kaposi sarcoma herpesvirus
(B) Several viral oncogenes and their progenitor
proto-oncogenes encode protein kinases specific 6. What are the most common symptoms of acute
for tyrosine. HIV infection?
(C) Some proto-oncogenes encode cellular growth (A) Rash and sore throat
factors and receptors for growth factors. (B) Fever and malaise
(D) Proto-oncogenes are closely related to (C) Diarrhea
transposons found in bacteria. (D) Jaundice and hepatitis
(E) Neuropsychiatric and behavioral changes
Chapter 44: Aids and Lentivirus
7. A 36-year-old nurse suffered a needlestick with
1. HIV-1 is classified as a member of the Lentivirus blood from an HIV-positive patient. Six months later,
genus in the Retroviridae family. Lentiviruses the nurse’s serum was positive in an EIA test, gave
(A) Contain a DNA genome equivocal results in a repeat EIA test, and was
(B) Cause tumors in mice negative by Western blot. The nurse
(C) Infect cells of the immune system (A) Is probably infected with HIV
(D) Have related sequences endogenous in normal (B) Is in the window between acute infection with
cells HIV and seroconversion
(E) Cause rapidly progressive neurologic disease (C) Is probably not infected with HIV
(D) May be infected with a drug-resistant strain of
2. HIV-1 encodes an envelope glycoprotein, gp120. HIV
This protein (E) May be a long-term nonprogressor
(A) Causes membrane fusion
(B) Binds to the viral coreceptor on the cell surface 8. A 41-year-old HIV-infected male who had refused
(C) Is highly conserved among different isolates antiretroviral therapy is diagnosed with
(D) Fails to elicit neutralizing antibody Pneumocystis jiroveci infection. This patient
(E) Induces chemokine production (A) Probably has a CD4 T-cell count below 200
cells/μL
3. HIV/AIDS has become a worldwide epidemic that (B) Is at elevated risk for lung cancer
continues to expand. The geographic area with the (C) Is not longer a candidate for HAART
largest number of HIV infected people after sub- (D) Probably has declining levels of plasma viremia
Saharan Africa is (E) Is unlikely to develop dementia at this stage
(A) Central and South America and the Caribbean
(B) China 9. A 48-year-old HIV-positive man with a CD4 count
(C) North America of 40 complains of memory loss to his doctor. Four
(D) South/Southeast Asia months later, he becomes paralyzed and dies. An
(E) Eastern Europe and Central Asia autopsy reveals demyelination of many neurons in
the brain, and electron microscopy shows
Jawetz Medical Microbiology 27th ed.
clusters of non-enveloped viral particles in the 15. Each of the following statements concerning
neurons. The most likely cause of the disease is HIV is correct except
(A) Adenovirus type 12 (A) The CD4 protein on the T-cell surface is one of
(B) Coxsackievirus B2 the receptors for the virus.
(C) Parvovirus B19 (B) There is appreciable antigenic diversity in the
(D) Epstein-Barr virus envelope glycoprotein of the virus.
(E) JC virus (C) One of the viral genes codes for a protein that
augments the activity of the viral transcriptional
10. Highly active antiretroviral combination therapy promoter.
for HIV infection usually includes a protease (D) A major problem with testing for antibody to the
inhibitor such as saquinavir. Such a protease virus is its cross-reactivity with human T-
inhibitor lymphotropic virus type 1.
(A) Is effective against HIV-1 but not HIV-2
(B) Seldom gives rise to resistant mutants of HIV
(C) Inhibits a late step in virus replication
(D) Degrades the CD4 receptor on cells
(E) Interferes with virus interaction with coreceptor Section V: Mycology

11. In a person with HIV infection, potentially Chapter 45: Medical Mycology
infectious fluids include all of the following except
(A) Blood 1. Which statement regarding fungi is correct?
(B) Saliva visibly contaminated with blood (A) All fungi are able to grow as yeasts and molds.
(C) Urine not visibly contaminated with blood (B) Although fungi are eukaryotes, they lack
(D) Genital secretions mitochondria.
(E) Amniotic fluid (C) Fungi are photosynthetic.
(D) Fungi have one or more nuclei and
12. Of the more than 1 million persons estimated to chromosomes.
be living with HIV in the United States in 2011, how (E) Few fungi possess cell membranes.
many are thought to be unaware of their infection?
(A) About 5% 2. Which statement regarding fungal growth and
(B) About 10% morphology is correct?
(C) About 20% (A) Pseudohyphae are produced by all yeasts.
(D) About 25% (B) Molds produce hyphae that may or may not be
(E) About 30% partitioned with cross-walls or septa.
(F) About 50% (C) Conidia are produced by sexual reproduction.
(D) Most yeasts reproduce by budding and lack cell
13. Each of the following statements concerning walls.
HIV is correct except (E) Most pathogenic dimorphic molds produce
(A) Screening tests for antibodies and nucleic acid hyphae in the host and yeasts at 30°C.
are useful to prevent transmission of HIV through
transfused blood. 3. Which statement regarding fungal cell walls is
(B) The opportunistic infections seen in AIDS are correct?
primarily the result of a loss of cell-mediated (A) The major components of fungal cell walls are
immunity. proteins such as chitin, glucans, and mannans.
(C) Zidovudine (azidothymidine) inhibits the RNA- (B) The cell wall is not essential for fungal viability
dependent DNA polymerase. or survival.
(D) The presence of circulating antibodies that (C) Ligands associated with the cell walls of certain
neutralize HIV is evidence that an individual is fungi
protected against HIV induced Disease. mediate attachment to host cells.
(D) Fungal cell wall components are the targets for
14. Highly active antiretroviral therapy (HAART) is the major classes of antifungal antibiotics, such as
less than ideal because the polyenes and azoles.
(A) It does not eliminate latent HIV infection (E) Fungal cell wall components rarely stimulate an
(B) Its cost is too great for 90% of AIDS sufferers immune response.
(C) It often has severe side effects
(D) Some HIV strains are resistant to it 4. A 54-year-old man developed a slowly worsening
(E) All of the above headache
Jawetz Medical Microbiology 27th ed.
followed by gradual, progressive weakness in his (A) No diagnostic value because this mold is an
right arm. A brain scan revealed a left cerebral airborne
lesion. At surgery an abscess surrounded by contaminant.
granulomatous material was found. Sections of (B) Consider treatment for rhinocerebral
the tissue and subsequent culture showed darkly mucormycosis
pigmented septate hyphae indicating (zygomycosis).
phaeohyphomycosis. Tis infection may be caused (C) Strongly suggestive of ketoacidosis.
by species of which genus below? (D) Strongly suggestive of HIV infection.
(A) Aspergillus (E) Te patient has been exposed to indoor mold
(B) Cladophialophora contamination.
(C) Coccidioides
(D) Malassezia 8. An 8-year-old boy develops a circular dry, scaly,
(E) Sporothrix and pruritic lesion on his leg. What is the diagnostic
signifcance of observing branching, septate,
5. A 35-year-old man is a farmer in a tropical area nonpigmented hyphae in a potassium
of West Africa. He developed a persistent scaly hydroxide/calcofluor white preparation of a scraping
papule on his leg. Ten months later a new crop of from this skin lesion?
wart-like purplish scaly lesions appeared. (A) Chromomycosis
Tese lesions slowly progressed to a cauliflower-like (B) Dermatophytosis
appearance. Chromoblastomycosis (C) Phaeohyphomycosis
(chromomycosis) was diagnosed. (D) Sporotrichosis
Which statement regarding this disease is most (E) No diagnostic signifcance
correct?
(A) In tissue, the organisms convert to spherical 9. Which statement regarding the epidemiology of
cells that candidiasis is
reproduce by fission and exhibit transverse correct?
septations. (A) Patients receiving bone marrow transplants are
(B) The etiologic agents are endogenous members not at risk
of the mammalian flora and possess melanized cell for systemic candidiasis.
walls. (B) Patients with impaired or low numbers of
(C) The disease is caused by a single species. neutrophils and
(D) Most infections are systemic. monocytes are not at risk for systemic candidiasis.
(E) Most infections are acute and clear (C) Patients with any form of diabetes have
spontaneously. enhanced resistance to candidiasis.
(D) Patients with AIDS frequently develop
6. A 42-year-old HIV-positive male, originally from mucocutaneous
Vietnam but candidiasis, such as thrush.
now residing in Tucson, Arizona, presents with a (E) Pregnancy lowers the risk of candidal vaginitis.
painful ulcerative lesion on his upper lip (cheilitis). A
biopsy was obtained, and the histopathologic slide 10. Which statement regarding dermatophytosis is
(hematoxylin and eosin stain) correct?
revealed spherical structures (20–50 μm in (A) Chronic infections are associated with zoophilic
diameter) with dermatophytes, such as Microsporum canis.
thick refractory cell walls. What is the likely disease (B) Acute infections are associated with zoophilic
consistent dermatophytes, such as Microsporum canis.
with this finding? (C) Chronic infections are associated with
(A) Infection with Penicillium marneffei anthropophilic dermatophytes, such as
(B) Cryptococcosis Microsporum canis.
(C) Blastomycosis (D) Acute infections are associated with
(D) Coccidioidomycosis anthropophilic dermatophytes, such as
(E) No diagnostic significance Microsporum canis.

7. A 47-year-old man with poorly controlled 11. Which statement regarding the laboratory
diabetes mellitus developed a bloody nasal identifcation of
discharge, facial edema, and necrosis of his nasal fungi is correct?
septum. Culture of his cloudy nasal secretions (A) Histoplasma capsulatum typically requires less
yielded Rhizopus species. What is the most than 48 hours of incubation to yield positive cultures
important implication of this finding? from clinical specimens.
Jawetz Medical Microbiology 27th ed.
(B) Since many saprobic (nonpathogenic) molds 14. A 37-year-old man with AIDS, currently living in
resemble dimorphic mycotic agents in culture at Indianapolis, Indiana, presented with osteomyelitis
30°C, the identification of putative dimorphic of the lef hip. A needle biopsy of the bone marrow
pathogenic fungi must be confirmed by conversion was obtained, and the calcofluor white smear
to the tissue form in vitro or by the detection of revealed a variety of myelogenous cells,
species-specifc antigens or DNA sequence monocytes, and macrophages containing numerous
analysis. intracellular yeast cells that were elliptical and
(C) Molds are routinely speciated by a battery of approximately 2 × 4 μm. What is the most likely
physiologic tests, such as the ability to assimilate diagnosis?
various sugars. (A) Blastomycosis
(D) A positive germ tube test provides a rapid (B) Candidiasis
presumptive identifcation of Candida glabrata. (C) Cryptococcosis
(E) Budding yeast cells and abundant (D) Histoplasmosis
pseudohyphae are typical of Pneumocystis jiroveci. (E) No diagnostic significance

12. A 28-year-old female sex worker from southern 15. The potassium hydroxide examination of
California complained of headaches, dizziness, and sputum from a heart transplant patient with fever
occasional episodes of “spacing out” during the and pulmonary infiltrates contains oval budding
past 2 weeks. A lumbar puncture revealed reduced yeast cells and pseudohyphae. What is the
sugar, elevated protein, and 450 mononuclear diagnostic significance?
leukocytes per milliliter. She was seropositive for (A) Aspergillosis
HIV. Her history is compatible with fungal meningitis (B) Candidiasis
due to Cryptococcus neoformans, Coccidioides (C) Hyalohyphomycosis
posadasii, or a species of Candida. Which one of (D) Phaeohyphomycosis
the following tests is confrmatory? (E) No diagnostic significance
(A) Meningitis due to Coccidioides posadasii would
be confrmed by a positive test of the CSF for 16. A middle-aged male resident of southern
cryptococcal capsular antigen. California received a liver transplant. During the
(B) Meningitis due to Cryptococcus neoformans following months, he gradually developed fatigue,
would be confrmed by a positive test of the CSF for weight loss, cough, night sweats, dyspnea, and a
complement fixation nonhealing subcutaneous nodule on his nose. The
antibodies to coccidioidin. chest radiograph revealed hilar lymphadenopathy
(C) Meningitis due to a species of Candida would and diffuse infiltrates. Direct examination and
be confrmed culture of a respiratory specimen were negative.
by the microscopic observation of oval yeast cells Skin tests with PPD, blastomycin, coccidioidin, and
and pseudohyphae in the CSF. histoplasmin were negative. Serologic test results
(D) Meningitis due to Coccidioides posadasii would were as follows: Negative serum test for
be confrmed by a positive skin test to coccidioidin. cryptococcal capsular antigen in blood, positive
immunodiffusion test for serum precipitins to fungal
13. Which statement about phaeohyphomycosis is antigen F, and negative immunodiffusion tests for
correct? precipitins to antigens h, m, and A. Serum tests for
(A) The infection only occurs in immunocompetent fungal complement fixation antibodies were
patients. negative for Blastomyces dermatitidis, as well as
(B) Infected tissue reveals branching, septate both the mycelial and yeast antigens of
nonpigmented Histoplasma capsulatum but yielded a titer of 1:32
hyphae. to coccidioidin. Which interpretation of these data is
(C) The causative agents are members of the the most tenable?
normal microbial (A) Clinical and serologic findings are inconclusive.
flora and can be isolated readily from the skin and (B) Clinical and serologic findings are most
mucosa consistent with active disseminated histoplasmosis.
of healthy persons. (C) Clinical and serologic findings are most
(D) Phaeohyphomycosis may exhibit several clinical consistent with active disseminated blastomycosis.
manifestations, including subcutaneous or systemic (D) Clinical and serologic findings are consistent
disease, as well as sinusitis. with active disseminated coccidioidomycosis.
(E) Cases rarely respond to treatment with (E) Clinical and serologic findings exclude a
itraconazole. diagnosis of blastomycosis, histoplasmosis, and
coccidioidomycosis.
Jawetz Medical Microbiology 27th ed.
17. Which statement regarding aspergillosis is yeast cells with broad connections between the
correct? parent
(A) Patients with allergic bronchopulmonary yeast and bud.
aspergillosis rarely have eosinophilia. (E) All cases require treatment with amphotericin B.
(B) Patients receiving parenteral corticosteroids are
not at risk for invasive aspergillosis. 21. Which statement regarding dermatophytosis is
(C) The diagnosis of pulmonary aspergillosis is correct?
frequently (A) Chronic infections are associated with zoophilic
established by culturing Aspergillus from the dermatophytes, such as Trichophyton rubrum.
sputum and (B) Acute infections are associated with zoophilic
blood. dermatophytes, such as Trichophyton rubrum.
(D) The clinical manifestations of aspergillosis (C) Chronic infections are associated with
include local anthropophilic dermatophytes, such as
infections of the ear, cornea, nails, and sinuses. Trichophyton rubrum.
(E) Bone marrow transplant recipients are not at (D) Acute infections are associated with
risk for invasive aspergillosis. anthropophilic dermatophytes, such as
Trichophyton rubrum.
18. Which statement regarding sporotrichosis is
correct? 22. Which statement regarding
(A) The most common etiologic agent is paracoccidiomycosis is not correct?
Pseudallescheria boydii (Scedosporium (A) The etiologic agent is a dimorphic fungus.
apiospermum). (B) Most patients acquired their infections in South
(B) The etiologic agent is a dimorphic fungus. America.
(C) The ecology of the etiologic agent is unknown. (C) Although the infection is acquired by inhalation
(D) Most cases are subcutaneous and and is initiated in the lungs, many patients develop
nonlymphangitic. cutaneous and mucocutaneous lesions.
(E) Most patients are immunocompromised. (D) The vast majority of patients with active disease
are males.
19. A 24-year-old, HIV-negative migrant worker (E) The etiologic agent is inherently resistant to
from Colombia amphotericin B
presented with a painful ulcerative lesion on the
tongue. The 23. Your kidney transplant patient has developed
edge of the lesion was gently scraped and a nosocomial systemic candidiasis, but the patient’s
calcofluor white potassium hydroxide smear isolate of Candida glabrata is resistant to
revealed tissue cells, debris, and several large, fluconazole. A reasonable alternative would be
spherical, multiply budding yeast cells. Based on oral administration of:
this observation, what is the most likely diagnosis? (A) Flucytosine
(A) Blastomycosis (B) Posaconazole
(B) Candidiasis (C) Griseofulvin
(C) Coccidioidomycosis (D) Amphotericin B
(D) Histoplasmosis
(E) Paracoccidioidomycosis 24. Which one of the following antifungal drugs
does not target the
20. Which statement about blastomycosis is biosynthesis of ergosterol in the fungal membrane?
correct? (A) Voriconazole
(A) Similar to other endemic mycoses, this infection (B) Itraconazole
occurs (C) Terbinafne
equally in men and women. (D) Fluconazole
(B) Infection starts in the skin, and the organisms (E) Micafungin
commonly
disseminate to the lungs, bone, genitourinary tract, 25. Which one of the following pathogenic yeasts is
or not a common
other sites. member of the normal human flora or microbiota?
(C) The disease is endemic to certain areas of (A) Candida tropicalis
South America. (B) Malassezia globosa
(D) In tissue, one finds large, thick-walled, single (C) Cryptococcus neoformans
budding (D) Candida glabrata
(E) Candida albicans
Jawetz Medical Microbiology 27th ed.
(C) Cyclosporiasis
(D) Hookworm infection
(E) Trichuriasis
Section VI: Parasitology
(F) Ascariasis

Chapter 46: Medical Parasitology 5. Pathologic effects of flariae in humans are


1. A mother states that she has observed her 4- caused by the adult
year-old son worms in all but one species. In this case, the
scratching his anal area frequently. Te most likely principal damage
cause of this is caused by the microfilariae of
condition is (A) Brugia malayi
(A) Trichomonas vaginalis (B) Mansonella ozzardi
(B) Enterobius vermicularis (C) Dracunculus medinensis
(C) Ascaris lumbricoides (D) Wuchereria bancrofti
(D) Necator americanus (E) Onchocerca volvulus
(E) Entamoeba histolytica
6. An 18-year-old male complains of abdominal
2. Chagas disease is especially feared in Latin pain, bloating, frequent loose stools, and loss of
America because of energy. He returned a month ago from a 3-week
the damage that can occur to the heart and hiking and camping trek to the Mount Everest Base
parasympathetic nervous system and the lack of an Camp in Nepal. The trek involved only high-
effective drug for the symptomatic later stages. elevation hiking, since he flew in and out of the
Your patient is planning to reside in a Venezuelan 12,000-f starting point.
village for 1–2 years. Which one of the following Which of the following is an important consideration
suggestions for the diagnosis?
would be of special value for avoiding Chagas (A) Exposure to high-level UV radiation
disease? (B) The source and purification of water
(A) Boil or treat all of your drinking water. (C) The use of insect repellents while hiking
(B) Sleep under a bed net. (D) Te presence of domestic animals en route
(C) Do not keep domestic pets in your house. (E) Te degree of contact with villagers en route
(D) Never walk barefoot in the village compound.
(E) Do not eat lettuce or other raw vegetables or 7. Which one of the following diagnostic tests
unpeeled fruit. should be conducted for the patient in Question 6?
(A) Blood and urine bacteriologic examination
3. A sexually active 24-year-old woman complains (B) Series of ova and parasite tests and fecal
of vaginal itching and vaginal discharge. To verify smears
your tentative diagnosis of (C) ELISA or hemagglutination serologic tests for
trichomoniasis, you should include which of the malaria
following in (D) Skin snip microflarial test
your workup? (E) Endoscopic exam for whipworms
(A) Specifc serologic test
(B) Ova and parasite fecal smear 8. The parasite most likely to be responsible for the
(C) Wet mount of vaginal fluid illness of the
(D) Enzyme-linked immunoassay (ELISA) test of patient in Question 6 is
serum (A) Leishmania major
(E) Stool culture (B) Plasmodium vivax
(C) Trichomonas vaginalis
4. You are working in a rural medical clinic in China (D) Naegleria gruberi
and a 3-yearold girl is brought in by her mother. Te (E) Giardia lamblia
child appears emaciated and, upon testing, is found
to have a hemoglobin level of 5 9. Several Papua New Guinea villagers known to
g/dL. Her feet and ankles are swollen, and there is eat pork during
an extensive celebrations were reported to be suffering from an
rash on her feet, ankles, and knees. Te most likely outbreak of
parasitic epileptiform seizures. One of the frst things you
infection that causes the child’s condition is should investigate is
(A) Schistosomiasis (A) The prevalence of Ascaris infections in the
(B) Cercarial dermatitis population
Jawetz Medical Microbiology 27th ed.
(B) The prevalence of schistosomiasis in the
population 14. A 37-year-old sheep farmer from Australia
(C) The presence of Trypanosoma brucei presents with upper
gambiense in the right quadrant pain and appears slightly jaundiced.
villagers A stool
(D) Te presence of Giardia cysts in the drinking exam was negative for ova and parasites but a CT
water scan of the
(E) Te presence of Taenia solium in the pigs liver reveals a large 14-cm cyst that appears to
contain fluid.
10. A 32-year-old male tourist traveled to Senegal Which of the following parasites should be
for 1 month. During the trip, he swam in the Gambia considered?
river. Two months after his return, he began (A) Toxoplasma gondii
complaining of intermittent lower abdominal pain (B) Taenia solium
with dysuria. Laboratory results of ova and (C) Taenia saginata
parasites revealed eggs with a terminal spine. (D) Clonorchis sinensis
Which of the following (E) Schistosoma mansoni
parasites is the cause of the patient’s symptoms? (F) Echinococcus granulosus
(A) Toxoplasma gondii (G) Paragonimus westermani
(B) Schistosoma mansoni
(C) Schistosoma haematobium 15. An apparently fatigued but alert 38-year-old
(D) Ascaris lumbricoides woman has spent 6
(E) Taenia solium months as a teacher in a rural Thailand village
school. Her chief
11. What type of specimen was collected for complaints include frequent headaches, occasional
laboratory analysis based on the answer in the nausea and
previous question? vomiting, and periodic fever. You suspect malaria
(A) Tick blood smear and indeed
(B) Stool sample fnd parasites in red blood cells in a thin blood
(C) Urine sample smear. To rule
(D) Blood for serology out the dangerous falciparum form of malaria, which
(E) Sputum sample one of the
following choices is NOT consistent with a
12. A previously healthy 23-year-old woman diagnosis of Plasmodium falciparum malaria based
recently returned from her vacation afer visiting on a microscopic examination of the blood smear?
friends in Arizona. She complained of severe (A) Red blood cells containing trophozoites with
headaches, saw “flashing lights,” and had a Schuffner’s
purulent nasal discharge. She was admitted into the dots
hospital with a diagnosis of bacterial meningitis and (B) Red blood cells containing >1 parasite per RBC
died 5 days later. (C) Banana-shaped or crescent-shaped
Which of the following parasites should have been gametocytes
considered in the diagnosis? She had no prior (D) Parasites within normal-sized red blood cells
history of travel outside of the United States. (E) Parasites with double nuclei
(A) Plasmodium falciparum 16. Given a diagnosis of uncomplicated
(B) Toxoplasma gondii Plasmodium falciparum
(C) Strongyloides stercoralis malaria for the patient in Question 15, which one of
(D) Entamoeba histolytica the following treatment regimens is appropriate
(E) Naegleria fowleri where chloroquineresistance is known?
(A) Oral artemisinin-based combination therapy
13. How could the person have acquired the (ACT)
parasite in Question 12? (B) Oral chloroquine
(A) Ingesting cysts from fecally contaminated (C) Intravenous chloroquine
drinking water (D) Oral proguanil
(B) Eating improperly cooked fsh (E) Intravenous quinidine
(C) Eating improperly cooked beef 17. Given a diagnosis of Plasmodium falciparum,
(D) Walking barefoot in the park you should tell the
(E) Engaging in unprotected sexual intercourse patient in Question 15 that (select one)
(F) Getting bitten by a sandfly (A) Relapse occurs with Plasmodium vivax and
(G) Plunging into a natural hot spring Plasmodium
Jawetz Medical Microbiology 27th ed.
ovale, not Plasmodium falciparum and therefore no (C) One positive peripheral vein blood culture and
treatment for hypnozoites is necessary. one positive central line blood culture with
(B) Primaquine is used to prevent relapse of Escherichia coli
Plasmodium (D) One positive central venous line blood culture
falciparum. with a Corynebacterium species and two negative
(C) Returning to the tropics would be dangerous peripheral vein blood cultures
because (E) Two positive central line blood cultures with
hypersensitivity to the parasite may have Candida albicans
developed.
(D) Te use of insecticide treated bednets in 2. Two days ago, a 22-year-old man returned from
endemic areas is a 2-week trip to Mexico. Within 24 hours, he
not necessary since she already had malaria. developed diarrhea. Which of the following will not
(E) It is not necessary for her to take antimalarials establish the etiology of his diarrhea?
when traveling in endemic areas. (A) Stool culture for Salmonella, Shigella, and
18. A 52-year-old male, returning from a travel tour Campylobacter
in India and (B) Stool culture for rotavirus and Norwalk-like virus
Southeast Asia, was diagnosed with intestinal (C) Stool enzyme immunoassay for Giardia lamblia
amebiasis and antigen
successfully treated with iodoquinol. A month later, (D) Stool examination for Entamoeba histolytica
he returned
to the clinic complaining of the following conditions. 3. A 37-year-old man traveled to Peru during the
Which of time of the cholera epidemic. One day after
the conditions is most likely the result of systemic returning home, he developed severe watery
amebiasis diarrhea. To enhance the isolation of Vibrio
(even though the intestinal infection appears to be cholerae from his stool, the laboratory needs to
cured)? include:
(A) High periodic fever (A) MacConkey agar
(B) Bloody urine (B) Campylobacter blood agar
(C) Tender, enlarged liver (C) Tiosulfate citrate bile salts sucrose agar
(D) Draining skin lesion (D) Bismuth sulfte agar
(E) Enlarged painful spleen (E) Hektoen agar

4. A 42-year-old man is known to have HIV/AIDS.


Which of the following is the most appropriate
Section VII: Diagnostic Medical
method to follow the progress of his highly active
Microbiology antiretroviral therapy (HAART)?
Chapter 47: Principles of Diagnostic Medical (A) Determination of viral load
Microbiology (B) Following anti-HIV-1 antibody levels
(C) Using Western blot to assess his anti-p24 levels
1. A 47-year-old woman had a bone marrow (D) Repeated culture of his blood for HIV-1 to
transplant as part of her treatment for chronic determine when the culture becomes negative
myelogenous leukemia. While in the hospital she (E) Genotyping of his HIV-1 isolate to determine its
had a central venous catheter in place for antiretroviral susceptibility
administration of fluids. In the time following the
transplant, but before it had engrafted, the patient 5. A 2-year-old child develops diarrhea. Rotavirus
had a very low white blood cell count. She infection is suspected. Which of the following would
developed a fever, and blood cultures were done. be most useful in diagnosing a rotavirus infection?
Which of the following scenarios suggests that the (A) Fluorescent antibody staining of the stool
positive blood cultures resulted from a specimen
contaminant? (B) Light microscopy to detect mucosal cells with
(A) Two positive peripheral vein blood cultures with cytopathic effect
Staphylococcus aureus (C) Detection of virus antigen in stool by enzyme-
(B) Two positive peripheral vein blood cultures with linked immunosorbent assay
Staphylococcus epidermidis along with two positive (D) Virus culture
central line blood cultures with Staphylococcus
epidermidis 6. Which of the following is appropriate to determine
the etiologic diagnosis of infection?
(A) Culture and identifcation of the agent
Jawetz Medical Microbiology 27th ed.
(B) DNA–DNA or DNA–RNA hybridization to detect (E) Complement fxation test of serum for antibodies
pathogen-specifc genes in patients’ specimens against Coccidioides immitis
(C) Demonstration of a rise in antibody titer or cell-
mediated immune response to an infectious agent 10. A 5-year-old kidney transplant patient being
(D) Morphologic identifcation of the agent in stains treated with cyclosporine develops a
of specimens or sections of tissues by light or lymphoproliferative disorder. Which of the
electron microscopy following viruses is most likely responsible for this
(E) Detection of antigen from the agent by disorder?
immunologic assay (A) Cytomegalovirus
(F) All of the above (B) Herpes simplex virus
(C) Coxsackie B virus
7. A 45-year-old woman is admitted to the hospital (D) Hepatitis B virus
because of fever, a 6-kg weight loss, and a new (E) Epstein-Barr virus
heart murmur. Probable endocarditis is diagnosed.
How many blood cultures over what period of time 11. All of the following are appropriate indications
should be done to provide evidence of specific for the use of serologic tests for viruses except:
bacterial infection in endocarditis? (A) As an indication of one’s susceptibility to a
(A) One particular viral infection
(B) Two over 10 minutes (B) For diagnosis when the virus has a long
(C) Tree over 2 hours incubation period
(D) Tree over 24 hours (C) For screening purposes
(E) Six over 3 days (D) For confrmation of a viral infection
(E) To monitor the response to treatment
8. A 4-year-old boy develops bloody diarrhea.
Hemorrhagic colitis due to Escherichia coli O157:H7 12. In August, a 2-year-old boy presents acutely
is suspected. What medium should be inoculated to with fever, signs of headache, decreased mental
help the laboratory staff make the diagnosis of this status, and neck stiffness. On physical examination
infection? the fever is confrmed, mild nuchal rigidity is present,
(A) Blood agar and although the child is irritable and mildly
(B) Sorbitol MacConkey agar somnolent, he is arousable and is taking some oral
(C) Hektoen enteric agar fluids. The cerebrospinal fluid parameters reveal a
(D) CIN (cefsulodin, irgasan, novobiocin) agar protein of 60 μg/dL, glucose of 40 μg/dL, and a total
(E) Tiosulfate citrate bile salts sucrose agar of 200 WBCs, predominately mononuclear. The
most likely cause of this child’s infection is:
9. A 43-year-old black man frequently drove his (A) Bacterial
long-haul 18-wheel truck through the Central Valley (B) Viral
of California. Two months ago there was a major (C) Protozoan
windstorm while he was driving through the Valley. (D) Fungal
Two weeks afer that he developed fever with a (E) Mycobacterial
cough and pleuritic chest pain. An infltrate
was seen on chest x-ray. Pneumonia was 13. In the case above, the most useful test for
diagnosed, and the patient was given erythromycin. making a rapid defnitive diagnosis of the most likely
The fever, cough, pleuritic pain, and infltrate cleared causative agent is:
over a 3-week period. Two weeks ago, he (A) An antigen test for Streptococcus pneumoniae
developed progressively severe headaches, and for (B) A latex agglutination test for cryptococcal
the past 2 days he has had vomiting. His antigen
cerebrospinal fluid contains 150 white blood cells (C) A nucleic acid amplifcation test for viral RNA
per microliter, predominantl lymphocytes, and the detection
glucose concentration is low. Meningitis due to (D) Culture on selective media combined with a
Coccidioides immitis is suspected. Which of the probe test for
following tests is the most sensitive and useful in confrmation
confrming this diagnosis? (E) Giemsa-stained smear of cerebrospinal fluid
(A) Test for coccidioidal antigens performed on CSF
(B) Test of cerebrospinal fluid for antibodies against 14. Susceptibility testing using an MIC method, as
Coccidioides immitis opposed to disk
(C) PCR for Coccidioides immitis DNA diffusion, is preferred for all of the following types of
(D) Cerebrospinal fluid culture for Coccidioides infections
immitis except:
Jawetz Medical Microbiology 27th ed.
(A) Urinary tract infections 20. The proportion of antibiotic resistant bacteria
(B) Endocarditis has increased
(C) Osteomyelitis along with the widespread use of antibiotics. Tis is
(D) Bacteremia in a neutropenic patient due to the
(E) Bacterial meningitis fact that antibiotics:
(A) Are unstable in vivo
15. Bacterial vaginosis is best diagnosed by all of (B) Act as agents of selection for resistant
the following organisms
except: (C) Are mainly bacteriostatic in vivo
(A) Measurement of the vaginal pH (D) Are powerful mutagens
(B) Detection of a fshy odor when the discharge is
alkalinized
with KOH
(C) Bacterial culture for aerobes and anaerobes
(D) Examination of a Gram-stained smear for “clue
cells”

16. A 45-year-old man presents to the emergency


room with a
3-day history of cough productive of blood-tinged
sputum and
a fever. Te Gram stain of the sputum has many
white blood
cells and gram-positive diplococci. Te most likely
causative
organism is
(A) Staphylococcus aureus
(B) Streptococcus pneumoniae
(C) Mycoplasma pneumoniae
(D) Klebsiella pneumonia

17. How many organisms must be present in a


clean catch
midstream urine sample to be considered indicative
of an
infection?
(A) > 102 CFU/ml
(B) > 103 CFU/ml
(C) > 104 CFU/ml
(D) > 105 CFU/ml

18. Common contaminants of blood cultures


include:
(A) Gram-negative rods
(B) Coagulase-negative staphylococci
(C) Staphylococcus aureus
(D) Anaerobes

19. Which of the following specimens does not


usually contain
anaerobes?
(A) Aspiration from an infected maxillary sinus
(B) Troat swab from a patient with a sore throat
(C) Cerebrospinal fluid from a patient with
meningitis
(D) Expectorated sputum from a patient with
communityacquired pneumonia
Jawetz Medical Microbiology 27th ed.
9. D
ANSWERS 10. E
Chapter 1
1. A Chapter 6
2. D 1. D
3. A 2. D
4. A 3. C
5. E 4. B
6. B 5. C
7. E 6. E
8. D 7. D
9. D 8. C
10.C 9. E
10. C
Chapter 2 11. A
1. C 12. E
2. A
3. D Chapter 7
4. C 1. E
5. B 2. D
6. B 3. C
7. C
4. E
8. B
5. B
9. D
10. E 6. C
7. B
Chapter 3
1. C Chapter 8
2. D 1. C
2. D
3. E
3. B
4. B
4. A
5. D 5. D
6. E 6. B
7. B
Chapter 4 8. B
1. A 9. C
2. C 10. B
3. C 11. A
4. C 12. B
5. C 13. D
14. A
6. A
15. C
7. E
16. A
8. C 17. C
9. A 18. B
10. B 19. D
11. E 20. C

Chapter 5 Chapter 9
1. C 1. E
2. D 2. B
3. B 3. E
4. A 4. D
5. D 5. A
6. B 6. C
7. B 7. E
8. E 8. A
Jawetz Medical Microbiology 27th ed.
9. D 14. C
10. A 15. A
11. B
12. D
13. D Chapter 13
14. B 1. D
15. D 2. E
3. A
Chapter 10 4. C
1. E 5. E
2. C 6. D
3. D 7. E
4. A 8. D
5. D 9. D
6. E 10. C
7. A 11. E
8. B 12. A
9. D 13. D
10. B 14. C
11. D 15. D
12. B
13. D Chapter 14
14. D 1. C
15. C 2. E
3. B
Chapter 11 4. E
1. C 5. C
2. A 6. A
7. B
3. E
8. D
4. C
9. E
5. D 10. C
6. A 11. E
7. C 12. C
8. D 13. A
9. E 14. B
10. C 15. C
11. B
Chapter 15
12. D
1. E
13. A
2. D
14. B 3. B
15. D 4. B
5. C
Chapter 12 6. B
1. D 7. E
2. B 8. C
3. E 9. A
4. A 10. D
5. D 11. C
6. E 12. A
7. C 13. E
8. A 14. B
9. D 15. C
10. B
11. E Chapter 16
12. D 1. E
13. D 2. E
Jawetz Medical Microbiology 27th ed.
3. B 5. C
4. B 6. A
5. D 7. D
6. A 8. C
7. B 9. B
8. B 10. C
9. E 11. C
10. A 12. D
13. E
Chapter 17 14. A
1. E 15. A
2. A
3. E Chapter 21
4. A 1. C
5. D 2. B
6. D 3. D
7. D 4. A
8. D 5. E
9. E 6. A
10. B 7. E
11. B 8. D
12. A 9. D

Chapter 18 Chapter 22
1. D 1. B
2. B 2. B
3. E 3. A
4. B 4. E
5. A 5. E
6. E 6. E
7. C 7. B
8. D 8. C
9. A 9. B
10. C 10. A
11. B 11. B
12. C 12. D
13. C 13. B
14. A 14. D
15. D 15. C

Chapter 19 Chapter 23
1. A 1. C
2. E 2. B
3. E 3. B
4. C 4. E
5. D 5. A
6. E 6. E
7. A 7. B
8. B 8. D
9. C 9. B
10. A 10. A
11. D 11. E
12. C
Chapter 20 13. B
1. D 14. E
2. A 15. D
3. D
4. D Chapter 24
Jawetz Medical Microbiology 27th ed.
1. A 1. D
2. E 2. B
3. D 3. E
4. A 4. D
5. A 5. A
6. E 6. C
7. D 7. B
8. D 8. E
9. A 9. E
10. B 10. C
11. B
Chapter 25 12. D
1. B 13. D
2. E 14. E
3. C 15. D
4. E
5. C Chapter 29
6. D 1. C
7. E 2. C
8. E 3. D
9. D 4. A
10. B 5. B
6. C
Chapter 26 7. B
1. D 8. C
2. E 9. C
3. A 10. D
4. B 11. A
5. A 12. D
6. C 13. D
7. B 14. B
8. A 15. C
9. C
10. D
11. C
12. A Chapter 30
13. E 1. C
14. B 2. D
15. E 3. B
4. D
Chapter 27 5. E
1. A 6. D
2. B 7. B
3. C 8. C
4. C 9. D
5. C 10. A
6. C 11. C
7. A 12. D
8. B 13. D
9. E 14. B
10. B 15. C
11. D
12. C Chapter 31
13. B 1. B
14. A 2. C
15. B 3. B
4. A
Chapter 28 5. D
Jawetz Medical Microbiology 27th ed.
6. B 14. A
7. D
8. C Chapter 35
9. A 1. B
10. C 2. C
3. A
Chapter 32 4. B
1. E 5. C
2. D 6. D
3. B 7. E
4. D 8. A
5. A 9. D
6. C 10. A
7. D 11. G
8. B 12. D
9. B 13. C
10. A 14. B
11. C 15. C
12. D 16. C
13. C 17. D
14. D 18. B
19. E
Chapter 33 20. D
1. D
2. A Chapter 36
3. D 1. D
4. B 2. E
5. A 3. B
6. E 4. A
7. D 5. C
8. A 6. D
9. C 7. C
10. C 8. B
11. B 9. C
12. B 10. E
13. D 11. D
14. A 12. B
15. B 13. C
16. B 14. B
17. C 15. C
18. B
19. D Chapter 37
20. B 1. C
2. C
Chapter 34 3. B
1. E 4. D
2. E 5. A
3. B 6. B
4. A 7. D
5. C 8. A
6. D 9. D
7. C 10. B
8. D 11. C
9. D 12. A
10. E
11. B Chapter 38
12. B 1. D
13. D 2. C
Jawetz Medical Microbiology 27th ed.
3. D 5. C
4. E 6. A
5. B 7. C
6. B 8. C
7. A 9. D
8. E
9. C Chapter 42
10. C 1. E
11. D 2. E
12. A 3. B
13. C 4. D
14. A 5. C
15. C 6. E
7. E
Chapter 39 8. B
1. D 9. B
2. E 10. A
3. A 11. A
4. E 12. D
5. C 13. C
6. B 14. C
7. D 15. B
8. B
9. C Chapter 43
10. C 1. B
11. A 2. C
12. D 3. C
13. D 4. B
14. A 5. A
15. D 6. D
16. A 7. D
8. B
Chapter 40 9. E
1. C 10. A
2. E 11. D
3. B 12. C
4. B 13. D
5. C
6. A Chapter 44
7. D 1. C
8. B 2. B
9. D 3. D
10. A 4. D
11. C 5. A
12. B 6. B
13. D 7. C
14. B 8. A
15. C 9. E
16. A 10. C
17. B 11. C
18. A 12. C
19. A 13. D
14. E
Chapter 41 15. D
1. E
2. B Chapter 45
3. B 1. D
4. D 2. B
Jawetz Medical Microbiology 27th ed.
3. C 14. A
4. B 15. C
5. A 16. B
6. D 17. D
7. B 18. B
8. B 19. C
9. D 20. B
10. B
11. B
12. C
13. D
14. D
15. E
16. D
17. D
18. B
19. E
20. D
21. C
22. E
23. B
24. E
25. C

Chapter 46
1. B
2. B
3. C
4. D
5. E
6. B
7. B
8. E
9. E
10. C
11. C
12. E
13. G
14. F
15. A
16. A
17. A
18. C

Chapter 47
1. D
2. B
3. C
4. A
5. C
6. F
7. D
8. B
9. B
10. E
11. E
12. B
13. C

You might also like